Packrat 13

You might also like

Download as pdf or txt
Download as pdf or txt
You are on page 1of 83

ACKNOWLEDGEMENT

The time and effort provided by the following individuals who served as members of this committee are
greatly appreciated:
Kimberly Cavanagh, MPAS, PA-C, Project Director
Mark Archambault, DHSc, MHS, RPA-C
Jonathan Bowser, MS, PA-C
Petar Breitinger, MPAS, PA-C
Christine Bruce, MHSA, PA-C
Denise Dillingham, MPAS, PA-C
Kathleen Ehrhardt, MMS, PA-C
Ralph Rice, MPAS, PA-C
Eric Vangsnes, PhD, PA-C
James VanRhee, MS, PA-C
Donna Yeisley, MEd, PA-C

Subcommittee
Rex Hobbs, MPAS, PA-C
Alan Gindoff, DHSc, PA-C
Ryan Junsay, MS, PA-C
Marc Maller, MD

DEDICATION
This examination would not have been possible without the years of commitment of the MR. TIB
Development Committee. Numerous PA educators from across the nation provided their experience and
insight as questions for MR. TIB. It has been this data bank that served as the building blocks for
PACKRAT.
PAEA is proud to be able to continue in the tradition of quality fostered by the forerunners of the selfassessment examination for physical assistants. It is our honor to dedicate PACKRAT to:
Jesse C. Edwards, MS
Claire S. Parker, PhD
University of Nebraska, Physician Assistant Program

Copyright 2008. Physician Assistant Education Association

PHYSICIAN ASSISTANT EDUCATION ASSOCIATION


Physician Assistant Clinical Knowledge Rating and Assessment Tool (PACKRAT)
Form 13
Directions and Explanations

TABLE OF CONTENTS

I.

Introduction

II.

Explanation of the Score Report

Page 1: Scores Your total Score and Group Comparisons


Page 2: Your STRENGTHS, Weaknesses, and Quality of Responses
Page 3: Your Individual and Correct Responses
Page 4: Your Responses by Task and Category
Page 5-6: Your Profile Comparison: Demographic Profile

2
2
2
2
2

III.

Recommendations for Using the Feedback Package

IV.

Study Resources

V.

Answer Key

VI.

Examination Explanations

VII.

Comment Form

Copyright 2008. Physician Assistant Education Association

82

Copyright 2008. Physician Assistant Education Association. All rights reserved. No part of this publication may be reproduced or
transmitted in any form or by any means, electronic or mechanical, including photocopy or recording, or any information and
retrieval system, without permission in writing from the Physician Assistant Education Association.

PHYSICIAN ASSISTANT CLINICAL KNOWLEDGE RATING


AND ASSESSMENT TOOL (PACKRAT)

I. Introduction
The Physician Assistant Clinical Knowledge Rating and Assessment Tool (PACKRAT) was developed by
a volunteer committee of experts and is based on the content outline of a nationally recognized
competency examination. The following is a description of the content of PACKRAT:

PACKRAT EXAMINATION MATRIX


PACKRAT EXAMINATION MATRIX
CONTENT AREA
NUMBER OF ITEMS
CONTENT AREA
NUMBER OF ITEMS
1. History & Physical
36
1. History
& Physical
26 32
2. Diagnostic
Studies
2. Diagnostic
Studies
36 41
3. Diagnosis
3. Diagnosis
45 22
4. Health Maintenance
4. Health
Maintenance
22 31
5. Clinical
Intervention
5. Clinical
Intervention
32 41
6. Clinical
Therapeutics
6. Clinical
Therapeutics
48 22
7. Scientific
Concepts
7. Scientific Concepts
16
TOTALS
225
TOTALS
225
Additionally, questions also apply to the following clinical specialties:
Additionally, questions also apply to the following clinical specialties:
A. Cardiology
J. Obstetrics/Gynecology
A. Cardiology
Neurology
B. Dermatology
K. I.Orthopedics/Rheumatology
B. Dermatology
Obstetrics/Gynecology
C. Endocrinology
M.J.Psychiatry/Behavioral
Medicine
C. Endocrinology
Orthopedics/Rheumatology
F. Gastrointestinal/Nutritional
N. K.
Pulmonology
D. ENT
Pediatrics
H. Hematology
P. L.
Urology/Renal
E. Ophthalmology
M.
Psychiatry/Behavioral
Medicine
I. Neurology
Q. Infectious
Diseases
F. Gastrointestinal/Nutritional
N. Pulmonology
G. Geriatrics
O. Surgery
H. Hematology
P. Urology/Renal

The task and specialty categories for each item are listed in the answer key on page 5; your feedback
package contains a breakdown of responses by the task and clinical specialty category. Pay particular
attention to the questions you answered incorrectly and determine the specialty for that question and use
this information to identify weaknesses.

Copyright 2008. Physician Assistant Education Association

The PACKRAT provides a detailed feedback report of performance and it is available to anyone at any
time. Explanations were developed for all the questions to provide a rationale for correct, as well as
incorrect, answers. This information will help determine strengths and weaknesses with respect to the
PACKRAT content outline. If you have weaknesses in specific areas, you may need to obtain additional
clinical experience in those areas.

This booklet is designed to explain and interpret the information contained in the accompanying
computerized score report. You can use the report package to learn more about your abilities.
II. Explanation of the Score Report
This section provides an interpretation of each
page of the computerized score report you
received. You should have your computer
score report in front of you. Begin on page 1
of the report and read the following
information.
Page 1: Scores Your Total Score and
Group Comparisons
Page 1 is an overview of the PACKRAT
feedback report. Toward the bottom of the
page is your examination score. This score
shows the number of questions you answered
correctly out of a possible 225. The average
score for all first-year and second-year
candidates who have taken the PACKRAT to
date is also given.
Page 2: Your Strengths, Weaknesses, and
Quality of Responses
Page 2 of the score report gives an overview
of the content area in which your performance
is categorized as Strong, Satisfactory, or
Needing Improvement. These areas are
based on the examination matrix on page 1.
In each content area, your answers have
been classified as correct, acceptable,
unsatisfactory, or harmful. A definition of
these classifications is also provided on this
page. Pay particular attention to the areas
under Needing Improvement, as these areas
should be noted for further study. Also check
the answer key for the specialty area of these
items. If you selected a harmful answer in any
content area, it will be automatically placed in
the
Needing
Improvement
category,
regardless of the number of correct answers
selected. Carefully review these questions
and their explanations and specialty
classifications in Section VI to help you
understand why your answers were correct.

Copyright 2008. Physician Assistant Education Association

Page 3: Your Individual and Correct


Responses
Page 3 lists your answers to all questions.
When your answer differs from the correct
one, the proper response appears in
parentheses. Use Section VI with this page to
review the rationale for each option that is
provided in the explanations, which are
referenced to the study resources. The
explanations may help you understand why
one answer is more appropriate than another,
or not the best answer, and why some of your
answers may have been incorrect. If the
option you chose was judged potentially
harmful to the patient or others, an asterisk (*)
appears before your answer. Options
classified as potentially harmful may identify
serious weaknesses. Go over these questions
carefully and read the explanations for the
correct answers. You may be able to identify
areas where you need further study.
Page 4: Your Responses by Task and
Specialty Category
Page 4 lists your responses by both specialty
and task category. You will be able to identify
the areas of the content outline where you
may have difficulty. The numbers reflect how
many items you answered correctly out of the
total possible correct within each task and
specialty area. Categories 1 through 7 identify
the task areas and A-P the clinical specialty
areas. If you missed a significant number of
items in an area, check the key and go over
the explanations for the items in these areas.
Page 5-6: Your Profile
Demographic Profile

Comparison:

Page 5 is the beginning of the Demographic


Profile Comparison for programs. This profile
shows the reported demographic information
and compares your students information to
the entire group of individuals who have taken
the PACKRAT to date. The demographic data

are based on the information your students


provided when answering the demographics
questions. The summary demographic
information shown reflects all the data
compiled for either first or second-year
students who taken the PACKRAT.
RESPONDENT DEMOGRAPHIC
INFORMATION
C. Number of months of clinical rotations
completed at the time of this exam?
Your response:

(
(
(
(
(
(

)
)
)
)
)
)

1.
2.
3.
4.
5.
6.

All Second-year
Respondents:

None
Less than 3 months
3 to 6 months
7 to 9 months
10 to 12 months
Greater than 12 months

( 0% )
( 5% )
( 15%)
( 20%)
(45%)
(15%)

Programs can utilize this information in order


to compare the characteristics of their first
year students and second year students to the
national demographics of first year and
second year students who have taken the
PACKRAT test.
III. Recommendations for using The
Feedback Package

again to see why the answer you chose was


incorrect. If there appears to be a deficit in
your exposure to a particular clinical specialty,
perhaps further study would make you more
familiar with these situations.
Once you have completely reviewed your
score report and this booklet, PAEA hopes
you will use this information to improve your
overall performance, either on the job or on
future certification examinations. Should you
wish to provide suggestions about PACKRAT
to PAEA, you will find a comment form on the
last page of this booklet.
IV. Study Resources
A variety of textbooks are currently available
to assist candidates in preparing for the
certification examination. For additional
information, you may contact a faculty
member at an educational program or an
experienced colleague if you need help
determining which references to review in a
specific content area. A short list of general
textbooks is below. All examination questions
are related to material found in these
resources.
Please note that the books on this list are not
available from PAEA. This is not intended as
an all-inclusive list, and the materials listed
below are suggested study materials only.
1.

As a current physician assistant student,


PACKRAT can be a useful self-evaluation
tool. Through careful review of question
explanations, noting specific tasks and
content areas, you will be able to assess your
current strengths and weaknesses. You will
be able to identify particular areas in which to
concentrate more effort as you continue your
studies. By concentrating your effort on the
areas in which you did not do well, you may
improve your performance, and you may have
a better chance of passing the proctored
examination.
However,
PAEA
cannot
guarantee that this will occur, since the
conditions under which you attempted the
PACKRAT may have been different from
those in a standardized administration of a
proctored examination.
Use the explanations in Section VI to analyze
why you chose various options. Again, pay
particular attention to the options that were
judged potentially harmful or unsatisfactory.
Look at the question and the four options

Copyright 2008. Physician Assistant Education Association

2.

3.

4.

5.

6.

Andreoli TE, et al (eds). Cecils


Essentials of Medicine. 7th ed.
Philadelphia, PA: WB Saunders,
an Elsevier company, 2007
Ballweg R et al. Physician Assistant:
A Guide to Clinical Practice. 3rd ed.,
WB Saunders, an Elsevier
company, 2003. ISBN#0721600174
Bickley, LS. Bates Guide to
Physical Examination and History
Taking. 9th ed. Philadelphia, PA: JB
Lippincott Co., 2007
Beckmann CR, et al. Obstetrics &
Gynecology. 5th ed. Philadelphia,
PA: Lippincott Williams & Wilkins,
2006.
Behrman RE, et al. Nelsons
Textbook of Pediatrics. 18th ed.
Philadelphia, PA: WB Saunders, an
Elsevier company, 2007
Berkowitz, C. Pediatrics: A
Primary Care Approach, 2nd ed.
Philadelphia, PA: WB Saunders, an
Elsevier company 2000.

7.

8.

9.

10.

11.

12.

13.

14.

15.

16.

17.

18.

19.

20.

Fauci AS, et al ( eds). Harrisons


Principles of Internal Medicine.
16th ed. New York, NY:
McGraw-Hill, Inc., 2005.
DeCherney AH & Pernoll ML (eds.)
Current Obstetric & Gynecological
Diagnosis & Treatment, 10th ed.,
Norwalk, CT: Appleton & Lange,
2006
Ellsworth AJ, et al. (eds).
Mosbys Medical Drug Reference.
Philadelphia, PA: Mosby, 2007.
Wolf K, Johnson RA, and Surmond
D. Fitzpatricks Color Atlas and
Synopsis of Clinical Dermatology.
5th ed. New York, NY: McGraw-Hill,
Inc., 2005.
Goldman J and Bennet JC.
Cecil Textbook of Medicine. 23rd ed.
Philadelphia, PA: WB Saunders.
2008.
Goroll, AH, Mulley, AG & May, LA.
Primary Care Medicine: Office
Evaluation and Management of the
Adult Patient. 5th ed., Lippincott
Williams &Wilkins, 2005
Hay WW, et al. Current Pediatric
Diagnosis and Treatment. 18th ed.
Norwalk, CT: McGraw Hill, 2005.
Kaplan HI and Sadock BJ (eds).
Synopsis of Psychiatry. 10th ed.,
Philadelphia, PA: Williams &
Wilkins, 2007.
Katzung BG. Basic and Clinical
Pharmacology. 10th ed. Stamford,
CT: Appleton & Lange, 2006.
Mandel GL, Bennett JE, and Dolin
R. Principles and Practice of
Infectious Disease. 6th ed.,
Philadelphia, PA: Churchill
Livingston, an Elsevier company,
2005. ISBN#0443066434.
Marx, J., Hockberger, RS, and
Walls, RM. Rosens Emergency
Medicine: Concepts and Clinical
Practice. 6th ed., Philadelphia, PA:
Mosby, 2005.
McPhee SJ, et al. Pathophysiology
of Disease. 5th ed., McGraw Hill,
2005.
Mercier LR, et al. Practical
Orthopedics. 6th ed. Philadelphia,
PA: Mosby, an Elsevier company,
2008. ISBN#9780323036184
Mettler FA, et al. Primary Care
Radiology. Philadelphia, PA: WB

Copyright 2008. Physician Assistant Education Association

21.

22.

23.

24.

25.

26.

27.

28.

29.

30.

31.

32.

Saunders, Co., 2000. (In PAEA


library 2/07)
Howland RD and Mycek MJ.
Lippincotts Illustrated Reviews:
Pharmacology. 3rd ed. Baltimore,
MD: Williams & Wilkins, 2005.
Noble J, et al. Textbook of Primary
Care Medicine. 3rd ed. Philadephia
PA: Mosby, 2001. (In PAEA library
2/07)
Riordan-Eva, P. and Whitcher, JP.
Vaughn & Asburys General
Ophthalmology. 17th ed., McGraw
Hill, 2007.
Sacher RA and McPherson RA.
Widmann's Clinical Interpretation of
Laboratory Tests. 11th ed. FA
Davis Co., 2000. (In PAEA library
2/07)
Schwartz SI, et al. Principles of
Surgery. 8th ed. New York, NY:
McGraw-Hill, Inc., 2004.
Skinner HB (ed) Current Diagnosis
& Treatment in Orthopedics. 4th ed.,
Norwalk,CT:Appleton & Lange,
2008.
Steinberg GG. Orthopedics in
Primary Care. 3rd ed. Philadelphia,
PA: Lippincott Williams & Wilkins,
3rd ed, 1999.
Tierney LM, et al. Current Medical
Diagnosis and Treatment. 47th ed.
Stamford, CT: Appleton & Lange,
2008.
Tintinalli JE, Kelen GD, and
Stapezynski JS. Emergency
Medicine: A Comprehensive Study
Guide. 6th ed. New York, NY:
McGraw-Hill, Inc., 2004.
Townsend CM. Sabistons Textbook
of Surgery. The Biological Basis of
Modern Surgical Practice. 18 ed.
Philadelphia, PA: WB Saunders, an
Elsevier company, 2008.
Doherty GM. Current Surgical
Diagnosis and Treatment. 12th ed.,
McGraw Hill, 2005.
Wilson WR. Current Diagnosis and
Treatment in Infectious Disease.
Norwalk, CT: Appleton & Lange,
2001.

THIS PAGE INTENTIONALLY


LEFT BLANK FOR ANSWER
KEY

Copyright 2008. Physician Assistant Education Association

PHYSICIAN ASSISTANT EDUCATION ASSOCIATION


Physician Assistant Clinical Knowledge Rating and Assessment Tool
(PACKRAT) Form 13

EXPLANATIONS

1. Scientific Concepts/Cardiology
Which of the following factors in patients with chronic venous insufficiency predisposes them to development of
skin ulcers?
A. Increased intravascular oncotic pressure
B. Leakage of fibrinogen and growth factors into the interstitial space
C. Decreased capillary leakage
D. Inherited deficiency of protein C
Explanations
(u) A. Decreased intravascular oncotic pressure can cause swelling
(c) B. Leakage of fibrinogen and growth factors into the interstitial space, leukocyte aggregation and activation, and
obliteration of the cutaneous lymphatic network can predispose a patient to skin ulcers
(u) C. Increased capillary leakage causes venous insufficiency.
(u) D. Inherited deficiency of protein C predisposes patients to thrombosis.
Ref: (28)

2. History & Physical/Obstetrics/Gynecology


A 26 year-old monogamous female presents with cyclic pelvic pain that has been increasing over the last 6
months. She complains of significant dysmenorrhea and dyspareunia. She uses condoms for birth control. On
physical examination her uterus is retroverted and non-mobile, and she has a palpable adnexal mass on the left
side. Her serum pregnancy test is negative. Which of the following is the most likely diagnosis?
A. Ovarian cancer
B. Endometriosis
C. Functional ovarian cyst
D. Pelvic inflammatory disease
.
Explanations
(u) A. It is important to consider ovarian cancer in a patient with a pelvic mass however, ovarian cancer usually
occurs in older women over age 55 and patients are often asymptomatic until the disease is more advanced
(c) B. With endometriosis, the uterus is often fixed and retroflexed in the pelvis. The palpable mass is an
endometrioma or "chocolate cyst". The patient with endometriosis also often has dysmenorrhea, dyspareunia, and
dyschezia.
(u) C. Functional ovarian cysts occur from ovulation and usually are not symptomatic.
(u) D. With PID the patient will have abdominal tenderness, adnexal tenderness, cervical motion tenderness and an
elevated temperature.
Ref: (4)

3. Health Maintenance/ENT/Ophthalmology
At what age does the first tooth usually erupt in an infant?
A. 2-4 months
B. 6-8 months
C. 10-12 months
D. 14-16 months
Explanations
(u) A. See B for explanation.
(c) B. The first tooth in an infant to erupt is the central incisor at the average age of 6-8 months.
(u) C. See B for explanation.
(u) D. See B for explanation.
Ref: (3)

4. Clinical Therapeutics/Urology/Renal
A 7 year-old boy wets the bed on most nights. Which of the following is the preferred pharmacological agent to
decrease the incidence of bed wetting episodes?
A. Imipramine (Tofranil)
B. Phenytoin (Dilantin)
C. Pramipexole (Mirapex)
D. Hyoscyamine (Urised)
Explanations
(c) A. Imipramine is an anti-cholinergic and when given before bedtime has been shown to decrease the incidence of
bed wetting.
(u) B. Phenytoin is an anticonvulsant and is not used in enuresis.
(u) C. Pramipexole is a dopamine agonist used in the treatment of restless leg syndrome.
(u) D. Hyoscyamine is an anti-spasmodic used to treat overactive bladder.
Ref: (5)

5. Diagnosis/Cardiology
A newborn is being evaluated for perioral cyanosis while feeding associated with sweating. Vital signs are rectal
temperature, 37.8 degrees C (100 degrees F), blood pressure 80/45 mmHg, pulse 180/min, and respirations 40/min.
A grade 3/6 harsh systolic ejection murmur with a single loud S2 is heard at the left upper sternal border.
Electrocardiogram (ECG) shows right ventricular hypertrophy with right axis deviation. Chest x-ray shows a bootshaped heart and decreased pulmonary vascular markings. Which of the following is the most likely diagnosis?
A. Atrial septal defect
B. Total anomalous pulmonary venous return
C. Coarctation of the aorta
D. Tetralogy of Fallot
Explanations
(u) A. Although the murmur may be consistent with an ASD with pulmonary hypertension the chest x-ray would not
show decreased pulmonary vascular markings. With a large left to right shunt large pulmonary arteries and increased
vascularity would be seen.
(u) B. The murmur for TAPVR is a soft systolic murmur at the left upper sternal border with a split S2 in addition to a
short mid-diastolic murmur at the low left sternal border.
(u) C. Cyanosis is usually not the presenting sign for coarctation of the aorta. Infants may present with heart failure,
ECG will show evidence of LVH.
(c) D. This is a common presentation for tetralogy of fallot.
Ref: (28)

6. Diagnostic Studies/Endocrinology
Which of the following is considered to be the modality of choice for the identification of a pituitary macroadenoma
that is suspected on the basis of a visual field deficit?
A. Skull x-ray
B. PET scan
C. CT of the brain
D. MRI of the brain
Explanations
(u) A. Skull x-ray is not the modality of choice for the identification of pituitary macroadenoma.
(u) B. PET scan is not the modality of choice for the identification of pituitary macroadenoma.
(u) C. CT of the brain is not the optimal imaging technique for evaluation of the pituitary stalk.
(c) D. MRI of the brain provides the best visualization of pituitary tumors.
Ref: (28)

7. Clinical Intervention/Infectious Diseases


A 2 year-old male presents with a four day history of fever and general malaise. On examination the vitals reveal an
oral temperature of 102 degrees F. The child appears to have rubor on the trunk which started one day prior to this
visit. Physical examination reveals a maculopapular rash with defervescence. Which of the following is the most
appropriate management at this time?
A. Ibuprofen (Motrin)
B. Aspirin
C. Amoxicillin
D. Valacyclovir (Valtrex)
Explanations
(c) A. Motrin is indicated for management of the fever in Roseola infantum caused by the herpesvirus.
(h) B. Aspirin is contraindicated in children for management of fever.
(u) C. Amoxicillin is not indicated for viral infections.
(u) D. Valacyclovir is indicated for Varicella-Zoster virus infection for decreasing the incidence of varicella pneumonia
only. In healthy children valacyclovir is not indicated due to its marginal therapeutic benefit.
Ref: (13)

8. Clinical Therapeutics/Orthopedics/Rheumatology
A 42 year-old female experiences pain on the plantar surface of her left foot in the area of the third metatarsal head.
The pain is associated with wearing tight shoes and is relieved by removing shoes. Examination reveals a palpable
mass and reproduction of pain with deep palpation of the third intermetatarsal space. The patient has tried wearing
wider shoes with metatarsal cushions and taking NSAIDS but her symptoms persist. What is the best therapeutic
option at this point?
A. Casting of the involved foot
B. Physical therapy
C. Steroid injection
D. Surgical excision
Explanations
(u) A. Casting the foot in a patient with Morton's neuroma is not effective therapy.
(u) B. Physical therapy has not been shown to be of benefit in treating Morton's neuroma.
(c) C. Steroid injection is the treatment of choice for Morton's neuroma when conservative measures fail.
(u) D. Surgical excision is recommended for treatment of Morton's neuroma only if conservative measures and steroid
injection have failed.
Ref: (26)

10

9. History & Physical/Pulmonology


Which of the following is the most likely to develop into a persistent cough in the adult patient?
A. Pertussis
B. Allergic rhinitis
C. Pharyngitis
D. Heart failure
Explanations
(c) A. Pertussis is suspected in patients with persistent cough that lasts longer than 2-3 weeks. Allergic rhinitis,
pharyngitis and heart failure are all potential causes of acute cough.
(u) B. See A for explanation.
(u) C. See A for explanation.
(u) D. See A for explanation.
Ref: (28)

10. Diagnosis/Gastrointestinal/Nutritional
A 30 year-old patient presents with weight loss, diarrhea, and steatorrhea. Labs reveal that the antiendomysial
antibody (AEA) is positive. What is the most likely diagnosis?
A. Celiac sprue
B. Ulcerative colitis
C. Whipple's disease
D. Zollinger-Ellison syndrome
Explanations
(c) A. Celiac sprue is not only characterized by these classic symptoms. The antiendomysial antibody has a 90-95%
sensitivity and 90-95% specificity for celiac sprue.
(u) B. Ulcerative colitis could be responsible for the symptoms mentioned however antiendomysial antibody (AEA)
would not be positive.
(u) C. Whipple's disease is an infectious disorder known to cause diarrhea and weight loss. It also results in arthralgia
and symptoms involving both the central nervous and cardiac systems that are not easily missed due to their severity.
Diagnosis involves biopsies of the involved tissues looking for PAS-positive macrophages.
(u) D. Zollinger-Ellison syndrome (ZE) is a hypersecretory disorder. Diarrhea can occur but will not generally be the
predominant symptom.
Ref: (7)

11. Clinical Intervention/Dermatology


A patient sustained a 6 cm laceration on his anterior tibia that was primarily closed in the emergency department.
What is the most appropriate time frame for removal of these sutures?
A. 1-2 days
B. 3-5 days
C. 6-8 days
D. 7-14 days
Explanations
(u) A. See D for explanation.
(u) B. See D for explanation.
(u) C. See D for explanation.
(c) D. Suture removal is based upon the area of the body that was sutured. Facial sutures are placed for 3-4 days,
scalp sutures for 5-7 days, trunk sutures are placed for 6-8 days, and sutures on the extremity are placed for 7-14
days. Sutures on the extremities can stay for longer periods of time if the area is under maximal tension.
Ref: (2)

11

12. Clinical Therapeutics/Cardiology


A hospitalized patient is found with confirmed pulseless ventricular tachycardia. IV access is obtained following the
second shock given. Which of the following medications is to be administered immediately?
A. Amiodarone
B. Magnesium
C. Atropine
D. Epinephrine
Explanations
(u) A. Antiarrhythmics are given after the third shock and epinephrine has been administered.
(u) B. Magnesium is useful for torsades de pointes.
(u) C. Atropine may be used for asystole or a slow pulseless electrical activity (PEA) rate.
(c) D. Epinephrine should be given as soon as IV access is obtained before or after the second shock.
Ref: (11)

13. Diagnostic Studies/Psychiatry/Behavioral Medicine


A patient with advanced AIDS complicated by toxoplasmosis presents with altered mental status, recent onset of
seizures, and focal neurologic deficits. Which of the following diagnostic studies is most helpful?
A. Toxoplasma gondii antibody titers
B. CT scan of the brain
C. Lumbar puncture
D. MRI of brain
Explanations
(u) A. Antibody titers cannot be depended upon since most patients have IgG titers that reflect past infection,
significant rises are infrequent, and IgM antibody is rare.
(u) B. A CT scan may be helpful, but an MRI is the more sensitive test.
(h) C. Lumbar puncture is contraindicated secondary to possible mass effect.
(c) D. An MRI showing multiple isodense or hypodense ring-enhancing mass lesions is the most useful test for such a
patient.
Ref: (7)

14. Health Maintenance/Pulmonology


A pediatric patient presents with a history of multiple recurrent respiratory infections associated with failure to thrive.
A sweat chloride test is elevated. Which of the following is a common cause of death in patients with this condition?
A. Diabetic ketoacidosis
B. Pulmonary infection
C. Intestinal obstruction
D. Acute respiratory failure
Explanations
(u) A. While patients with cystic fibrosis most likely will eventually develop insulin-dependent diabetes mellitus,
diabetic ketoacidosis is not a common cause of death.
(c) B. This patient has cystic fibrosis. The most common causes of death include pulmonary complications, such as
infections, and terminal chronic respiratory failure associated with cor pulmonale.
(u) C. While intestinal obstruction may occur in patients with cystic fibrosis, it is not a common cause of death.
(u) D. See B for explanation.
Ref: (28)

15. History & Physical/Neurology


In addition to tremor, which of the following are cardinal symptoms of Parkinson disease?
A. Cognitive decline and rigidity
B. Personality change and bradycardia
C. Eye movement abnormalities and hyperkinesias
D. Rigidity and bradykinesia

12

Explanations
(u) A. See D for explanation.
(u) B. See D for explanation.
(u) C. See D for explanation.
(c) D. Tremor, rigidity, bradykinesia and postural instability are the cardinal features of Parkinsonism and may be
present in any combination.
Ref: (28)

16. Scientific Concepts/Hematology


Normal hemoglobin A is made of what combination of heme and globin chains?
A. 2-alphas and 2-betas
B. 4-gammas
C. 2-alphas and 2-gammas
D. 4-betas
Explanations
(c) A. Hemoglobin A1 is composed of two alpha and two beta chains.
(u) B. Barts Hemoglobin-Hydrops Fetalis has 4-gamma chains and is incompatible with life.
(u) C. Fetal hemoglobin consists of 2-alpha and 2-gamma chains.
(u) D. Hemoglobin H consists of 4-beta chains and is nonfunctional.
Ref: (28)

17. Clinical Intervention/Gastrointestinal/Nutritional


Which of the following is the most important intervention in acute pancreatitis?
A. IV fluid administration
B. Antibiotic administration
C. Calcium replacement
D. Antiemetics
Explanations
(c) A. The mainstay of management in acute pancreatitis is fluid resuscitation. Isotonic solutions are best to maintain
renal perfusion and urine output > 100 ml/hour.
(u) B. 90% of patients will recover with supportive measures only. Parenternal antibiotics are not indicated in acute
pancreatitis.
(u) C. Hypocalcemia is a marker of prognosis, but correction of hypocalcemia does not supercede the importance of
fluid resuscitation from the overwhelming inflammatory response.
(u) D. Other supportive measures in acute pancreatitis include pain management and antiemetics, but the mainstay is
fluid resuscitation.
Ref: (29)

18. Diagnosis/Cardiology
An electrocardiogram (ECG) shows a sinus rhythm with varying T wave heights, axis changes every other beat and a
wandering baseline. Which of the following is most likely the diagnosis?
A. Artifact
B. Digoxin toxicity
C. Pericardial effusion
D. Poor lead placement
Explanations
(u) A. Artifact could show a wandering baseline, but not the distinct axis changes.
(u) B. Digoxin toxicity can cause bidirectional tachycardia, but not electrical alternans.
(c) C. This ECG pattern best represents pericardial effusion due to a swinging heart in fluid and is known as electrical
alternans.
(u) D. Poor lead placement would show different patterns per the leads.
Ref: (28)

13

19. Clinical Therapeutics/Endocrinology


Which of the following is the first-line treatment for a patient with mild syndrome of inappropriate secretion of ADH
(SIADH)?
A. Sodium supplementation to correct the hyponatremia
B. Restriction of free water
C. High volume hypertonic saline infusion
D. Pituitary ablation via transsphenoidal approach
Explanations
(u) A. Patients with SIADH will have euvolemic hyponatremia but this is a water problem and not a sodium problem.
Supplementation of additional sodium only helps with water retention in the body and can produce edema but it is not
valuable in the correction of this hyponatremia.
(c) B. Restriction of free water intake is the first-line therapy for patients with euvolemic hyponatremia that is caused
by SIADH. Water intake should be restricted to 0.5 to 1 liter per day. A gradual increase in serum sodium will occur
over days with this treatment.
(h) C. High volume hypertonic saline infusion can result from too rapid correction of hyponatremia by causing central
pontine myelinolysis.
(h) D. Patients with SIADH are treated medically in order to increase the serum sodium rather than ablation of the
pituitary gland which is not indicated for this condition.
Ref: (28)

20. Diagnostic Studies/Pulmonology


A 62 year-old homeless patient presents complaining of fever, weight loss, anorexia, night sweats and a chronic
cough that recently became productive of purulent sputum that is blood streaked. On physical examination, the
patient appears chronically ill and malnourished. Which of the following chest x-ray findings supports your suspected
diagnosis?
A. Hyperinflation and flat diaphragms
B. Interstitial fibrosis and pleural thickening
C. Cavitary lesions involving the upper lobes
D. "Eggshell" calcification of hilar lymph nodes
Explanations
(u) A. Chest x-ray findings of hyperinflation and flat diaphragms suggest long-standing chronic obstructive lung
disease.
(u) B. Interstitial fibrosis and pleural thickening on a chest x-ray are found in cases of interstitial lung disease.
(c) C. This patient most likely has tuberculosis. A chest x-ray finding of cavitary lesions involving the upper lobes
would support this suspected diagnosis.
(u) D. Chest x-ray findings of "eggshell" calcification of hilar lymph nodes strongly supports a diagnosis of silicosis.
Ref: (1)

21. History & Physical/Orthopedics/Rheumatology


Abduction of the shoulder against resistance helps localize pain in which of the following muscles of the shoulder
girdle?
A. Supraspinatus
B. Infraspinatus
C. Teres minor
D. Subscapularis
Explanations
(c) A. Abduction against resistance tests the supraspinatus.
(u) B. Lateral rotation against resistance tests the infraspinatus and teres minor.
(u) C. See B for explanation.
(u) D. Medial rotation against resistance tests the subscapularis.
Ref: (4)

14

22. Health Maintenance/Obstetrics/Gynecology


What is the recommended method for screening pregnant women for gestational diabetes?
A. Fasting blood sugar and 2 hour post prandial
B. 50 gram glucose load followed by a blood sugar in 1 hour
C. 75 gram glucose load followed by a blood sugar in 2 hours
D. 100 gram glucose load followed by a blood sugar at 1 hour, 2 hours, and 3 hours
Explanations
(u) A. Fasting blood sugar and 2 hour postprandial blood test is used to follow patient with gestational diabetes.
(c) B. One hour Glucola is the screening test for gestational diabetes. It is a 50 gram glucose load, with a serum
glucose obtained 1 hour after the dose. Normal value is less than 140 mg/dL.
(u) C. A 75 gram glucose load is used in non-pregnant patients.
(u) D. This describes a three-hour GTT, which is ordered if the 1 hour Glucola is elevated above 140 gm/dL.
Ref: (4)

23. Diagnosis/ENT/Ophthalmology
A 45 year-old smoker presents with a sore mouth and increasing difficulty eating for two weeks. Physical examination
reveals a 1 cm white lesion on the buccal mucosa that cannot be rubbed off. Which of the following is the most likely
diagnosis?
A. Oral cancer
B. Oral candidiasis
C. Aphthous ulcer
D. Necrotizing ulcerative gingivitis
Explanations
(c) A. The presence of leukoplakia in a smoker over the age of 40 should be biopsied to rule out the presence of oral
cancer.
(u) B. Oral candidiasis presents with white patches. Unlike leukoplakia, the patches easily rub off.
(u) C. While aphthous ulcers are commonly found on the buccal mucosa, they are usually 1 to 2 mm round ulcerative
lesions.
(u) D. Necrotizing ulcerative gingivitis is common in young adults under stress. Clinically, it presents with painful
acute gingival inflammation and necrosis.
Ref: (28)

24. Clinical Therapeutics/Cardiology


A 25 year-old female presents to the emergency department after an episode of substernal chest pain with radiation
to the middle of her back that came on suddenly and lasted for about four minutes this morning while in bed. She
denies previous episodes. Examination is unremarkable, but she appears jittery. Toxicology screen is positive for
cocaine. Which of the following medications is contraindicated in this patient?
A. Lorazepam (Ativan)
B. Diltiazem (Cardizem)
C. Nitroglycerin (Nitrostat)
D. Propanolol (Inderal)
Explanations
(u) A. Lorazepam is not contraindicated and can help with agitation, psychosis or seizures.
(u) B. Diltiazem is not contraindicated but does not have a definitive role in treating cocaine toxicity.
(u) C. Nitroglycerin is not contraindicated but does not have a definitive role in treating cocaine toxicity.
(c) D. Pure Beta blockers, such as propranol, can cause a paradoxical hypertension because of unopposed alphaadrenergic effects.
Ref: (28)

15

25. Scientific Concepts/Infectious Diseases


Which immunoglobulin is the first to respond during the primary immune response for a gram positive bacterial
infection?
A. IgM
B. IgG
C. IgA
D. IgE
Explanations
(c) A. IgM is the first immunoglobin to respond during the acute exposure. This immunoglobulin promotes
opsonization and phagocytosis. IgG is the immunoglobulin that responds during the secondary exposure. IgE is the
immunoglobulin that responds during an allergic response as well as during a parasitic infection. IgA is an antibody
found in colostrums and GI secretions.
(u) B. See A for explanation.
(u) C. See A for explanation.
(u) D. See A for explanation.
Ref: (32)

26. History & Physical/Psychiatry/Behavioral Medicine


A patient with obsessive compulsive disorder would most likely have which of the following findings?
A. Raw, red hands
B. Priapism
C. Memory impairment
D. Abdominal pain
Explanations
(c) A. Common manifestations of obsessive compulsive disorder include phobias of germ and contaminants which
results in frequent handwashing leading to chafed and reddened hands. The other answers are inconsistent with
obsessive compulsive disorder.
(u) B. See A for explanation.
(u) C. See A for explanation.
(u) D. See A for explanation.
Ref: (14)

27. Health Maintenance/Gastrointestinal/Nutritional


As a rule, solid foods such as cereal and fruits are best introduced into an infant's diet at approximately
A. 1 to 3 weeks.
B. 4 to 8 weeks.
C. 4 to 6 months.
D. 10 to 14 months.
Explanations
(u) A. See C for explanation.
(u) B. See C for explanation.
(c) C. The inclusion of solid foods in the diet is best done at approximately 4-6 months. Before 4 to 6 months of age
does not contribute significantly to the infant's health and it increases their risk of allergies and atopy.
(u) D. After 6 months, meat, eggs, and starchy foods can be introduced.
Ref: (13)

16

28. Clinical Intervention/Cardiology


A 56 year-old female four days post myocardial infarction presents with a new murmur. On examination the murmur
is a grade 3/6 pansystolic murmur radiating to the axilla. She is dyspenic at rest and has rales throughout all her lung
fields. Blood pressure is 108/68 mmHg, pulse 70 bpm. Which of the following would be the definitive clinical
intervention?
A. Intra-aortic balloon counterpulsation
B. Mitral valve replacement
C. Coronary artery bypass surgery
D. Immediate fluid bolus
Explanations
(u) A. Although part of the primary treatment to reduce mitral regurgitation, it is not definitive.
(c) B. MVR is the definitive intervention to correct MR caused by papillary muscle rupture.
(u) C. CABG may be necessary if significant blockage is found, but it will not correct the mitral regurgitation.
(u) D. A fluid bolus is indicated if the patient is hypotensive.
Ref: (28)

29. Diagnosis/Neurology
During an influenza epidemic, a 6 year-old male is seen with fever and a severe sore throat. The parents report that
his symptoms have not improved despite administration of aspirin. The next day, the parent calls to report that the
child has persistent vomiting and increased lethargy. On examination, he is found to be delirious and disoriented with
hyperactive reflexes. The liver edge is 3 cm below the right costal margin in the midclavicular line. Which of the
following is the most likely diagnosis?
A. Reye's syndrome
B. Measles encephalitis
C. Guillain-Barre syndrome
D. Acute bacterial meningitis
Explanations
(c) A. The suspected influenza associated with development of vomiting, progressive mental status changes,
hyperreflexia, and hepatomegaly are consistent with a diagnosis of Reye's syndrome.
(u) B. Measles encephalitis is a complication of rubeola, which is not suggested by this patient's presentation.
(u) C. Fever is uncommon with Guillain-Barre, which is also characterized by ascending symmetrical weakness of the
lower extremities.
(u) D. While acute bacterial meningitis might present with acutely evolving symptoms, hepatomegaly is not usually
seen.
Ref: (13)

30. Clinical Therapeutics/Obstetrics/Gynecology


What is the treatment of magnesium sulfate toxicity?
A. Nifedipine
B. Terbutaline
C. Potassium carbonate
D. Calcium gluconate
Explanations
(u) A. Nifedipine, a calcium-channel blocker is used to treat both preterm labor and hypertension in pregnancy. It
works by inhibiting calcium transport through slow-type channels, causing reduction in systemic and pulmonary
vascular resistance and tocolysis.
(u) B. Terbutaline is a beta-blocker that is used to treat pre-term labor.
(u) C. Potassium carbonate is a treatment for metabolic acidosis, not magnesium sulfate toxicity.
(c) D. 10% calcium gluconate is used to treat magnesium sulfate toxicity.
Ref: (4)

17

31. Diagnostic Studies/Orthopedics/Rheumatology


A 26 year-old male who is an avid swimmer has been experiencing right shoulder pain for the past month. On
examination, pain is elicited with palpation below the anterior acromion. Anterior shoulder pain is also reported when
the patient flexes and extends his arm. Which of the following diagnostic tests is most appropriate at this time?
A. Shoulder x-ray
B. Shoulder arthroscopy
C. Shoulder MRI
D. Subacromial lidocaine injection
Explanations
(u) A. This patient is experiencing mild impingement syndrome and, although plain films may demonstrate bone spurs
on the undersurface of the acromion, his age and relatively recent onset of symptoms suggest that this study would
likely be normal.
(u) B. Arthroscopy is not indicated unless less invasive modalities fail to aid in the diagnosis and symptoms of
impingement persist despite therapy.
(u) C. While MRI would likely support the diagnosis, it is an expensive modality and not necessary to diagnose mild
impingement.
(c) D. Subacromial injection of lidocaine leading to a transient but dramatic improvement in pain with shoulder
extension makes the diagnosis of impingement highly likely.
Ref: (27)

32. Scientific Concepts/Pulmonology


Which of the following is the major pathogenetic mechanism that causes asthma?
A. Airway inflammation
B. Increased pulmonary secretions
C. Presence of Ghon complexes
D. Irreversible fibrosis
Explanations
(c) A. Airway inflammation is the major pathogenetic mechanism that leads to the development of asthma.
(u) B. Increased pulmonary secretions are the mechanism in chronic bronchitis.
(u) C. The presence of Ghon complexes is noted in pulmonary tuberculosis.
(u) D. Irreversible fibrosis of the lung parenchyma is associated with interstitial lung diseases.
Ref: (1)

33. History & Physical/ENT/Ophthalmology


When performing a Weber test on a patient with impacted cerumen in the right canal, the sound should be
A. referred to the right ear.
B. referred to the left ear.
C. equal in both ears.
D. louder with air conduction.
Explanations
(c) A. In unilateral conductive hearing loss, the sound is referred to the impaired ear.
(u) B. See A for explanation.
(u) C. See A for explanation.
(u) D. Bone conduction as noted with the Rinne test is louder than air with conductive hearing loss.
Ref: (3)

18

34. Health Maintenance/Urology/Renal


Which of the following is used to monitor possible recurrence of prostate cancer?
A. Prostate specific antigen
B. Acid phosphatase
C. Transrectal ultrasound
D. Bone scan
Explanations
(c) A. Increasing levels of prostate specific antigen are consistent with progression of disease.
(u) B. Acid phosphatase levels may increase in prostate cancer, but are not as sensitive as prostate specific antigen
for recurrence of disease.
(u) C. Transrectal ultrasound is mainly used for staging of disease and not monitoring of recurrence.
(u) D. Radionuclide bone scan is used to detect bony metastases.
Ref: (28)
35. Diagnosis/Dermatology
A patient presents with profound itching. Examination reveals short, reddish lesions on the wrists, elbows, and finger
webs. Papules are also noted in these areas. There appears to be burrow marks emanating proximal to the finger
webs. Which of the following is the most likely diagnosis?
A. Scabies
B. Body lice
C. Rocky Mountain Spotted Fever
D. Lyme Disease
Explanations
(c) A. Scabies is an infestation of a mite that is usually spread by skin to skin contact. Patients present with
intractable pruritus, often with minimal cutaneous findings. There may be an associated inflammatory papule or
nodule along with a burrow that is sometimes seen early in the course of the infestation.
(u) B. Body lice or pediculosis pubis is an infestation of hair-bearing regions, most commonly in the pubic area but
may also be seen on hairy parts of the chest, axillae, and upper eyelashes. It is characterized by mild to moderate
pruritus, papular urticaria, and excoriation.
(u) C. Patients develop Rocky Mountain Spotted Fever after a tick bite. It is classically associated with a sudden
onset of fever, severe headache, myalgia, and a characteristic acral exanthem.
(u) D. Lyme disease is associated with an oval shaped skin lesion with central clearing known as erythema
chronicum migrans. It occurs following a tick bite and may be associated with Bell's palsy, arthralgias, and heart
block.
Ref: (10)

36. Clinical Intervention/Endocrinology


Which of the following is the most accurate method to differentiate benign from malignant thyroid nodules?
A. Fine needle aspiration biopsy
B. Ultrasound of the thyroid gland
C. Measurement of thyroid antibodies
D. Radionuclide iodine uptake
Explanations
(c) A. Fine needle aspiration biopsy is the best method used in the evaluation of thyroid cancer. Fine needle
aspiration biopsy can be done on an outpatient basis and can be done under ultrasound guidance for patients who
have small thyroid nodules.
(u) B. Ultrasound of the thyroid gland is performed in order to help to guide the practitioner in performing a fine needle
aspiration biopsy. It can also be used to differentiate solid from cystic masses but it cannot predict which nodules are
cancer and which are benign.
(u) C. Measurement of thyroid antibodies is most helpful in patients who have subclinical hypothyroidism or overt
hyperthyroidism. There is a lot of controversy regarding whether treatment should be given to patients with subclinical
disease and most providers will use thyroid antibodies to determine whether treatment should be given or delayed.
(u) D. Radionuclide iodine uptake is performed to assess the activity of the thyroid gland. This test is most helpful to
determine whether the gland is metabolically active (as seen with Graves' disease) or whether the gland is just
releasing preformed thyroid hormone (subacute thyroiditis). The test provides a percentage of activity rather than a
picture (thyroid scan) provides a picture of whether a nodule is "hot" or cold."
Ref: (28)

19

37. Clinical Therapeutics/Cardiology


A 16 year-old male with a history of tetralogy of Fallot presents to clinic for a follow-up visit status post replacement of
his right ventricle to pulmonary artery conduit. He has complaints of chest pain with inspiration, fever and general
malaise. Cardiac examination reveals a rub with muffled heart sounds. Labs show an elevated erythrocyte
sedimentation rate (ESR) and leukocytosis. Which of the following is the most effective treatment?
A. Acetaminophen/oxycodone
B. Azithromycin
C. Indomethacin
D. Furosemide
Explanations
(u) A. See C for explanation.
(u) B. See C for explanation.
(c) C. Indomethacin is suitable for controlling pain in Dressler's syndrome. ASA is preferred. Narcotics, diuretics or
antibiotics are not recommended.
(u) D. See C for explanation.
Ref: (11)

38. Diagnostic Studies/Gastrointestinal/Nutritional


Which of the following laboratory tests, if positive, would be most indicative of Crohn's disease?
A. Antineutrophil cytoplasmic antibodies (ANCA)
B. Antiendomysial antibodies (AEA)
C. Antinuclear antibodies (ANA)
D. Anti-Saccharomyces cerevisiae antibodies (ASCA)
Explanations
(u) A. Antineutrophil cytoplasmic antibodies is positive in 60-70% of patients with ulcerative colitis while only 5-10% of
Crohn's disease patients have positive results.
(u) B. The antiendomysial IgA antibodies are directed against tissue transglutaminase and has a 90-95% specificity
for celiac sprue.
(u) C. Antinuclear antibodies are elevated in a wide variety of autoimmune disorders and therefore are a nonspecific
finding.
(c) D. About 60-70% of patients with Crohn's disease are positive for these antibodies which are directed at the cell
walls of S. cerevisiae while only 10-15% of ulcerative colitis is found to be positive.
Ref: (7)

39. Scientific Concepts/Orthopedics/Rheumatology


A 22 year-old female complains of worsening pain, swelling, and tenderness in her left heel for 1 week. She
sustained a penetrating injury to the heel two weeks ago when she stepped on a nail while running in tennis shoes.
Examination reveals a draining puncture wound with surrounding erythema and exquisite tenderness. X-ray of the left
foot demonstrates periosteal reaction associated with the wound. Which organism is classically responsible for this
infection?
A. Escherichia coli
B. Streptococcus pyogenes
C. Pseudomonas aeruginosa
D. Staphylococcus aureus
Explanations
(u) A. While gram negative enteric organisms can be a cause of osteomyelitis, E. coli is rarely seen outside of the
neonatal period.
(u) B. Streptococcus pyogenes is an uncommon (~10 %) cause of acute osteomyelitis. The mechanism of injury
suggests a different pathogen.
(c) C. Pseudomonas aeruginosa is frequently associated with osteomyelitis involving puncture wounds of the foot.
This is believed to result from direct inoculation with P. aeruginosa via the foam padding found in tennis shoes.
(u) D. Staphylococcus aureus is the most common infecting organism in cases of acute osteomyelitis, however, the
mechanism of injury in this case suggests P. aeruginosa.
Ref: (5)

20

40. History & Physical/Hematology


What examination finding would be expected in a patient with von Willebrand disease?
A. Gingival bleeding
B. Splenomegaly
C. Muscle weakness
D. Hemarthrosis
Explanations
(c) A. A patient with von Willebrand disease most commonly presents with mucosal bleeding seen in epistaxis,
gingival bleeding, and menorrhagia.
(u) B. Splenomegaly is seen with hemolytic anemia, not with von Willebrand disease.
(u) C. Muscle weakness is not seen with von Willebrand disease.
(u) D. Hemarthrosis is found in hemophilia, not in von Willebrand disease.
Ref: (28)

41. Diagnosis/Psychiatry/Behavioral Medicine


A 63 year-old retired engineer presents with one month of difficulty sleeping. He has a hard time staying asleep and
says he is just restless. He also states that he has been more forgetful and can't pay attention very well. He lives with
his wife. He denies fever, chills, recent trauma, or difficulty walking. When asked about the specifics of his symptoms,
he repeatedly replies, "I don't know," without really trying. He states his wife thinks he just sits around the house all
day since he retired. His neurological exam is unremarkable, except for some mild psychomotor retardation. Which of
the following is the most likely diagnosis?
A. Dementia
B. Delirium
C. Depression
D. Dissociative disorder
Explanations
(u) A. Dementia is a deterioration of selective mental functions including progressive loss of impulse control and an
attempt to cover up mental deficiencies.
(u) B. Delirium has a rapid onset of symptoms, mental status fluctuations. Anxiety and irritability are common.
(c) C. Depression often presents with difficulty thinking and concentrating, lessened sleep, and withdrawal from
activities.
(u) D. Dissociative disorder is associated with inability to recall important personal information usually of traumatic
nature that is too extensive to be explained by ordinary forgetfulness.
Ref: (28)

42. Health Maintenance/Orthopedics/Rheumatology


Which of the following is an established risk factor for osteoporosis?
A. Parity status
B. Carbohydrate intake
C. Lactation history
D. Low body weight
Explanations
(u) A. Parity status and lactation history have been shown to be poor predictors of bone mass; therefore, they do not
have an established role in the prediction of increased risk for development of osteoporosis.
(u) B. Carbohydrate intake has no association to the development of osteoporosis.
(u) C. See A for explanation.
(c) D. Established risk factors for osteoporosis include low body weight, female sex, advanced age, Caucasian race,
and bilateral oophorectomy before menopause without estrogen replacement.
Ref: (19)

21

43. Clinical Intervention/Obstetrics/Gynecology


A 52 year-old obese patient with persistent heavy menses undergoes an endometrial biopsy and is diagnosed with
atypical adenomatous hyperplasia. What is the next step in the management of this patient?
A. Total abdominal hysterectomy
B. Observation and endometrial biopsy in 3 months
C. Endometrial curettage followed by progesterone daily
D. Oral progesterone days 16-25 of the month for 6 months and repeat biopsy

Explanations
(c) A. Atypical adenomatous hyperplasia contains cellular atypia and mitotic figures in addition to glandular crowding
and complexity. This has a 20-30% risk of progression to endometrial cancer and the recommendation is
hysterectomy.
(h) B. Observation and biopsy again in 3 months would increase the risk of endometrial cancer for this patient.
(h) C. Endometrial curettage would remove the hyperplasia and progesterone will decrease the endometrial glandular
proliferation. This would be appropriate management in a patient with endometrial hyperplasia without atypia.
(h) D. Oral progesterone for 10 days of the month will cause the patient to have a withdrawal bleed every month. This
would be an appropriate treatment in a premenopausal patient with endometrial hyperplasia without atypia
Ref: (4)
44. Diagnostic Studies/Cardiology
A 72 year-old male presents to the emergency department with crushing chest pain, dyspnea and palpitations for 2
hours in duration. Enzymes are pending and he has been given aspirin and sublingual nitroglycerin. He is rushed to
the catheterization lab where they find a totally occluded distal right coronary artery. Which of the following
electrocardiogram (ECG) findings supports the diagnosis?
A. Q waves in leads I, aVL, V5-V6
B. ST segment elevation in leads II, III, aVF
C. Hyperacute T waves in leads I, aVL
D. Flipped T waves with repolarization changes in leads V1-V4
Explanations
(u) A. Q waves in leads I, aVL, V5-V6 represent infarction involving the circumflex artery.
(c) B. ST segment elevation in leads II, III, aVF, represents an acute process in the right coronary artery.
(u) C. Hyperacute T waves in leads I, aVL can represent the initial changes of an infarction involving the circumflex
artery.
(u) D. Flipped T waves with repolarization changes in leads V1-V4 can represent early stages of infarction involving
the left anterior descending artery.
Ref: (11)

45. Clinical Therapeutics/Pulmonology


A patient presents with occasional wheezing and chest tightness that occurs approximately once a week and at night
only about once a month. Peak expiratory flow is 85% of predicted. Which of the following is the most appropriate
initial treatment?
A. Albuterol (Proventil) inhaler
B. Montelukast (Singular)
C. Salmeterol (Serevent) inhaler
D. Sustained release theophylline
Explanations
(c) A. This patient has mild intermittent asthma which is initially treated with inhaled beta 2-agonists as needed. No
long-term control medications are indicated.
(u) B. Leukotriene modifiers, such as montelukast, may be added to the treatment of uncontrolled asthma as a longterm controller after the initiation of inhaled corticosteroids.
(u) C. Long-acting beta 2-agonists, such as salmeterol, are indicated for long-term control of asthma that is
categorized as moderate persistent to severe persistent.
(u) D. Sustained release theophylline is an alternative treatment for asthma that is at least categorized as mild
persistent; however its narrow therapeutic window and side effects limit its use.
Ref: (28)

22

46. History & Physical/Urology/Renal


Which of the following signs and symptoms is typically noted in patients with acute cystitis?
A. Fever and chills
B. CVA tenderness
C. Flank pain
D. Frequency and dysuria
Explanations
(u) A. Signs of systemic toxicity, such as fever and chills, are absent in acute cystitis.
(u) B. CVA tenderness and flank pain are associated with acute pyelonephritis.
(u) C. See B for explanation.
(c) D. Irritative voiding symptoms, such as frequency and dysuria, are common in acute cystitis.
Ref: (28)

47. Diagnosis/Endocrinology
A 54 year-old female presents to the office for radiographic and laboratory results. The radioactive iodine uptake is
elevated while the thyroid hormone levels are increased with TSH levels being suppressed. Which of the following is
the most likely diagnosis?
A. Graves' disease
B. Hashimoto's thyroiditis
C. Subacute thyroiditis
D. Pituitary failure
Explanations
(c) A. Graves' disease is associated with an elevated uptake on the radioactive thyroid scan due to an increase in the
activity for the thyroid gland. Because the gland is actually making thyroid hormone, the free T4 level will be
increased and the TSH will be suppressed as a result of negative feedback to the hypothalamus and pituitary gland.
(u) B. Patients with Hashimoto's thyroiditis will have an underactive thyroid gland with a decrease in the radioactive
iodine uptake. The thyroid gland is not producing enough thyroid hormones so the free T4 level will be low and the
TSH level will be increased as the body tries to stimulate the thyroid gland to produce more thyroid hormone.
(u) C. Patients with subacute thyroiditis have an increase in the release of thyroid hormone rather than an increase in
the actual activity of the gland. These patients will have a normal or decreased iodine uptake on the radioactive
thyroid scan which is the main differentiating feature between this condition and Graves' disease. The free T4 level
can be variable and depends upon when in the course of this condition the thyroid hormones are measured.
(u) D. Patients with pituitary failure will not be able to produce thyroid stimulating hormone so these patients will have
a decreased radioactive thyroid uptake, a low free T4 level and a low TSH.
Ref: (28)

48. Scientific Concepts/Neurology


The source of pain experienced during a migraine headache is a result of activation of which nerve?
A. Trigeminal
B. Vagus
C. Optic
D. Occulomotor
Explanations
(c) A. Headache may result in release of neuropeptides acting as neurotransmitters at trigeminal nerve branches.
(u) B. See A for explanation.
(u) C. See A for explanation.
(u) D. See A for explanation.
Ref: (28)

23

49. Clinical Intervention/ENT/Ophthalmology


A 17 year-old male is accidentally struck in the right eye while playing football and is immediately transported to the
hospital. In the emergency room, he complains of severe pain behind the eye as well as double vision. On
examination, he has exophthalmos, cannot move his right eye upward and blood is noted in the anterior chamber.
Which of the following is the most appropriate course of action at this time?
A. Apply ice packs and cold compresses
B. Immediately refer the patient to an ophthalmologist
C. Attempt to keep the patient calm and order a skull x-ray
D. Administer a dose of intramuscular broad-spectrum antibiotic
Explanations
(u) A. See B for explanation.
(c) B. This scenario describes a "blow-out" fracture of the orbit with hyphema and, because of the signs and
symptoms presented, warrants an immediate consult by an ophthalmologist.
(u) C. See B for explanation.
(u) D. See B for explanation.
Ref: (29)

50. Health Maintenance/Cardiology


Which of the following population groups represent the greatest risk for developing primary hypertension?
A. White non-Hispanic
B. Hispanic
C. Mediterranean
D. Black non-Hispanic
Explanations
(u) A. White non-Hispanic adults have a low risk of hypertension compared to Hispanic and Black non-Hispanics.
(u) B. Hispanic adults are lower risk of hypertension than Black non-Hispanic, but not compared to White nonHispanic.
(u) C. Mediterranean adults have a lower risk of hypertension than Black non-Hispanics.
(c) D. Black non-Hispanic adults have the highest risk of hypertension.
Ref: (28)

51. Clinical Therapeutics/Gastrointestinal/Nutritional


In a patient with metabolic acidosis due to ingestion of methanol, which substance below is considered therapeutic?
A. Salicylates
B. Paraldehyde
C. Ethylene glycol
D. Ethanol
Explanations
(u) A. Salicylates, paraldehyde, ethylene glycol all cause metabolic acidosis with an elevated anion gap.
(u) B. See A for explanation.
(u) C. See A for explanation.
(c) D. Ethanol, along with hemodialysis and supportive measures, is indicated for metabolic acidosis caused by
methanol ingestion.
Ref: (28)

52. History & Physical/Dermatology


Which of the following is the only disease that forms an ulcer at the site of inoculation?
A. Tularemia
B. Scabies
C. Lyme disease
D. Rocky Mountain Spotted Fever

24

Explanations
(c) A. Tularemia is an acute infection that is transmitted by handling the flesh of infected animals, by the bites of
insect vectors and by inoculation of conjunctiva.
(u) B. Scabies is caused by the infestation of a mite and is associated with significant pruritus. There may be minimal
cutaneous findings although a burrow and skin tracks may be seen early in the course of this disease.
(u) C. Lyme disease is associated with a rash known as erythema chronicum migrans. The rash has an expanding
border and central clearing.
(u) D. Systemic symptoms predominate with Rocky Mountain Spotted Fever and the rash has an acral distribution
pattern that may occur on the palms and soles. It does not have any ulcer associated with it.
Ref: (10)
53. Diagnosis/Pulmonology
A 4 year-old patient presents with episodic wheezing and a non-productive cough for the last 4 weeks. His symptoms
are worse at night. Past medical history reveals a history of atopic dermatitis. Physical examination at this time is
unremarkable. Which of the following is the most likely diagnosis?
A. Asthma
B. Bronchiolitis
C. Croup
D. Cystic fibrosis
Explanations
(c) A. Asthma is a chronic inflammatory disorder of the airways. It is characterized by episodic or chronic symptoms
of airflow obstruction, breathlessness, cough, wheezing, and chest tightness. The strongest identifiable predisposing
factor for the development of asthma is atopy.
(u) B. Bronchiolitis is common in infants and young children presenting with acute onset of cough, rhinorrhea,
tachypnea, and expiratory wheezes.
(u) C. Croup usually presents with a prodrome of upper respiratory tract symptoms followed by onset of a barking
cough and stridor.
(u) D. Cystic fibrosis is an autosomal recessive disease and is characterized by a chronic cough, sputum production,
dyspnea, and wheezing. Steatorrhea, diarrhea, and abdominal pain are also common.
Ref: (13)

54. Diagnostic Studies/Obstetrics/Gynecology


A 23 year-old female is in active labor and has progressed from 3 cm to 6 cm in the last six hours. Fetal monitoring
demonstrates mild repetitive late decelerations. Which of the following is the most likely cause of this finding?
A. Fetal hypoxia
B. Head compression
C. Cord compression
D. Uteroplacental insufficiency
Explanations
(u) A. Fetal hypoxia would be a concern if deep late FHR decelerations were present with absent beat-to-beat
variability.
(u) B. Early decelerations are due to head compression of the fetus. Pressure on the fetal head causes an alteration
in cerebral blood flow causing a central vagal stimulation and subsequent FHR deceleration. The deceleration is a
mirror image of the contraction.
(u) C. Variable decelerations are from cord compression. The decelerations have a sharp, angular, decline in FHR
with duration less than 2 minutes.
(c) D. Late decelerations are from uteroplacental insufficiency. The decelerations have a smooth, gradual
symmetrical decrease in FHR beginning at or after the peak of the contraction.
Ref: (4)

55. Clinical Intervention/Orthopedics/Rheumatology


A 60 year-old female injured her right wrist when she slipped and fell onto her outstretched hand. Radiographs show
a fracture through the metaphysis of the distal radius with dorsal displacement and angulation. Which of the following
splints is the best method of temporary immobilization?

25

A. Dorsal forearm
B. Ulnar gutter
C. Volar forearm
D. Volar with thumb spica
Explanations
(u) A. The dorsal forearm splint is best used as an alternative to the ulnar or radial gutter splint for protection of
fractures of metacarpals two through five.
(u) B. Ulnar gutter splints are best for treatment of stable fractures and dislocations of the ulnar wrist and
metacarpals.
(c) C. The volar forearm splint is best for temporary immobilization of forearm, wrist and hand fractures and is the
splint of choice for Colles' fracture.
(u) D. A volar splint with thumb spica is used to immobilize the first metacarpophalangeal joint and is useful for
scaphoid fractures.
Ref: (27)

56. Clinical Therapeutics/Psychiatry/Behavioral Medicine


A 36 year-old patient presents requesting something to help him sleep. He reports that he has always had a problem
sleeping, admits to feeling nervous most days for the last 2 years, and that he has always been "uptight" and a "worry
wart." During the previous eight months he has frequently felt tense, shaky, sweaty, with palpitations and frequent
headaches. He reports being irritable with his 5 year-old son. Which of the following is the best treatment option for
this patient?
A. Alprazolam (Xanax)
B. Haloperidol (Haldol)
C. Paroxetine (Paxil)
D. Diphenhydramine (Benadryl)
Explanations
(u) A. Benzodiazapines should not be used in generalized anxiety disorder (GAD) as they only help symptoms shortterm and should not be used long-term.
(u) B. Antipsychotic medications are not useful in generalized anxiety disorder.
(c) C. SSRI's, specifically Paxil, are the mainstay for treatment of generalized anxiety disorder.
(u) D. Antihistamines are not useful in the treatment of generalized anxiety disorder.
Ref: (28)

57. Diagnosis/Cardiology
A 60 year-old male with hypertension is brought to the emergency department 30 minutes after the sudden onset of
severe chest pain that radiates to his back and arms. His blood pressure is 180/80 mmHg in his left arm; no blood
pressure reading can be obtained from the right arm. ECG shows sinus tachycardia with left ventricular hypertrophy.
A high pitched decrescendo diastolic murmur is heard along the left mid-sternal border. Which of the following is the
most likely diagnosis?
A. Acute myocardial infarction
B. Aortic dissection
C. Pulmonary embolism
D. Right subclavian arterial embolus
Explanations
(u) A. Although included as part of the differential the presentation is not consistent with AMI. ECG may show ST
changes and a murmur of mitral regurgitation may be present with papillary muscle rupture.
(c) B. This is a classic presentation for aortic dissection.
(u) C. Patients will also present with shortness of breath, feelings of impending doom and chest pain that varies with
respirations.
(u) D. Arterial embolus will present with symptoms related to the location of the occlusion. Pain and paresthesias are
usually the earliest symptoms.
Ref: (28)

26

58. Health Maintenance/Dermatology


Which of the following patients is at highest risk for the development of the skin disorder known as erythrasma?
A. Lactating women
B. Postmenopausal women
C. Seniors living in cold northern climate
D. People living in tropical climate
Explanations
(u) A. Lactating women are at increased risk for staph infections in the breast.
(u) B. Postmenopausal women are at increased risk for atrophic vaginitis but not for erythrasma.
(u) C. Seniors living in cold conditions are at risk for winter itch which is worse with dry heat.
(c) D. People living in warm, tropical climate, people wearing occlusive clothing or shoes, obese patients, and those
with hyperhidrosis are at increased risk for erythrasma. The diagnosis of this condition is made by demonstration of a
coral red fluorescence.
Ref: (10)

59. History & Physical/Gastrointestinal/Nutritional


What is the term for blue discoloration about the umbilicus?
A. Cullen's sign
B. Murphy's sign
C. Rovsing's sign
D. Turner sign
Explanations
(c) A. Cullen's sign is a blue discoloration about the umbilicus and can occur in hemorrhagic pancreatitis and results
from hemoperitoneum.
(u) B. Murphy's sign is seen in liver and gallbladder disease by which the patient abruptly halts deep inspiration due
to pain in the right upper quadrant while the examiner's hand is keeping stable pressure to the area.
(u) C. Rovsing's sign is positive when the patient experiences right sided abdominal pain with palpation to the left
side. This is generally seen in those with appendicitis.
(u) D. Turner sign is also seen in severe, acute pancreatitis but is represented by a green-brown discoloration of
bilateral flanks.
Ref: (29)

60. Clinical Therapeutics/ENT/Ophthalmology


A 22 year-old woman presents with sneezing, runny nose, postnasal drip, and nasal congestion for the last week.
She says this happens every spring. She is not allergic to any medications. Which of the following is the most
appropriate pharmacologic treatment for this patient?
A. Azithromycin (Zithromax)
B. Phenylephrine (Neo-synephrine)
C. Nedocromil
D. Pseudoephedrine
Explanations
(u) A. Azithromycin is used to treat bacterial infections not allergic disorders.
(u) B. Neo-synephrine is not indicated in the treatment of allergic rhinitis.
(c) C. Nedocromil inhibits mast cell degranulation and is an effective treatment for allergic rhinitis. It may take 2-6
weeks for full therapeutic effect.
(u) D. Pseudoephedrine is a decongestant that may relieve the nasal congestion, but has no effect on the allergic
response.
Ref: (28)

27

61. Diagnostic Studies/Urology/Renal


A 60 year-old patient presents with elevated blood pressure and peripheral edema. Laboratory testing reveals a BUN
of 58 mg/dl and a creatinine of 4.5 mg/dl, these results are unchanged from six months ago. Urinalysis today is
negative except for the following, specific gravity of 1.002, 2+ protein, and trace glucose. Which of the following
laboratory findings would be most consistent for this patient?
A. Hypercalcemia
B. Metabolic alkalosis
C. Hypokalemia
D. Anemia
Explanations
(u) A. In a patient with chronic renal failure, typical laboratory findings include hypocalcemia, metabolic acidosis, and
hyperkalemia.
(u) B. See A for explanation.
(u) C. See A for explanation.
(c) D. Anemia, due to low erythropoietin, is common in patients with chronic renal failure.
Ref: (11)

62. Scientific Concepts/Pulmonology


Which of the following types of pleural effusion result from increased production of fluid due to underlying
inflammatory conditions?
A. Exudate
B. Empyema
C. Transudate
D. Hemothorax
Explanations
(c) A. Increased production of fluid due to inflammatory or malignant processes results in an exudative pleural
effusion.
(u) B. Infection in the pleural space causes an empyema.
(u) C. Transudates result from increased hydrostatic or decreased oncotic pressures across normal capillaries.
(u) D. Bleeding into the pleural space causes a hemothorax.
Ref: (28)

63. History & Physical/Cardiology


Which of the following conditions is most suggestive of an abdominal aortic aneurysm?
A. Abdominal mass
B. Hypertension
C. Chest pain
D. Syncope
Explanations
(c) A. An abdominal aortic aneurysm presents with a pulsatile upper abdominal mass.
(u) B. Hypertension is not suggestive of an abdominal aortic aneurysm.
(u) C. Abdominal aortic aneurysm presents with midabdominal or lower back pain.
(u) D. Syncope is not common in abdominal aortic aneurysm, unless it ruptures.
Ref: (7)

64. Diagnosis/Obstetrics/Gynecology
A 16 year-old G0P0 patient presents complaining of lower pelvic pain that alternates from right to left side of her
pelvis. She states that it is related to her cycle and occurs most commonly midcycle. She denies sexual activity. She
reports that she has taken ibuprofen at the time of the discomfort with some relief. Her pelvic examination is
unremarkable. Which of the following is the most likely diagnosis?
A. Endometriosis
B. Mittelschmerz
C. Functional ovarian cyst
D. Pelvic inflammatory disease

28

Explanations
(u) A. With endometriosis, the uterus is often fixed and retroflexed in the pelvis. The palpable mass is an
endometrioma or "chocolate cyst". The patient with endometriosis also often has dysmenorrhea, dyspareunia, and
dyschezia.
(c) B. Women may experience pain at the time of ovulation, may alternate side to side.
(u) C. Functional ovarian cysts occur from ovulation and are not usually symptomatic.
(u) D. Patients with pelvic inflammatory disease often present with fever, pain, and more acute symptoms.
Ref: (4)
65. Clinical Intervention/Neurology
A 67 year-old female with history of hypertension, diabetes mellitus, and smoking presents to the emergency
department with mild expressive aphasia, right facial weakness and mild right arm weakness. She had awakened 60
minutes ago and was speaking to her husband when her speech suddenly became difficult to understand and
weakness was noted. Physical examination reveals a blood pressure of 165/85 mm Hg. A CT of the head shows no
intracranial hemorrhage. Which of the following is the most appropriate intervention?
A. Aspirin
B. Warfarin (Coumadin)
C. Tissue plasminogen activator (rt-PA)
D. Clopidogrel (Plavix)
Explanations
(u) A. Aspirin is used for preventative purposes and will not resolve a current embolism.
(u) B. Warfarin is a preventative medicine that will not help the current situation.
(c) C. IV thrombolytic therapy with recombinant tissue plasminogen activator is effective in reducing the neurological
deficit in selected patient without CT evidence of intracranial hemorrhage and when administered within 3 hours after
onset of ischemic stroke.
(u) D. Plavix is a platelet aggregation inhibitor and used for preventative measures.
Ref: (28)

66. Clinical Therapeutics/Gastrointestinal/Nutritional


Which of the following medications decreases the exchange of hydrogen for potassium by inhibiting hydrogen,
potassium-ATPase?
A. Ranitidine (Zantac)
B. Misoprostol (Cytotec)
C. Sucralfate (Carafate)
D. Omeprazole (Prilosec)
Explanations
(u) A. Ranitidine is an H2 receptor antagonist.
(u) B. Misoprostol is a prostaglandin analogue.
(u) C. Sucralfate is a coating agent.
(c) D. Omeprazole, a proton pump inhibitor, works by inhibiting hydrogen, potassium-ATPase.
Ref: (15)

67. Diagnosis/Orthopedics/Rheumatology
A 32 year-old male presents with an acute onset of pain and swelling to his left ankle. On physical examination the
ankle is warm, swollen and erythematous. Evaluation of the synovial fluid reveals only leukocytosis with a low
glucose. Which of the following is the most likely diagnosis?
A. Gout
B. Pseudogout
C. Acute rheumatic fever
D. Septic arthritis
Explanations
(u) A. Gout and pseudogout are excluded by the failure to find crystals on synovial fluid analysis.
(u) B. See A for explanation.
(u) C. Acute rheumatic fever commonly involves multiple joints.
(c) D. Leukocytosis and a low synovial glucose are indicative of septic arthritis.
Ref: (28)

29

68. History & Physical/Endocrinology


Which of the following clinical signs or symptoms is most likely to be present following iatrogenic injury during subtotal
thyroidectomy?
A. Inability to shrug the shoulders
B. Hoarseness
C. Impaired gag reflex
D. Impaired taste sensation anterior two-thirds of tongue
Explanations
(u) A. Patients with injury to the spinal accessory (Cranial Nerve XI) will be unable to shrug the shoulders against
resistance. This nerve is most likely to be injured during carotid endarterectomy and not subtotal thyroidectomy.
(c) B. The recurrent laryngeal nerve is most commonly injured during subtotal thyroidectomy. If injured, hoarseness is
the most common presentation for this nerve impairment.
(u) C. Injury to the glossopharyngeal and vagus nerves are most likely to result in impaired gag reflex. These nerves
are most likely to be injured during carotid endarterectomy and not during subtotal thyroidectomy.
(u) D. The facial nerve (Cranial Nerve VII) provides taste to the anterior two-thirds of the tongue. It is not at risk for
injury during subtotal thyroidectomy.
Ref: (3)

69. Diagnostic Studies/Infectious Diseases


A patient states that he has been camping in the mountains of North Carolina for the past two weeks. He presents to
the clinic complaining of "flu-like" symptoms for the past 10-14 days however he notes that he started to develop a
slight rash on his wrist and ankles about seven days ago. Which of the following tests would provide a confirmatory
diagnosis?
A. Immunofluorescent assay
B. C-reactive protein
C. Heterophile agglutination
D. Anti-streptolysin O titer
Explanations
(c) A. The immunofluorescent assay will confirm antibodies to Rickettsia.
(u) B. C-reactive protein is a protein found in patients with inflammation during the acute phase illness. It is nonspecific.
(u) C. Heterophile agglutination is used to detect antibodies to the Epstein-Barr virus which is responsible for
infectious mononucleosis.
(u) D. ASO titer is done to detect antibodies to streptococcal infection.
Ref: (28)

70. Health Maintenance/Cardiology


Who is most likely to require subacute bacterial endocarditis (SBE) prophylaxis prior to a dental procedure?
A. 22 year-old female with mitral valve prolapse
B. 36 year-old male with a bio-prosthesic mitral valve
C. 45 year-old female with an ASD closure 8 months ago with no residual defect
D. 15 year-old male with a bicuspid aortic valve
Explanations
(u) A. See B for explanation.
(c) B. The AHA recommends that patients with prosthetic heart valves receive antibiotic prophylaxis. As should
cardiac transplant recipients with valve disease, unrepaired cyanotic CHD, repaired CHD with prosthetic material or
device during the first six months of the procedure and repaired CHD with residual defects at site of patch or
prosthetic device.
(u) C. See B for explanation.
(u) D. See B for explanation.
Ref: (28)

30

71. Clinical Intervention/Pulmonology


A 3 month-old male presents with two days of worsening hoarse cough and thick purulent rhinorrhea associated with
increasing problems breathing and trouble feeding. Examination reveals a temperature of 100.2 degrees F and
respiratory rate of 80/minute with nasal flaring and retractions. Lung examination reveals a prolonged expiratory
phase with inspiratory rales. He is tachycardic. Pulse oximetry reveals oxygen saturation of 89%. Chest x-ray reveals
hyperinflation with diffuse interstitial infiltrates. Which of the following is the most appropriate intervention?
A. Antibiotics
B. Hospitalization
C. Inhaled corticosteroids
D. Racemic epinephrine
Explanations
(u) A. Antibiotics are utilized to treat bacterial, not viral, illnesses.
(c) B. This infant most likely has bronchiolitis. While most cases are mild and can be treated at home, hospitalization
is recommended for infants with hypoxia on room air, moderate tachypnea with feeding difficulties and marked
respiratory distress with retractions. Additionally hospitalization is recommended for infants less than 2-3 months of
age, a history of apnea or an underlying chronic cardiopulmonary disease.
(u) C. The use of corticosteroids in children with bronchiolitis has not been studied and does not appear to be helpful.
(u) D. Racemic epinephrine is not indicated in the treatment of bronchiolitis.
Ref: (14)

72. Clinical Therapeutics/Obstetrics/Gynecology


A patient with preterm labor may be given corticosteroids to
A. decrease uterine activity.
B. prevent chorioamnionitis.
C. enhance fetal lung maturity.
D. prevent the development of gestational diabetes.
Explanations
(u) A. Tocolytics are given to decrease uterine activity with preterm labor.
(u) B. IV antibiotics are given to patients with chorioamnionitis.
(c) C. Corticosteroids may be given from 24-34 weeks in patients with preterm labor or who have pregnancy
complications which may cause premature birth. The corticosteroids enhance pulmonary maturity.
(u) D. There are no medications to prevent the development of gestational diabetes, however, patients who have
gestational diabetes may be treated with a diabetic diet and/or insulin to manage the condition and decrease
complications.
Ref: (4)

73. History & Physical/ENT/Ophthalmology


Which of the following would indicate an optic nerve lesion?
A. Excessive conjunctival edema
B. Ptosis
C. Inability to gaze laterally
D. Afferent pupillary defect
Explanations
(u) A. Excessive edema of the conjunctiva is a feature of chemosis.
(u) B. Ptosis is not indicative of an optic nerve lesion.
(u) C. Inability to gaze laterally would be due to paralysis of the lateral rectus muscle controlled by cranial nerve VI.
(c) D. Pupil size, controlled centrally by the Edinger-Westphal nucleus in the midbrain, is primarily based on the
afferent light stimulus transmitted via the optic nerve.
Ref: (3)

31

74. Scientific Concepts/Cardiology


When evaluating jugular venous pulsations a prominent a wave represents which of the following?
A. Atrial contraction against a closed tricuspid valve
B. Rapid filling of the right atrium
C. Tricuspid regurgitation
D. Poor left ventricle compliance
Explanations
(c) A. The a wave corresponds to right atrial contraction.
(u) B. See A for explanation.
(u) C. See A for explanation.
(u) D. See A for explanation.
Ref: (11)

75. Diagnosis/Psychiatry/Behavioral Medicine


A patient presents with an episode of an expansive, elevated mood during which she cleaned excessively without
sleeping. Which of the following is the most likely diagnosis?
A. Major depressive disorder
B. Bipolar disorder
C. Schizoaffective disorder
D. Dysthymic disorder
Explanations
(u) A. Criteria for major depressive disorder do not include elevated expansive moods.
(c) B. Bipolar disorder is characterized by episodic mood shifts from depression to manic type moods which is often
rapid with depression lasting longer than manic episodes. Bipolar disorder may initially present with a manic episode.
(u) C. Schizoaffective disorder presents with a mood disorder and characteristics of schizophrenia.
(u) D. Dysthymia has no elevated moods or manic type behaviors.
Ref: (28)

76. Clinical Intervention/Urology/Renal


A 27 year-old patient presents with crush injuries to both lower extremities after being involved in an automobile
accident. Within six hours of the accident, urine output has decreased to less than 10 cc per hour. Within 24 hours
the serum creatinine increased from 0.9 mg/dl to 2.9 mg/dl and serum CPK is now 12,000 U/L. In addition to a fluid
challenge with normal saline, which of the following is the most appropriate treatment as this time?
A. IV calcium chloride
B. Oral captopril (Capoten)
C. IV sodium bicarbonate
D. Oral sevelamer (Renagel)
Explanations
(u) A. While hypocalcemia is common in rhabdomyolysis and acute renal failure, treatment is not needed unless
clinical signs such as tetany are noted.
(h) B. The use of ACE inhibitors, such as captopril, may worsen renal function and are not indicated in this case.
(c) C. Treatment of acute renal failure due to rhabdomyolysis is best accomplished with IV fluids and forced alkaline
diuresis.
(u) D. Sevelamer is a phosphate binder used to treat elevated phosphate levels in patients with end stage renal
disease.
Ref: (11)

32

77. Clinical Therapeutics/Neurology


A 38 year-old female has a 10 year diagnosis of migraine headaches. She had been using ergotamine to abort her
headaches, but is now having one or two headaches per week that are interfering with work. Which of the following is
the most appropriate preventive therapy?
A. Sumatriptan (Imitrex)
B. Promethazine (Phenergan)
C. Propranolol (Inderal)
D. Ketorolac (Toradol)
Explanations
(u) A. Imitrex is not approved for preventive therapy but is used as abortive therapy.
(u) B. Phenergan is an antiemetic that helps with nausea and vomiting for people who get migraines. It is not a
preventive medication.
(c) C. Propanolol is one beta blocker that is frequently used as a first-line prophylaxis for migraines.
(u) D. Ketorolac is not indicated for scheduled, daily use as a preventive for migraines.
Ref: (28)

78. History & Physical/Cardiology


An 18 year-old male high school basketball player comes to clinic for a routine physical exam. His height is 193 cm
(76 in.); arm span is 201 cm (79 in.). He has long fingers and toes. Blood pressure is 146/62 mmHg and pulse is
64/min. Which of the following exam findings is most consistent with the diagnosis?
A. Grade 2/6 high-frequency diastolic murmur at the third right intercostal space
B. Grade 2/6 systolic ejection murmur at the second left intercostal space with a fixed widely split S2
C. Grade 2/6 continuous murmur heard best at the high left sternal border
D. Grade 2/6 systolic murmur at the fourth left intercostal space that decreases with squatting
Explanations
(c) A. This murmur is most consistent with aortic regurgitation which can be present in patients with Marfans
syndrome and a dilated aortic root.
(u) B. This murmur is most consistent with an atrial septal defect.
(u) C. This murmur is most consistent with a patent ductus arteriosus and unlikely in this age patient.
(u) D. This murmur is most consistent with hypertrophic obstructive cardiomyopathy.
Ref: (28)

79. Diagnostic Studies/Dermatology


A patient is concerned about being exposed to condyloma acuminata. Which of the following tests is most
appropriate to perform in order to better identify these lesions?
A. Acetowhitening
B. Tzanck smear
C. Potassium hydroxide test
D. Wood's light fluoroscopy
Explanations
(c) A. Acetowhitening is used to facilitate the diagnosis of condyloma acuminata lesions. A 3 to 5% acetic acid
solution is applied to these suspected genital warts for five to ten minutes. Condyloma lesions will whiten and appear
as circumscribed macular or papular lesions with a granular surface.
(u) B. Tzanck smears are used in making the diagnosis of herpetic lesions, such as herpes simplex, varicella, and
herpes zoster infection.
(u) C. Potassium hydroxide tests are used to rule out the presence of dermatophyte (fungal and yeast) infections.
(u) D. Wood's light fluoroscopy is used in the assessment of dermatophyte infection and a bacterial infection known
as erythrasma.
Ref: (2)

33

80. Health Maintenance/Pulmonology


Which of the following is an indication for a pediatric patient to receive the 23-valent polysaccharide vaccine
(Pneumovax)?
A. Children at any age with a history of asthma
B. All children at 2,4,6 and 12-18 months of age
C. All children at 12-23 months of age in a two dose series
D. Children age 24-59 months at high risk for invasive pneumococcal disease
Explanations
(u) A. Pediatric patients with cystic fibrosis, not asthma, are included in the indications for vaccination with
Pneumovax, however they must be at least 2 years old.
(u) B. The 7-valent pneumococcal conjugate vaccine (Prevnar) is currently recommended to be given to children
under the age of two on the schedule outlined.
(u) C. While a two dose series is recommended for appropriate pediatric patients that receive Pneumovax, the
recommended timing between doses is 3-5 years.
(c) D. Pneumovax is licensed for use in children over the age of 23 months and is indicated for all pediatric patients at
increased risk for pneumococcal disease.
Ref: (13)

81. Diagnosis/Endocrinology
A patient presents to the office with worsening fatigue, weight loss, and weakness. She notes that she is having
recurrent bouts of abdominal pain and has been losing her pubic hair. Patient is found to have orthostatic
hypotension. Which of the following conditions is most likely?
A. Cushing's syndrome
B. Pheochromocytoma
C. Primary hyperparathyroidism
D. Addison's disease
Explanations
(u) A. Cushing's syndrome is caused by an increase in the cortisol levels in the body. These patients will have
hypertension, buffalo hump, striae, and proximal muscle weakness.
(u) B. Pheochromocytoma is caused by an increase in the release of catecholamines from the adrenal medulla.
These patients will have episodic hypertension followed by sustained hypertension and bouts of diaphoresis and
shakiness.
(u) C. Patients with primary hyperparathyroidism are most likely to be asymptomatic. If these patients have
symptoms, it is most likely that they will have abdominal pain, renal stones, and bone pain because of the resultant
increase in the serum calcium levels.
(c) D. Patients with Addison's disease have primary adrenal failure from an autoimmune problem in the adrenal gland
or due to hemorrhage into the adrenal gland. These patients are not able to make glucocorticoids, mineralocorticoids,
or sex hormones which result in hypotension, hyperpigmentation (from an increase in the ACTH and MSH hormones)
and are hyponatremic.
Ref: (28)

82. Clinical Intervention/Gastrointestinal/Nutritional


A patient presents to the emergency department with right upper quadrant pain over eight hours, nausea, and
vomiting. On exam there is a fever of 101.2 degrees F. Ultrasound shows a distended gallbladder. What is the most
appropriate management of this patient?
A. Oral analgesics
B. Diagnostic peritoneal lavage
C. Proton pump inhibitors
D. Laparoscopic cholecystectomy
Explanations
(u) A. See D for explanation.
(h) B. Diagnostic peritoneal lavage is used to detect intraabdominal bleeding from trauma and not to treat acute
cholecystitis and may delay appropriate treatment.
(u) C. Proton pump inhibitors are used to treat GERD or PUD.
(c) D. The proper treatment for acute cholecystitis is IV fluids, antibiotics, pain control, and surgery. Cholecystectomy
is the definitive treatment for acute cholecystitis and laparoscopic cholecystectomy is the procedure of choice.
Ref: (25)

34

83. Clinical Therapeutics/Hematology


A 56 year-old male is noted to have a recent diagnosis of polycythemia vera. His current hemoglobin is 21 gms/dl.
What treatment should be instituted for this patient at this time?
A. Phlebotomy
B. Iron chelation therapy
C. Bone marrow radiation therapy
D. Normal saline IV hydration
Explanations
(c) A. This patient has a diagnosis of polycythemia vera treatment begins with phlebotomy instituted on a weekly
basis until the hematocrit is less than 45%. Maintainance of the hematocrit at 45% is achieved with repeated
phlebotomy as necessary.
(u) B. See A for explanation.
(h) C. Radiation therapy has no place in this treatment.
(h) D. IV hydration with normal saline will place the patient into volume overload.
Ref: (28)

84. History & Physical/Obstetrics/Gynecology


An 18 year-old female comes to the clinic with the complaint of increased vaginal discharge and vaginal odor. She
also complains of urinary frequency. On physical examination there is evidence of thin, gray, frothy discharge in the
vagina. The cervix appears erythematous and the vaginal pH is 6. Which of the following is the most likely diagnosis?
A. Candida vaginitis
B. Bacterial vaginosis
C. Trichomonas vaginitis
D. Chlamydia trachomatis
Explanations
(u) A. Patients with a yeast infection most often present with vulvar/vaginal pruritus, burning, dyspareunia "cottagecheese" discharge which is odorless, pH is often normal 4-4.5.
(u) B. Symptoms from bacterial vaginosis include ivory to gray discharge, thin, homogeneous, adherent, often
increased pH 5-6.5 (basic), distinctive "fishy" odor, itching may be present. Malodorous discharge is especially
noticeable by the patient after menses or intercourse.
(c) C. Signs of trichomonas include: thin frothy or bubbly, pale yellow-green to gray adherent vaginal discharge, can
have erythema of vulva and vagina, may have petechiae on the cervix, amine odor may be present, may also
complain of dysuria and dyspareunia, pH 5 to 6.5 (basic).
(u) D. Patients with Chlamydia are often asymptomatic but may have mucopurulent discharge and cervical
inflammation.
Ref: (4)

85. Diagnostic Studies/Orthopedics/Rheumatology


An obese 15 year-old male presents with complaint of a limp and right knee pain for two weeks. He denies recent
trauma or history of previous injury. Physical examination of the right knee is unremarkable. Examination of the right
hip reveals pain with passive range of motion and limited internal rotation and abduction. Flexion of the hip results in
external rotation of the thigh. Gait is antalgic with the right hip externally rotated. Which of the following radiographic
findings supports the most likely diagnosis?
A. Displacement of the femoral epiphysis
B. Irregularity and fragmentation of the joint space
C. Capsular swelling of the joint
D. Dislocation of the hip
Explanations
(c) A. This patient has slipped capital femoral epiphysis (SCFE) and the classic x-ray findings will demonstrate
displacement of the femoral head rotation of the femoral neck anteriorly.
(u) B. Irregularity and fragmentation of the joint space is associated with avascular necrosis of the femoral head as
seen in Legg-Calve-Perthes disease. This typically occurs in a younger male population and is not associated with
the classic externally rotated hip with ambulation seen in SCFE.
(u) C. Capsular swelling of the joint may be seen in transient synovitis of the hip but is not associated with SCFE.
(u) D. Hip dislocation at this age is associated with major trauma, such as that sustained in a fall from height or
dashboard injury. SCFE does not lead to hip dislocation.
Ref: (5)

35

86. Scientific Concepts/Cardiology


Which of the following is the most likely initial effect on the left ventricle from aortic stenosis?
A. Dilitation of the ventricle with diastolic dysfunction
B. Wall stiffness due to ischemia from decreased coronary blood flow
C. Paradoxical wall motion abnormalities due to increased preload
D. Concentric hypertrophy with preserved function
Explanations
(u) A. Dilitation of the ventricle is a later finding.
(u) B. This is not an effect from aortic stenosis but coronary artery blockage.
(u) C. Preload is the end-diastolic pressure or volume within the ventricle, ischemic heart disease or infarct would
cause paradoxical wall motion abnormalities due to the increased preload.
(c) D. Hypertrophy would be the initial changes of the left ventricle as a response to the increased pressure.
Ref: (28)
87. Diagnosis/ENT/Ophthalmology
A 56 year-old female presents complaining of intense left eye pain associated with unilateral headache, nausea, and
colored rings around lights. On examination you note decreased visual acuity, a pupil that is fixed and mid-dilated,
and ciliary flushing. Which of the following is the most likely diagnosis?
A. Acute glaucoma
B. Migraine
C. Episcleritis
D. Acute uveitis
Explanations
(c) A. Acute glaucoma is an ocular emergency that presents as an acutely painful eye and elevated intraocular
pressure. Patients typically complain of acute eye pain associated with unilateral headache, nausea/vomiting, cloudy
vision, and colored rings around lights. On exam the pupil is fixed and mid dilated with prominent ciliary flush.
(u) B. Migraine headaches have associated unilateral headache and nausea however there would be no pupillary
changes.
(u) C. Episcleritis is an inflammation of the thin layer of connective tissue between the conjunctiva and sclera.
Episcleritis resembles conjunctivitis but is a more localized process and discharge is absent.
(u) D. Acute uveitis is frequently due to systemic disorders associated HLA-B27-related conditions ankylosing
spondylitis, reactive arthritis, psoriasis, ulcerative colitis, and Crohn's disease. The pupil is usually small, inflammatory
cells and flare within the aqueous are present.
Ref: (28)
88. Clinical Therapeutics/Pulmonology Topic: 1e Author: Donna Yeisley
A 2 month-old infant has been diagnosed with pneumonia due to Chlamydia trachomatis. Which of the following is the
treatment of choice?
A. Ceftriaxone (Rocephin)
B. Doxycycline
C. Levofloxacin (Levaquin)
D. Erythromycin
Explanations
(u) A. Ceftriaxone is a third-generation cephalosporin that may be safely used in children, however is not indicated for
the treatment of Chlamydial pneumonia.
(h) B. Doxycycline is a tetracycline and is contraindicated in children under eight years of age secondary to damaging
effects on bone and teeth enamel.
(h) C. Levofloxacin is a fluoroquinolone and is contraindicated for use in children under 18 years of age secondary to
damaging effects that may occur with growing cartilage.
(c) D. Erythromycin or sulfisoxazole is the treatment of choice for an infant with Chlamydial pneumonia.
Ref: (13)

36

89. Clinical Intervention/Psychiatry/Behavioral Medicine


Which of the following is the most appropriate intervention for a patient suffering from a specific phobia, such as fear
of snakes?
A. Lithium
B. Behavioral therapy
C. Insight-oriented therapy
D. Electroconvulsive therapy
Explanations
(u) A. Lithium is indicated for treatment of bipolar disease, not phobias.
(c) B. Specific phobias most commonly are treated with behavioral therapy including exposure therapy utilizing
systemic desensitization. Hypnosis, supportive therapy and family therapy may also be useful adjunct treatment.
(u) C. Insight-oriented therapy, while helpful in allowing patients to understand the origin of their fear and potential
reasons for secondary gain, will not help to alleviate the patient's phobic symptoms.
(u) D. The primary indication for electroconvulsive therapy (ECT) is in treatment of a major depressive disorder that
has not responded to medical management.
Ref: (14)

90. Health Maintenance/Endocrinology


A patient is diagnosed with hypothyroidism and started on levothyroxine (Synthroid). When is it ideal to recheck the
TSH level?
A. Two weeks
B. Three to four weeks
C. Two to three months
D. Six months
Explanations
(u) A. See B for explanation.
(c) B. Patients taking levothyroxine (Synthroid) for thyroid replacement will achieve peak levels of T4 within three to
four weeks. The half-life of levothyroxine is 7 days so it will take three to four weeks in order to achieve a steady state
which means that TSH levels or T4 levels should not be checked sooner than this recommended time of three to four
weeks.
(u) C. See B for explanation.
(u) D. See B for explanation.
Ref: (28)

91. Diagnostic Studies/Gastrointestinal/Nutritional


A patient presents uncertain as to whether he has completed the Hepatitis B vaccination series. What laboratory test
would be most helpful in determining his immunization status?
A. Anti-HBs
B. HBeAg
C. HBsAg
D. Anti-HBc
Explanations
(c) A. Anti-HBs is the circulating antibody that develops to surface antigen in response to either past Hepatitis B
infection or to Hepatitis B immunization. It represents immunity to the Hepatitis B virus in otherwise
immunocompetent patients.
(u) B. HBeAg is seen with active disease and not from immunization.
(u) C. The surface antigen to Hepatitis B (HBsAg) is most often used to show acute infection and not immunity.
(u) D. Anti-HBc is seen with active disease and not from immunization.
Ref: (7)

37

92. Diagnosis/Cardiology
A 65 year-old female who recently had an anterior MI returns to clinic for follow-up six weeks after. She has no chest
pain, but reports decreased exercise tolerance. Electrocardiogram (ECG) shows persistent ST elevation in leads V2V4. Which of the following is the most likely diagnosis?
A. Right ventricular infarction
B. Re-occlusion of the right coronary artery
C. Pericarditis
D. Ventricular aneurysm
Explanations
(u) A. RV infarction is present in one-third of patients who had an inferior wall MI and typically show ST elevation in
V4 along with the inferior leads.
(u) B. Occlusion of the right coronary artery would show ST elevation in the inferior ECG leads.
(u) C. Pericarditis shows ST elevation in multiple leads with a history consistent of a viral illness or fever.
(c) D. Persistent ST elevation in the leads where a previous or recent anterior MI occurred is most likely due to
ventricular aneurysm.
Ref: (28)

93. History & Physical/Pulmonology


Which of the following is a physical examination finding that is consistent with a diagnosis of lobar pneumonia?
A. Resonant to percussion
B. Late inspiratory crackles
C. Decreased tactile fremitus
D. Tracheal shift toward uninvolved side
Explanations
(u) A. Physical examination findings that are consistent with a diagnosis of lobar pneumonia include dullness to
percussion, increased tactile fremitus and a trachea that is midline.
(c) B. Late inspiratory crackles are a physical examination finding that is consistent with lobar pneumonia.
(u) C. See A for explanation.
(u) D. See A for explanation.
Ref: (3)

94. Clinical Therapeutics/Urology/Renal


A 35 year-old male presents with fever, perineal pain, and dysuria. On physical examination, the patient is toxicappearing, febrile, and his prostate is very tender to palpation. Laboratory testing reveals leukocytosis, pyuria, and
bacteriuria. Which of the following is the treatment of choice for this patient?
A. Ampicillin and gentamicin
B. Ceftriaxone and doxycline
C. Trimethoprim-sulfamethoxazole
D. Nitrofurantoin
Explanations
(c) A. Acute prostatitis is best treated acutely with parenteral antibiotics, such as ampicillin and gentamicin.
(u) B. Ceftriaxone and doxycycline are used in the treatment of acute epididymitis due to sexually transmitted
infection.
(u) C. Trimethoprim-sulfamethoxazole can be used but is second line in toxic patients and is best used after the
patient is stable.
(u) D. Nitrofurantoin is used in the treatment of acute cystitis and not indicated in acute prostatitis.
Ref: (28)

38

95. Clinical Intervention/Orthopedics/Rheumatology


A 57 year-old male presents with acute bilateral lower extremity weakness and urinary incontinence that began after
he fell earlier today. His examination is significant for bilateral lower extremity sensory deficits and weakness along
with decreased rectal sphincter tone. Which of the following is the most appropriate intervention?
A. Epidural steroids
B. Oral NSAIDs
C. Physical therapy
D. Surgery
Explanations
(h) A. While epidural steroids can be effective in treating lumbar disc herniation, in the case of cauda equina
syndrome, immediate surgical decompression is mandatory.
(h) B. NSAIDs may be beneficial in some cases of lumbar muscle strain and disc herniation. They are not appropriate
for management of cauda equina syndrome, immediate surgical decompression is mandatory.
(h) C. Physical therapy may be beneficial in some cases of lumbar muscle strain and disc herniation but it is not
appropriate for management of cauda equina syndrome.
(c) D. Cauda equina syndrome is a rare but serious surgical emergency because the duration of nerve compression
is inversely correlated with the likelihood of full neurologic recovery.
Ref: (27)

96. Health Maintenance/Neurology


A 45 year-old male presents for a routine physical. His mother and father both had ischemic strokes in their 70's. He
does not smoke. His blood pressure is 128/80 mmHg, pulse 78/minutes and regular, respiratory rate of 12/minute. He
has no bruits on examination. Which of the following studies should be ordered to further evaluate this patient's risk of
stroke?
A. Electrocardiogram
B. Fasting lipid profile
C. Carotid Doppler ultrasound
D. MRI with gadolinium
Explanations
(u) A. ECG does not show risk of stroke.
(c) B. This patient has a genetic risk for stroke secondary to family history. Having an elevated LDL cholesterol and
low HDL cholesterol puts him at a greater risk for stroke.
(u) C. Carotid Doppler US will demonstrate the presence of a plaque in the carotid arteries, but still does not evaluate
stroke risk.
(u) D. MRI with contrast will only show abnormalities in structure which cannot predict risk of stroke. Often abnormal
findings are picked up on MRI that have no relationship to a patient's symptoms.
Ref: (7)

97. Scientific Concepts/Obstetrics/Gynecology


Mastitis associated with breastfeeding is most commonly caused by what bacteria?
A. Listeria monocytogenes
B. Escherichia coli
C. Staphylococcus aureus
D. Streptococcus pyogenes
Explanations
(u) A. See C for explanation.
(u) B. See C for explanation.
(c) C. The most common pathogen associated with postpartum mastitis is Staphylococcus aureus which arises from
the nursing infant's throat and nose.
(u) D. See C for explanation.
Ref: (4)

39

98. Diagnosis/Gastrointestinal/Nutritional
A patient with a history of severe peptic ulcer disease is 5 weeks status post Billroth I surgery. One week ago he
restarted his normal diet and has had the onset of severe nausea, abdominal cramping, and light-headedness that
occur approximately thirty minutes after eating. The abdominal exam reveals a healing surgical scar without areas of
unusual tenderness or any palpable masses. Which of the following is the most likely diagnosis?
A. Anxiety disorder
B. Celiac sprue
C. Dumping syndrome
D. Irritable bowel syndrome
Explanations
(u) A. Anxiety disorders can cause a wide variety of somatic syndromes such as those mentioned, but generally not
with such a straightforward dietary trigger.
(u) B. Celiac sprue can cause similar symptoms as those listed, can develop at any age and can be worsened by the
ingestion of gluten containing products. The patient's recent surgery makes dumping syndrome a much greater
possibility.
(c) C. Dumping syndrome typically occurs after Billroth type I surgeries as well as gastric bypass surgeries when the
patient attempts to eat a large amount of simple sugars.
(u) D. Irritable bowel syndrome is a diagnosis of exclusion and is associated with alternation in bowel habits.
Ref: (7)

99. Clinical Therapeutics/ENT/Ophthalmology


An 18 year-old patient is diagnosed with bacterial conjunctivitis. Gram stain reveals gram-negative intracellular
diplococci. Which of the following is the most appropriate treatment of this infection?
A. Gentamicin
B. Penicillin
C. Bactrim
D. Ceftriaxone
Explanations
(u) A. Gentamicin is used in the treatment of gram negative rods and does not provide coverage of Neisseria
gonorrhoeae.
(u) B. Penicillin does provide coverage for some Neisseria species but is not considered the treatment of choice for
Neisseria gonorrhoeae because of increased resistance.
(u) C. Bactrim is used to treat Chlamydia trachoma not Neisseria gonorrhoeae.
(c) D. Neisseria gonorrhoeae is confirmed by the Gram stain findings of intracellular gram-negative diplococci. The
treatment of choice for Neisseria gonorrhoeae is ceftriaxone.
Ref: (28)

100. History & Physical/Orthopedics/Rheumatology


Which of the following is the most sensitive to determine whether there is a small effusion in the knee?
A. Compress the patella and move it against the femur, noting any crepitus
B. Flex the knee to about 90 degrees and palpate for tenderness over the joint line
C. Milk the medial aspect of the knee, press lateral margin of the patella, and note a bulge of returning fluid medial to
the patella
D. Force fluid into space between the patella and the femur, tap the patella over the femur to detect a click
Explanations
(u) A. Crepitus without pain is not significant and does not indicate an effusion.
(u) B. Tenderness over the joint line indicates a meniscal injury, but does not demonstrate an effusion.
(c) C. A small bulge of returning fluid after milking fluid upward from the knee is useful for detecting small effusions.
(u) D. Ballottement of the patella against the femur is useful for detecting large effusions, but not small ones.
Ref: (3)

40

101. Diagnostic Studies/Cardiology


A 15 year-old male is brought to the emergency department 1 hour after an episode of syncope while running in a
400-meter race. He had a similar episode 2 years ago. His mother and maternal first cousin died suddenly at the
ages of 32 and 17 years, respectively. Examination shows abrasions of the face, hands, and knees. Neurologic
examination shows no abnormalities. Which of the following is the most appropriate initial test?
A. Tilt table test
B. CT scan of the head
C. Electrocardiogram
D. Electroencephalography
Explanations
(u) A. Tilt table testing should be performed after structural heart disease has been ruled out.
(u) B. CT scanning of the head would not be the initial test of choice.
(c) C. ECG is the most appropriate to rule out any underlying rhythm abnormalities, Holter or event monitoring would
also be included.
(u) D. EEG testing routinely would not be helpful as an initial test in this patient.
Ref: (11)

102. Clinical Intervention/Pulmonology


A 42 year-old male is brought to the emergency department with a stab wound to his right lateral chest wall. On
physical examination, the patient is stable with decreased breath sounds on the right with dullness to percussion. An
upright chest x-ray reveals the presence of a moderate pleural effusion. Subsequent diagnostic thoracentesis
contains bloody aspirate. Which of the following is the next most appropriate intervention?
A. Thoracotomy
B. Needle aspiration
C. Close observation
D. Tube thoracostomy
Explanations
(u) A. A thoracotomy is indicated in a small percentage of patients that do not respond to IV administration of fluids
and evacuation of the hemothorax via tube thoracostomy.
(u) B. Needle aspiration as treatment for a hemothorax is not recommended as it fails to adequately drain the bloody
fluid.
(u) C. Close observation is only indicated for small effusions in a patient that remains hemodynamically stable.
(c) D. This patient has a hemothorax. Drainage of a hemothorax is best obtained through insertion of a chest tube
(tube thoracostomy).
Ref: (29)

103. Health Maintenance/Infectious Diseases


Routine prophylaxis for patients going to an area of malaria infestation requires the administration of which
chemoprophylaxis agent?
A. Chloroquine
B. Ciprofoxacin
C. Tetracycline
D. Erythromycin
Explanations
(c) A. The use of Chloroquine is still recommended as the drug of choice for prophylaxis in areas of non-resistant
falciparum malaria.
(u) B. The use of quinolones, tetracyclines, and macrolides is not recommended for the prevention of malaria.
(u) C. See B for explanation.
(u) D. See B for explanation.
Ref: (28)

41

104. Diagnosis/Orthopedics/Rheumatology
A 41 year-old female complains of 3 weeks of gradually worsening pain at the base of the thumb and radial aspect of
the wrist. She and her husband have been renovating their home for the past 2 months and it has become
increasingly difficult for her to hold a hammer. She denies numbness or tingling. She denies any history of previous
trauma to the wrist. On examination, there is tenderness over the distal radial styloid and pain reproduced with ulnar
deviation of a fist clenched over the abducted thumb. Which of the following is the most likely diagnosis?
A. Carpal tunnel syndrome
B. deQuervain's tenosynovitis
C. Ganglion cyst
D. Volar flexor tenosynovitis
Explanations
(u) A. Carpal tunnel syndrome typically presents with pain and paresthesias in the median nerve distribution.
(c) B. deQuervain's tenosynovitis typically results from repetitive activity involving pinching the thumb while moving
the wrist. There is often pain and tenderness over the radial styloid and Finkelstein's is positive in this patient.
(u) C. Ganglion cysts classically present with a visible or palpable, usually painless swelling over the dorsum of the
wrist.
(u) D. With volar flexor tenosynovitis, pain is expected with extension of the fingers and localized tenderness of the
volar tendon sheaths.
Ref: (19)

105. Scientific Concepts/Urology/Renal


Which of the following is the most common cause of acute epididymitis in men under the age of 40?
A. Chlamydia trachomatis
B. Ureaplasma urealyticum
C. Pseudomonas aeruginosa
D. Escherichia coli
Explanations
(c) A. In men under the age of 40, acute epididymitis is typically caused by Chlamydia trachomatis and Neisseria
gonorrhoeae.
(u) B. See A for explanation.
(u) C. See A for explanation.
(u) D. See A for explanation.
Ref: (28)

106. History & Physical/Dermatology


The bite from which of the following is associated with fever, lacrimation, rhinorrhea, bradycardia, hypertension, and
tachyarrhythmias?
A. Brown recluse spider
B. Black widow spider
C. Black flies
D. Bedbugs
Explanations
(u) A. Brown recluse spider bites may cause reactions ranging from mild urticaria to full thickness necrosis but
typically, there are no systemic symptoms.
(c) B. Black widow spider bites can inject venom that contains a neurotoxin which can produce reactions at the site of
the bite along with varying degrees of systemic symptoms.
(u) C. Black fly bites produce local reactions as well as fever, nausea, and general lymphadenopathy.
(u) D. Bedbugs have nocturnal feedings that produce a linear arrangement of papular urticaria. There are no systemic
symptoms associated with their bites.
Ref: (10)

42

107. Clinical Therapeutics/Cardiology


What is the optimal INR for a patient with a mechanical mitral valve prosthesis on warfarin (Coumadin)?
A. 1.5-2.5
B. 2.0-3.0
C. 2.5-3.5
D. 3.5-4.5
Explanations
(u) A. See C for explanation.
(u) B. See C for explanation.
(c) C. Patients with mechanical MV prostheses should maintain an INR between 2.5-3.5. Aortic mechanical valves
can be maintained at an INR of 2.0-3.0.
(u) D. See C for explanation.
Ref: (28)

108. Clinical Intervention/Obstetrics/Gynecology


A 40 year-old female presents with a Pap smear abnormality revealing atypical glandular cells (AGUS). What is the
most appropriate intervention?
A. HPV DNA testing
B. Colposcopy with endometrial curretage(ECC)
C. Repeat Pap smear in 3 months
D. Colposcopy and endometrial sampling
Explanations
(u) A. HPV DNA testing is recommended to further evaluate patients with Pap smears with dysplasia.
(u) B. Colposcopy with ECC would be recommended in patients with ASCUS, LGSIL, HGSIL, or squamous cell
findings on a Pap smear.
(u) C. Repeat Pap smear would be recommended in patients with ASCUS, not AGUS, results on a Pap smear.
(c) D. Colposcopy and endometrial sampling are important to perform in patients with AGUS Pap results because
glandular cells are associated with squamous and glandular precursor lesions and carcinoma.
Ref: (4)

109. Diagnostic Studies/ENT/Ophthalmology


Which of the following diagnostic studies would be most helpful in diagnosing a retropharyngeal abscess?
A. CBC
B. Neck CT scan
C. Rapid strep screen
D. Heterophile antibody
Explanations
(u) A. A CBC would be an important test, but it is not definitive for peritonsillar abscess.
(c) B. A neck CT scan would identify a peritonsillar abscess.
(u) C. A rapid strep screen may have been performed prior to placement on antibiotics, but does not provide a
definitive diagnosis for a peritonsillar abscess.
(u) D. A heterophile antibody is not indicated in the diagnosis of a retropharyngeal abscess.
Ref: (7)

110. Health Maintenance/Psychiatry/Behavioral Medicine


Which of the following is the leading cause of injury-related death in children between the ages of 1 and 15?
A. Drowning
B. Firearms
C. House fires
D. Motor vehicle injuries

43

Explanations
(u) A. Drowning is a frequent, but not the leading, cause of death in early childhood.
(u) B. Childhood death due to firearm related injuries is becoming more common, but it is not the leading cause.
(u) C. House fires are a common, but not the leading, cause of death in early childhood.
(c) D. Motor vehicle injuries are the leading cause of death in children.
Ref: (13)

111. Diagnosis/Pulmonology
A 45 year-old male presents with complaints of a chronic cough productive of mucopurulent sputum. The cough has
been present for the past 3 years, but he attributed it to a "smoker's cough". He has been coughing up a lot of sputum
lasting all winter long for the past 2 years. He denies any hemoptysis, weight loss or chest pain. Physical examination
reveals a moderately obese male in no acute respiratory distress. Lung fields reveal presence of scattered rhonchi
and wheezes. There is 1+ peripheral edema. Which of the following is the most likely diagnosis?
A. Lung cancer
B. Bronchiectasis
C. Chronic bronchitis
D. Interstitial lung disease
Explanations
(u) A. While the respiratory complaints of lung cancer are associated with the location and type of primary tumor,
anorexia and weight loss is seen in the majority of patients. Patients will also usually have a new cough or a change
in a chronic cough and may complain of hemoptysis and nonspecific chest pain.
(u) B. While bronchiectasis presents with a chronic cough productive of copious amounts of purulent sputum, these
patients most commonly also have associated complaints of hemoptysis, weight loss and pleuritic chest pain.
Examination of the lungs reveals persistent crackles at the bases.
(c) C. This patient most likely has chronic bronchitis which is defined as sputum production and cough for at least 3
months of the year for 2 consecutive years which is primarily caused by cigarette smoking.
(u) D. Interstitial lung disease is characterized by progressive exertional dyspnea and cough, however sputum
production is minimal and the examination of the lungs reveals fine, late inspiratory crackles at the bases in the
majority of patients.
Ref: (28)

112. Clinical Therapeutics/Neurology


A 72 year-old man with a long-standing history of diabetes mellitus, renal insufficiency, and hypertension presents to
the clinic complaining of burning and tingling pain in his feet. Which of the following medications would help control
pain in this patient?
A. Phenobarbital
B. Amitriptyline (Elavil)
C. Celecoxib (Celebrex)
D. Codeine
Explanations
(u) A. This is an antiepileptic medication that does not help diabetic neuropathy.
(c) B. Amitriptyline has been recommended for pain associated with diabetic neuropathy.
(h) C. Celecoxib is not indicated for diabetic neuropathy and should not be used if there is decreased kidney function.
(u) D. Treatment with codeine puts the patient at risk for dependence and this does not help the actual neuropathic
pain.
Ref: (28)

113. History & Physical/Gastrointestinal/Nutritional


A patient presents with abdominal pain in the right lower quadrant, examination reveals increased pain in the right
lower quadrant on deep palpation of the left lower quadrant. This commonly known as which of the following?
A. Psoas sign
B. Murphy's sign
C. Rovsing's sign
D. Obturator sign

44

Explanations
(u) A. Psoas sign is right lower quadrant pain with right leg extension.
(u) B. Murphy's sign is seen in liver and gallbladder disease in which the patient abruptly halts deep inspiration due to
discomfort as the examiners hand applies pressure to the right upper quadrant.
(c) C. A positive Rovsing's sign can be elicited in a patient with appendicitis when increased pain occurs in the right
lower quadrant upon palpation of the left lower quadrant.
(u) D. Obturator sign is right lower quadrant pain with internal rotation of the hip.
Ref: (3)

114. Clinical Intervention/Cardiology


Which of the following can optimize quality of life and is an definitive treatment for a patient with refractory heart
failure?
A. Ventricular assist device
B. Intra-aortic balloon counterpulsation
C. Cardiac transplantation
D. Partial resection of the left ventricle
Explanations
(u) A. Ventricular assist devices can help to provide temporary circulatory support to those awaiting transplantation.
(u) B. IABC is useful in cardiogenic shock, but does not improve quality of life.
(c) C. Cardiac transplantation is effective, with survival rates of 80-90% in 1 year, 60-70% over 5 years. It does
improve quality of life despite the immunosuppression medications.
(u) D. Partial resection of the left ventricle is still experimental and has not shown to have successful results.
Ref: (11)

115. Scientific Concepts/Orthopedics/Rheumatology


A football player complains of burning pain, numbness, and tingling extending from the left shoulder down into the
hand after he tackled a player. These symptoms resolved spontaneously in minutes. Following resolution of the
symptoms, he has full strength and normal sensation in the left arm. What is the most likely etiology of his
symptoms?
A. Acute muscle strain
B. Mild concussion
C. Stretching of nerve roots and brachial plexus
D. Thoracic outlet obstruction
Explanations
(u) A. Acute cervical muscle strain might result from the mechanism of injury described but symptoms would not
quickly resolve.
(u) B. Concussion involves generalized symptoms such as loss of consciousness but would not affect one limb
exclusively.
(c) C. Brachial plexus neurapraxia, commonly called "stinger" injuries, results from stretching of the cervical nerve
roots and brachial plexus by a mechanism such as that described in this question.
(u) D. Thoracic outlet syndrome is most commonly caused by cervical rib and is usually only symptomatic when the
arm is elevated.
Ref: (26)

116. Diagnostic Studies/Hematology


A positive osmotic fragility test is seen in which of the following?
A. G6PD deficiency
B. Sickle cell anemia
C. Hereditiary spherocytosis
D. Autoimmune hemolytic anemia

45

Explanations
(u) A. G6PD deficiency may result in episodic hemolysis in response to oxidant drugs or infections and is not
associated with a positive osmotic fragility test.
(u) B. Sickle cell anemia is an abnormal hemoglobinopathy leading to chronic hemolytic anemia and is confirmed with
hemoglobin electrophoresis. An osmotic fragility test would be negative.
(c) C. Hereditary spherocytosis will have a positive osmotic fragility test secondary to a defective red blood cell
membrane.
(u) D. Coombs' testing forms the basis for the diagnosis of autoimmune hemolytic disorders.
Ref: (28)

117. Diagnostic Studies/Endocrinology


Patients with primary adrenal insufficiency will have which of the following electrolyte abnormalities?
A. Hyperkalemia
B. Hypernatremia
C. Hypercalcemia
D. Hyperosmolality
Explanations
(c) A. Patients with primary adrenal insufficiency will not have the adrenal gland producing cortisol, aldosterone, and
the sex hormones. As a result of the lack of the mineralocorticoid aldosterone, the kidney will not save sodium and
will instead save potassium.
(u) B. Patients with primary adrenal failure will lack production of aldosterone and will not be able to save sodium.
(u) C. Patients with hyperparathyroidism will have an increase in the serum calcium level as a result of the
parathyroid gland retaining calcium.
(u) D. Increased serum osmolality is the result of diabetes insipidus which results in a dilute urine being produced
with resultant increase in the serum osmolality, as compared to the urine osmolality.
Ref: (28)

118. Health Maintenance/Obstetrics/Gynecology


What is the most common cause of secondary amenorrhea?
A. Pregnancy
B. Ovarian failure
C. Imperforate hymen
D. Hypothalamic amenorrhea

Explanations
(c) A. Pregnancy is the most common cause of amenorrhea and is essential to exclude by a serum or urine
pregnancy test.
(u) B. With ovarian failure, the ovarian follicles are resistant to stimulation. Ovarian failure can be caused by
chromosomal abnormalities, premature menopause, or a complication of chemotherapy.
(u) C. An imperforate hymen would prevent menstrual bleeding, this is a cause of primary amenorrhea.
(u) D. The pulsatile release of GnRH is disrupted and the anterior pituitary gland is not stimulated to release FSH and
LH. This can be caused by different etiologies including: weight loss, weight gain, excessive exercise, drug induced,
tumors, anorexia, and other chronic medical illnesses.
Ref: (4)

119. History & Physical/Cardiology


Which of the following physical exam findings suggests worsening or severe aortic stenosis?
A. An ejection click preceding the murmur
B. A split S2 with respiration variation
C. Grade 2/6 systolic murmur radiating to the carotids
D. Palpable thrill over the right second intercostal space

46

Explanations
(u) A. This finding is typical in mild to moderate AS.
(u) B. This can be a normal finding.
(u) C. This is the typical murmur for aortic stenosis, but does not suggest the degree of severity by the grade.
(c) D. A palpable thrill or LV heave with associated murmur suggests severe AS.
Ref: (28)

120. Clinical Therapeutics/Urology/Renal


A 60 year-old male presents with difficulty voiding and having to get up twice a night to urinate. On physical
examination, the prostate is firm, smooth, and enlarged. Prostate specific antigen level is normal. Which of the
following is the best treatment option for this patient?
A. Leuprolide (Lupron)
B. Terazosin (Hytrin)
C. Ciprofloxacin (Cipro)
D. Naproxen (Naprosyn)
Explanations
(u) A. Leuprolide is a luteinizing hormone-releasing hormone agonists used in the treatment of prostate cancer.
(c) B. Terazosin is an alpha-blocker and used to treat benign prostatic hyperplasia.
(u) C. Ciprofloxacin is an antibiotic used to treat acute prostatitis not benign prostatic hypertrophy.
(u) D. Naproxen is an anti-inflammatory agent and not used in the treatment of benign prostatic hypertrophy.
Ref: (28)

121. Clinical Intervention/ENT/Ophthalmology


In patents with diabetic retinopathy, what clinical intervention is most successful in preserving vision?
A. Panretinal laser photocoagulation
B. Iridectomy
C. Radial keratotomy
D. Vitrectomy
Explanations
(c) A. Panretinal laser photocoagulation is indicated for preservation of vision in patients with diabetic retinopathy.
(h) B. Iridectomy is of no value in preserving the retina and iridectomy is harmful in this situation due to the trauma it
causes to the eye.
(h) C. Radial keratotomy is indicated to correct myopia. This surgery destroys normal eye architecture and has no
benefit in diabetic retinopathy.
(h) D. Vitrectomy is indicated for treatment of retinal tears and not to preserve an intact retina.
Ref: (7)

122. Diagnostic Studies/Neurology


A 53 year-old female with sudden onset "of the worst headache she has ever had" presents to the emergency
department. She has a history of migraines but states that the current headache is not like her usual headaches.
Results of her physical examination are unremarkable. Which of the following is the next best step in the evaluation of
this patient?
A. Angiogram
B. CT scan
C. Transcranial Doppler
D. MRI
Explanations
(u) A. Angiogram is necessary to define details of aneurysm and anatomic configuration, but not as an initial
diagnostic study.
(c) B. This patient's history is highly suggestive of subarachnoid hemorrhage. CT is best to screen for intracranial
hemorrhage. It is faster than MRI and more sensitive in the first 24 hours.
(u) C. Transcranial Doppler can detect cerebral artery vasospasm but cannot detect aneurysm.
(u) D. MRI is not as sensitive for an acute bleed, but is appropriate for old bleeds.
Ref: (28)

47

123. Diagnosis/Gastrointestinal/Nutritional
A 45 year-old male presents with a long history of ulcerative colitis and recent progressive complaints of right upper
quadrant pain, weight loss, fever and most recently, a rapid onset of jaundice with pruritus. Labs revealed elevated
bilirubin and alkaline phosphatase. Viral serologies were negative. An endoscopic cholangiogram showed areas of
stenosis and dilation throughout the bile duct system. What is the most likely diagnosis?
A. Choledocholithiasis
B. Hepatic carcinoma
C. Portal hypertension
D. Primary sclerosing cholangitis
Explanations
(u) A. Choledocholithiasis can most certainly cause elevated bilirubin and other liver function tests when obstruction
occurs. It also occurs more often in persons with sclerosed or narrowed bile ducts however it does not generally carry
a poor prognosis and can be generally recognized by its typical symptoms.
(u) B. Hepatic carcinoma does not cause areas of varied stenosis and dilation in the biliary tree.
(u) C. Portal hypertension may present with jaundice but generally has significant ascites as well that helps to
distinguish this disorder. It also does not result in the bile duct pattern mentioned.
(c) D. Primary sclerosing cholangitis (PSC) results in diffuse intra- and extrahepatic duct sclerosing with dilatation
proximal to these areas.
Ref: (7)

124. Clinical Therapeutics/Orthopedics/Rheumatology Topic: 6a Author: Jonathan Bowser


A 43 year-old female presents with a two year history of frequent episodes of pain and morning stiffness in both
hands and wrists. She experiences some symptomatic relief with ibuprofen, but feels that the episodes are becoming
more frequent and severe. On examination, you observe joint swelling of several MCP joints on both hands. X-ray of
the hands shows joint space narrowing of the MCP joints. In addition to NSAIDs, what is the most appropriate firstline long-term medication to treat this patient?
A. Prednisone (Deltasone)
B. Infliximab (Enbrel)
C. Methotrexate (Rheumatrex)
D. Sulfasalazine (Azulfidine)
Explanations
(u) A. Prednisone can produce impressive clinical improvement in rheumatoid arthritis but the side effects associated
with their long-term use limit their utility.
(u) B. Tumor necrosis factor inhibitors, such as infliximab, are very effective disease modifying antirheumatic drugs
(DMARDS). This class of medication is inappropriate as a first-line medication due to very high cost.
(c) C. Methotrexate is the standard first-line medication in the treatment of rheumatoid arthritis. This DMARD is
generally effective, well-tolerated, and affordable.
(u) D. Sulfasalazine is inexpensive but associated with potentially serious hematologic side effects and is considered
a second-line DMARD.
Ref: (28)

125. Scientific Concepts/Cardiology


Perfusion of the coronary arteries occurs primarily during which of the following?
A. Systole
B. Diastole
C. Afterload
D. Preload
Explanations
(u) A. See B for explanation.
(c) B. Coronary artery perfusion occurs primarily during diastole.
(u) C. See B for explanation.
(u) D. See B for explanation.
Ref: (11)

48

126. History & Physical/Pulmonology


Which of the following is a common presenting clinical manifestation of a patient with interstitial lung disease?
A. Early inspiratory crackles
B. Progressive dyspnea on exertion
C. Productive cough with copious sputum
D. Decreased breath sounds with hyperresonant percussion
Explanations
(u) A. Late, not early, inspiratory crackles are associated with interstitial lung disease.
(c) B. Patients with interstitial lung disease commonly present with progressive dyspnea on exertion and a cough with
minimal sputum production.
(u) C. A productive cough of copious amounts of sputum is most typical of a patient with chronic bronchitis.
(u) D. Physical examination findings of decreased breath sounds with hyperresonant percussion is consistent with a
diagnosis of chronic obstructive lung disease.
Ref: (28)

127. Clinical Intervention/Psychiatry/Behavioral Medicine


A patient is started on a new antipsychotic medication for his disorder. Three days later he develops altered
consciousness, lead-pipe rigidity, diaphoresis and catatonia. Vital signs reveal respiratory rate of 20, temperature of
105.6 degrees F, and pulse oximetry of 95% room air. Which of the following would be the most appropriate initial
intervention in this patient?
A. Immediate oral SSRI's and Benadryl
B. Supportive care with fluids and antipyretics
C. IV antibiotics and naloxone
D. Intubation and mechanical ventilation
Explanations
(u) A. See B for explanation.
(c) B. Neuroleptic malignant syndrome is characterized by extrapyramidal signs, blood pressure changes, altered
consciousness, hyperpyrexia, muscle rigidity, dysarthria, cardiovascular instability, fever, pulmonary congestion and
diaphoresis. Controlling fever and fluid support are the best initial management. With a normal pulse oximetry
mechanical ventilation is not indicated.
(u) C. See B for explanation.
(u) D. See B for explanation.
Ref: (28)

128. Diagnosis/ENT/Ophthalmology
A patient presents with a nontender, painless, nodule involving a meibomian gland. Which of the following is the most
likely diagnosis?
A. Chalazion
B. Dacryocystitis
C. Entropion
D. Hordeolum
Explanations
(c) A. Chalazion is characterized by a hard, nontender swelling on the upper or lower lid with redness and swelling of
the adjacent conjunctiva and is due to granulomatous inflammation of a meibomian gland.
(u) B. Dacryocystitis is an infection of the lacrimal sac due to obstruction of the nasolacrimal system.
(u) C. Entropion is an outward turning of the lower lid.
(u) D. A hordeolum is a bacterial inflammation of the base of the eyelash.
Ref: (3)

49

129. Health Maintenance/Gastrointestinal/Nutritional


Which of the following is the most effective prophylaxis for traveler's diarrhea?
A. Metronidazole
B. Ciprofloxacin
C. Tetracycline
D. Ampicillin
Explanations
(u) A. Ampicillin, tetracycline, and metronidazole do not cover the common causes of traveler's diarrhea.
(c) B. Options for prophylaxis of traveler's diarrhea include norfloxacin, ciprofloxacin, ofloxacin, or
trimethoprim/sulfamethoxazole.
(u) C. See A for explanation.
(u) D. See A for explanation.
Ref: (28)

130. Diagnostic Studies/Obstetrics/Gynecology


Which of the following tests is the most specific for the diagnosis of syphilis?
A. Rapid plasma reagin (RPR)
B. Weil-Felix agglutination test
C. Venereal Disease Research Laboratory (VDRL)
D. Fluorescent treponemal antibody absorption (FTA-ABS)
Explanations
(u) A. The RPR is a non-specific test. False positives are common.
(u) B. The Weil-Felix agglutination test is used for rickettsial infections not syphilis.
(u) C. The VDRL if positive must by confirmed with an additional testing because of a large number of false positives
including bacterial and viral infections, pregnancy, chronic liver disease, connective tissue disorders.
(c) D. The FTA-ABS and the MTA-TP are specific treponemal tests used for the confirmation of syphilis.
Ref: (4)
131. Scientific Concepts/Dermatology
Which of the following types of infection is most likely to benefit from hyperbaric oxygen therapy?
A. Clostridial infection
B. Group A beta hemolytic streptococcus
C. Serratia marcescens
D. Methicillin resistant Staphylococcus aureus
Explanations
(c) A. Patients with clostridium myonecrosis (gas gangrene) will benefit from hyperbaric oxygen therapy, penicillin
therapy, and radical surgical excision.
(u) B. Patients with group A beta hemolytic strep, staph aureus, and community acquired methicillin resistant staph
aureus will benefit from antibiotic therapy but these organisms are not anaerobic so hyperbaric oxygen therapy is not
useful.
(u) C. See B for explanation.
(u) D. See B for explanation.
Ref: (28)
132. History & Physical/Urology/Renal
Which of the following is most frequently associated with renal cell carcinoma?
A. Hematuria
B. Inguinal pain
C. Hypocalcemia
D. Fever
Explanations
(c) A. Gross or microscopic hematuria, flank pain, or mass is common in renal cell carcinoma.
(u) B. See A for explanation.
(u) C. Hypercalcemia, not hypocalcemia, may be noted in patients with renal cell carcinoma.
(u) D. Fever is uncommon in renal cell carcinoma and typically only noted with advanced disease.
Ref: (28)

50

133. Clinical Therapeutics/Cardiology


What is the mechanism of action for aspirin?
A. Inhibits platelet aggregation by blocking cyclooxygenase-1 activity
B. Exerts antiplatelet effects by blocking the platelet membrane adenosine diphosphate receptors
C. Inhibits the platelet membrane glycoprotein IIb/IIIa receptor
D. Converts plasminogen to the natural fibrinolytic agent plasmin
Explanations
(c) A. Aspirin inhibits platelet aggregation by blocking cyclooxygenase-1 activity.
(u) B. This the mechanism of action for ADP antagonists such as clopidogrel or ticlopidine.
(u) C. This is the mechanism of action for glycoprotein IIb/IIIa receptors such as abciximab (ReoPro).
(u) D. This is the mechanism of action for tissue plasminogen activators (tPA).
Ref: (11)

134. Clinical Intervention/Pulmonology


A 68 year-old male with history of COPD is brought to the emergency department following a motor vehicle collision.
On physical examination there is evidence of head trauma. The left side of the chest wall appears to move inward
with inspiration and outward with expiration. A chest x-ray reveals multiple rib fractures on the left. Which of the
following is the most appropriate intervention?
A. Surgical fixation of the fractured ribs
B. Application of elastic binders and adhesive tape
C. Endotracheal intubation and mechanical ventilation
D. Chest physiotherapy that encourages frequent coughing
Explanations
(u) A. Surgical fixation of flail chest is less reliable than positive-pressure ventilation and is performed only rarely in
the United States.
(u) B. While application of elastic binders and adhesive tape was historically utilized to stabile the chest, this
intervention has been found to decrease chest expansion and worsen lung atelectasis.
(c) C. Indications for early endotracheal intubation and mechanical ventilation in treatment of flail chest include
patients that are over the age of 65, have comorbid lung disease and associated severe head trauma. Other
indications include shock, three or more associated injuries and fracture of eight or more ribs.
(u) D. Conservative treatment for mild to moderate flail chest includes analgesic relief of pain, chest physiotherapy
that encourages frequent coughing and restriction of fluids to prevent fluid overload, however this patient needs early
ventilatory support.
Ref: (29)

135. Diagnosis/Orthopedics/Rheumatology
A 13 year-old girl reports two weeks of worsening right knee pain with no history of antecendent injury or recent
trauma. She reports frequent episodes of nighttime awakening with knee pain in the past two weeks. Examination of
the knee reveals edema and a tender mass over the anterior proximal right tibia. Her knee exam is otherwise within
normal limits. Radiographs of the right knee show a lytic mass with a multi-laminated periosteal reaction involving the
proximal anterior tibia. What is the most likely diagnosis?
A. Ewing sarcoma
B. Osteochondroma
C. Multiple myeloma
D. Osteoid osteoma
Explanations
(c) A. The distinctive feature of Ewing sarcoma is the radiographic appearance of a periosteal "onion skin" reaction.
(u) B. This benign tumor typically presents as a painless mass and appears in plain film radiographs as a stalk or
broad-based projection from the surface of the bone.
(u) C. The classic radiographic appearance of multiple myeloma is a lytic lesion but this is a condition that is seen in a
much older population and is more likely to present with back pain.
(u) D. Although the presentation may be similar to Ewing's sarcoma, the radiographs in osteoid osteoma typically
show a round lucency surrounded by sclerotic bone.
Ref: (5)

51

136. Diagnostic Studies/ENT/Ophthalmology


In a patient with amaurosis fugax what is the most appropriate initial diagnostic study?
A. Ophthalmoscopy
B. Schiotz tonometry
C. MR angiography
D. Carotid ultrasound
Explanations
(u) A. Ophthalmoscopy provides visualization of retina but does not help elucidate the source of the plaque.
(u) B. Schiotz tonometry is used to measure intraocular pressure.
(u) C. MR angiography is a useful test to identify retinal vascular anatomy but is not the first choice in the search for
causes of amaurosis fugax.
(c) D. The most common cause of amaurosis fugax is an atherosclerotic plaque in the carotid artery which can be
identified with ultrasound.
Ref: (28)

137. Health Maintenance/Hematology


An adult patient who has undergone a splenectomy secondary to idiopathic thrombocytopenic purpura should receive
which prophylactic agent?
A. Bicillin 1.2 million units IM every month
B. Pneumococcal vaccine IM one time only
C. Gamma globulin IM every month
D. Hepatitis B vaccine IM series
Explanations
(u) A. Patients who are being treated for rheumatic fever may receive Bicillin 1.2 million units every month for Group
A beta-streptococcal prophylaxis.
(c) B. Pneumococcal vaccine should routinely be given to all patients who undergo splenectomy for prophylaxis to
Streptococcal pneumoniae.
(u) C. Gamma globulin is routinely given to patients for prophylaxis to hepatitis exposure.
(u) D. Hepatitis B vaccine has no specific indication in patients after splenectomy.
Ref: (28)

138. History & Physical/Cardiology


A 56 year-old male with a 30 pack-year smoking history presents with substernal chest pain. The pain is described as
a pressure that radiates to his jaw. The pain has lasted consistently for 30 minutes with variable relief. His current
medications include atorvastatin (Lipitor) and glyburide (Micronase). Which of the following aggravating or relieving
factors about the pain would support the diagnosis?
A. Precipitated by exercise and relieved with rest
B. Unrelieved by nitroglycerin
C. Aggravated by deep breaths
D. Relieved with food
Explanations
(u) A. Precipitated by exercise and relief with rest is consistent with angina.
(c) B. Patient is having a myocardial infarction which is unrelieved by rest or nitroglycerin.
(u) C. Aggravated by deep breaths suggests pericarditis.
(u) D. Relieved with food suggests peptic ulcer disease.
Ref: (11)

52

139. Clinical Therapeutics/Gastrointestinal/Nutritional


A 30 year-old female presents with a one to two year history of daily bouts of mid-abdominal crampy pain and
bloating that are briefly relieved with defecation. Loose to watery bowel movements occur four to five times daily. She
denies any nocturnal symptoms. Previous laboratory studies, abdominal CT scans and a colonoscopy have all been
normal. Attempted dietary changes, exercise and probiotics have provided no relief. What is the next step in
treatment for this patient?
A. Nortriptyline (Pamelor)
B. Omeprazole (Prilosec)
C. Polyethylene glycol (Miralax)
D. Promethazine (Phenergan)
Explanations
(c) A. Nortriptyline, and other tricyclic antidepressants (TCAs), is an appropriate choice for someone with diarrhea
predominant irritable bowel syndrome due to their numerous mechanisms of action.
(u) B. Omeprazole, a proton pump inhibitor, works on parietal cells in the stomach lining to reduce production of
hydrochloric acid. Though beneficial for peptic ulcer and reflux disease it would provide no relief of symptoms in a
patient with IBS.
(u) C. Polyethylene glycol is commonly used in constipation predominant IBS but would probably worsen this patient's
symptoms.
(u) D. The anticholinergic properties of the promethazine might be helpful even though its somnolent effects would be
significant and would not be superior to nortriptyline. Promethazine's strength is in nausea control and less often its
antihistamine affects.
Ref: (28)
140. Diagnosis/Pulmonology
A 28 year-old man presents to the emergency department complaining of sudden onset of shortness of breath
associated with sharp right-sided chest pain increased with breathing. On physical examination, respirations are 20
per minute and blood pressure is 120/76 mm Hg. Auscultation of the chest reveals absent breath sounds over the
right apex with normal heart sounds. Percussion of the right apex is noted to be hyperresonant. Which of the
following is the most likely diagnosis?
A. Hemothorax
B. Pneumothorax
C. Pulmonary embolus
D. Foreign body aspiration
Explanations
(u) A. A hemothorax usually results from trauma. While breath sounds would be absent over the involved area, the
percussion note would be dull, not hyperresonant.
(c) B. This patient most likely has a spontaneous pneumothorax which is supported by the presenting symptoms of
sudden onset of dyspnea and pleuritic chest pain as well as the physical exam findings of absent breath sounds and
hyperresonance to percussion.
(u) C. While a pulmonary embolism most commonly presents with dyspnea and pain on inspiration, the physical
examination is often unremarkable and would not reveal the findings of absent breath sounds and hyperresonance to
percussion.
(u) D. Foreign body aspiration is most common in children and the elderly. Aspiration of a food bolus is the most
common cause in adults which leads to a history of a choking episode followed by dysphagia, wheezing and
coughing. Physical examination findings are dependent on the location of the obstruction.
Ref: (28)

141. Clinical Intervention/Endocrinology


In the treatment of alcohol-related hypoglycemia, thiamine is administered before glucose to prevent which of the
following?
A. Acute renal failure
B. Wernicke's encephalopathy
C. Korsakoff psychosis
D. Mesenteric thrombosis

53

Explanations
(u) A. Acute renal failure is prevented by adequate hydration allowing appropriate blood flow to the kidney. It is
prevented by hydration with volume expanders rather than by thiamine and/or glucose.
(c) B. Alcoholics are typically deficient in thiamine. If glucose alone is given to treat hypoglycemia, Wernicke's
encephalopathy can be precipitated since thiamine is not available for nutritional purposes when glucose is replaced.
Therefore, thiamine is always given prior to glucose until a satisfactory diet can be given in order to prevent
Wernicke's encephalopathy.
(u) C. Korsakoff psychosis is a sequelae of Wernicke's encephalopathy so if thiamine is used to prevent Wernicke's
encephalopathy, its sequelae, Korsakoff's psychosis will also be prevented.
(u) D. Mesenteric thrombosis is mostly a complication of atherosclerosis or embolism to the mesenteric artery. It is
more common in low flow states such as hypotension and can somewhat be prevented by ensuring adequate
hydration. It is not dependent on thiamine or glucose, rather it just needs volume expanders.
Ref: (28)

142. History & Physical/Infectious Diseases


You suspect that a 13 year-old patient has bacterial conjunctivitis. What physical examination finding would you
expect?
A. Crusting and matting of the eyelids with discharge and diffuse injection of the conjunctival surface
B. Clear watery stringy discharge with conjunctival hyeremia and edema
C. Circumcorneal injection of the conjunctiva
D. Painless injection of the subconjunctival sclera bilaterally
Explanations
(c) A. Crusting and matting of the eyelids with discharge and diffuse injection of the conjunctival surface
is a common presentation of a bacterial conjunctivitis.
(u) B. Clear watery stringy discharge with conjunctival hyeremia and edema is a common presentation of an allergic
conjunctivitis.
(u) C. Circumcorneal injection of the conjunctiva is a common presentation of iridocyclitis.
(u) D. Painless injection of the subconjunctival sclera bilaterally is a common presentation of episcleritis.
Ref: (28)

143. Diagnostic Studies/Psychiatry/Behavioral Medicine


A 52 year-old female complains of bouts of anxiety and depression, the latter very deep but short lived (<24 hours).
She states "I often feel like I am going to jump out of my skin". Her periods have become less frequent over the past
6 months. She denies suicidal ideations. She is not sleeping through the night. Which of the following the most
appropriate next step in this patient?
A. Check a follicle-stimulating hormone level
B. Start a tricyclic antidepressant
C. Refer her to a psychiatrist
D. Endometrial biopsy
Explanations
(c) A. An FSH level should be assessed in this patient to evaluate for menopause as a cause for her psychiatric
symptoms.
(u) B. See A for explanation.
(u) C. See A for explanation.
(u) D. Endometrial biopsy is indicated in a perimenopausal female with menorrhagia.
Ref: (28)

144. Scientific Concepts/Obstetrics/Gynecology


Maternal blood pressure normally decreases the most during what period of pregnancy?
A. First trimester of pregnancy
B. Second trimester of pregnancy
C. Third trimester of pregnancy
D. During labor and delivery

54

Explanations
(u) A. Blood pressures during the first trimester will be similar to the non-pregnant state.
(c) B. Diastolic blood pressure and the mean arterial pressure reach their nadir at 16-20 weeks of gestation.
(u) C. Blood pressures during the third trimester will be similar to the non-pregnant state.
(u) D. Blood pressure during labor and delivery often are variable. Blood pressure often rises with pain and
apprehension. Blood pressure often decreases with the use of narcotics, epidurals, and significant blood loss.
Ref: (4)

145. Diagnosis/Neurology
A 32 year-old female presents with a seven month history of recurrent, brief episodes of weakness and tingling in the
extremities, diplopia, and vertigo. Which of the following is the most likely diagnosis?
A. Guillain-Barre syndrome
B. Myasthenia gravis
C. Multiple sclerosis
D. Amyotrophic lateral sclerosis
Explanations
(u) A. Guillain-Barre syndrome typically presents with progressive weakness that starts peripherally and proceeds
centrally over a brief period of time.
(u) B. Myasthenia gravis presents with weakness and fatigue in the upper limbs, cranial, and/or trunk musculature.
Blurry vision and diplopia are common visual complaints and dysphagia is common.
(c) C. Multiple sclerosis is most frequently seen in patients in their twenties and presents with episodes of weakness,
paresthesias, and diplopia.
(u) D. Amyotrophic lateral sclerosis presents with progressive weakness, fasciculations, and loss of muscle mass.
Ref: (28)

146. Clinical Therapeutics/Pulmonology


An elderly nursing home resident is admitted with methicillin-resistant Staphylococcus aureus pneumonia. Which of
the following is the most appropriate treatment to initiate?
A. Nafcillin
B. Vancomycin
C. Clindamycin
D. Doxycycline
Explanations
(u) A. Nafcillin is classified as a penicillinase-resistant penicillin, however would not be effective against methicillinresistant strains of Staphylococcus aureus.
(c) B. Vancomycin with or without the addition of gentamicin or rifampin and linezolid is the treatment of choice for
methicillin-resistant Staphylococcus aureus.
(u) C. Clindamycin is primarily utilized in treatment of severe anaerobic infections, but is not indicated for the
treatment of methicillin-resistant Staphylococcus aureus.
(u) D. Doxycycline is a tetracycline and is only utilized to treat respiratory infections that are susceptible to
tetracycline, such as Mycoplasma or Chlamydial pneumonias.
Ref: (28)

147. Health Maintenance/Cardiology


A 26 year-old female presents to clinic with a left arm that is swollen and non-tender with bluish discoloration along
the upper arm and forearm. She is status post pacemaker insertion in the left upper chest for third degree heart block,
one week ago. Pulses are present and the arm is warm, but not red. The pacemaker incision is healing well despite a
hematoma and tenderness at the site. Which of the following statements would be appropriate patient education
about this condition?
A. Reassurance that the discoloration is an expected finding.
B. Apply cold compresses to the site of the hematoma.
C. Elevation of the involved extremity will increase the swelling.
D. Aspirin should be taken to help manage pain.

55

Explanations
(c) A. This is indicative of migratory ecchymosis and expected after insertion of a pacemaker.
(u) B. Warm compresses will help to decrease the hematoma.
(u) C. Elevation will help to decrease the swelling.
(u) D. ASA is not the pain medicine of choice in a patient with a hematoma due to its non-clotting properties.
Ref: (11)

148. Clinical Intervention/ENT/Ophthalmology


A 20 year-old presents 30 minutes after being struck by a hockey puck in the mouth. On physical examination a
central incisor is missing from its socket. The patient has the tooth wrapped in tissue paper and the root appears
intact. Which of the following is the most appropriate next step in the treatment of this patient?
A. Administration of IM penicillin
B. No treatment is warranted
C. Place tooth in saline and refer to plastic surgery for reimplantation
D. Immediately reimplant the tooth and refer to an oral surgeon
Explanations
(u) A. Penicillin is not indicated for treatment of an avulsed tooth.
(h) B. Reimplantation is warranted as this is a permanent tooth with root intact. Primary teeth are never reimplanted.
(u) C. See D for explanation.
(c) D. Avulsed permanent teeth should be cleansed, transported in Hanks solution or saline and reimplanted by an
oral surgeon within one hour.
Ref: (29)

149. History & Physical/Endocrinology


Which of the following is the most common presentation for an elderly female patient with primary
hyperparathyroidism?
A. Abdominal pain
B. Renal lithiasis
C. Acute pancreatitis
D. Asymptomatic
Explanations
(u) A. Patients with primary hyperparathyroidism may have increased calcium levels which result in abdominal pain
and constipation but this is not the most common manifestation.
(u) B. Patients with primary hyperparathyroidism may have renal lithiasis as a result of the hypercalcemia but it is not
the most common presentation.
(u) C. Patients with acute pancreatitis may have hypocalcemia as a result of the calcium soaps that are formed in
order to help to process the food that is eaten. It is not related to hyperparathyroidism; rather it is most commonly
caused by gallstones and alcoholism.
(c) D. Patients with primary hyperparathyroidism are most commonly found to have this disease by an incidental
finding of hypercalcemia on routine laboratory testing as a result of screening. Up to 0.1% of the adult population has
this condition which is most commonly seen in females over age 50.
Ref: (28)

150. Diagnosis/Obstetrics/Gynecology
A 35 year-old G2P1001 female presents to the office at 11 weeks gestation with vaginal bleeding, mid-lower
abdominal cramping, and bilateral lower pelvic discomfort. On examination, bright red blood is seen coming from the
cervical os. The cervix is closed. The uterus is 9-11 weeks in size by palpation. Her blood pressure is 120/70 mmHg
and her pulse rate is 96. What is the patient's most likely diagnosis?
A. Inevitable abortion
B. Incomplete abortion
C. Threatened abortion
D. Missed abortion

56

Explanations
(u) A. Inevitable abortion is when the patient presents during the first 20 weeks of pregnancy with bleeding and
crampy abdominal pain, also associated with a dilated cervix or a gush of fluid without the passage of the products of
conception.
(u) B. Incomplete abortion occurs when part of the products of conception are passed from the cervix, the cervix will
also be dilated.
(c) C. If bleeding occurs in the first 20 weeks of pregnancy and the cervix is closed, threatened abortion is the
diagnosis.
(u) D. A missed abortion occurs when a patient presents with smaller gestational size by examination than by dates
and no fetal heart tones.
Ref: (4)

151. Diagnostic Studies/Dermatology


A 23 year-old African American female presents with concern regarding "lightening" of the skin in her knees, face,
clavicles, and wrists. These areas appear to be increasing in size. She was recently diagnosed with pernicious
anemia. In order to confirm her diagnosis, which of the following tests is indicated?

A. Wood's light
B. KOH prep
C. Skin biopsy
D. Gram stain
Explanations
(u) A. Wood's light is primarily used to evaluate macules but there are no pathognomonic fluorescent patterns that
would confirm the diagnosis of vitiligo.
(u) B. KOH prep would be indicated to search for tinea versicolor by showing a spaghetti and meatball appearance
but this patient's condition is consistent with vitiligo.
(c) C. Skin biopsy in a patient with vitiligo will show complete absence of melanocytes and will confirm the diagnosis.
(u) D. Gram stain is helpful in diseases secondary to a bacterial etiology not vitiligo.
Ref: (10)

152. Clinical Therapeutics/Orthopedics/Rheumatology


A 52 year-old male presents with a severely painful, swollen right great toe. He denies recent trauma, but reports
several similar episodes of toe pain and swelling over the past two to three years. He has a history of alcohol abuse
and hypertension, for which he "takes medication" of unknown type. Examination reveals bright erythema and edema
associated with the right first MCP joint. Which of the patient's antihypertensive medications may be contributing to
this condition?
A. Diltiazem (Cardizem)
B. Hydrochlorothiazide (HCTZ)
C. Lisinopril (Prinivil)
D. Metoprolol (Lopressor)
Explanations
(u) A. Calcium-channel blockers, such as diltiazem, are not associated with increased risk of hyperuricemia and gout.
(c) B. Thiazide diuretics, such as hydrochlorothiazide, are associated with increased risk of hyperuricemia and gout.
(u) C. ACE inhibitors, such as lisinopril, are not associated with increased risk of hyperuricemia and gout.
(u) D. Beta-blockers, such as metoprolol, are not associated with increased risk of hyperuricemia and gout.
Ref: (11)

153. Clinical Intervention/Cardiology


A 66 year-old female with a history of coronary artery disease presents with a new onset of dizziness and fatigue for
two weeks. She recalls nearly passing out on one occasion. Examination is unremarkable except for bradycardia.
Electrocardiogram (ECG) reveals a heart rate of 50 with a normal PR interval followed by a normal QRS. There are
several non-conducting P waves and no lengthening of the PR interval. Which of the following interventions is the
therapy of choice?

57

A. Permanent pacemaker
B. Radio-frequency ablation
C. Maze procedure
D. Automatic Implantable Cardioverter Defibrillator
Explanations
(c) A. This patient has symptomatic second degree type II heart block and requires a pacemaker.
(u) B. Radio-frequency ablation is useful for supraventricular tachyarrhythmias.
(u) C. Maze procedure is a surgical procedure for decreasing the incidence of atrial fibrillation by creating cuts
simulating a pathway in the atriums.
(u) D. Automatic Implantable Cardioverter Defibrillator (AICD) is used in patients at risk for sudden death.
Ref: (11)

154. History & Physical/Neurology


A mother brings in her 4 year-old son with complaints that he falls frequently and he "stands funny". The mother also
notes that he has lost the ability to easily stand from a seated position. She reports that he met developmental
milestones as an infant. Which of the following is the most likely cause?
A. Developmental hip dysplasia
B. Myasthenia gravis
C. Cerebral palsy
D. Muscular dystrophy
Explanations
(u) A. Signs and symptoms of developmental hip dysplasia would have been present more at birth and when
beginning to crawl and walk.
(u) B. Myasthenia gravis (MG) is less likely due to age and MG typically affects the ocular, pharyngeal and respiratory
muscles.
(u) C. Cerebral palsy (CP) typically presents with ataxia, spasticity or tightness, and uncoordination. Patients with CP
typically do not meet developmental milestones.
(c) D. Muscular dystrophy age of onset is by age 5, and begins in the pelvic girdle.
Ref: (13)

155. Clinical Therapeutics/Pulmonology


Which of the following classes of medications is most likely to cause a persistent cough?
A. Tricyclic antidepressants
B. Calcium channel blockers
C. Beta-adrenoceptor blocking agents
D. Angiotensin converting enzyme inhibitors
Explanations
(u) A. Adverse effects of tricyclic antidepressants are due to blockage of acetylcholine receptors causing such
symptoms as blurred vision, dry mouth, urinary retention and constipation.
(u) B. Side effects are uncommon with calcium channel blockers, but include constipation, dizziness, headache and a
feeling of fatigue.
(u) C. Common adverse effects of beta-adrenoceptor blocking agents include bradycardia and central nervous
system symptoms, such as fatigue or insomnia.
(c) D. Common side effects of angiotensin converting enzyme inhibitors (ACE inhibitors) include a dry cough.
Ref: (21)

156. Scientific Concepts/ENT/Ophthalmology


Which of the following is the most likely organism in a 2 year-old child with acute otitis media?
A. Staphylococcus aureus
B. Moraxella catarrhalis
C. Pseudomonas aeruginosa
D. Streptococcus pneumoniae

58

Explanations
(u) A. See D for explanation.
(u) B. See D for explanation.
(u) C. See D for explanation.
(c) D. The most common pathogens in children with acute otitis media are Streptococcus pneumoniae, Haemophilus
influenzae, and Streptococcus pyogenes.
Ref: (28)

157. Diagnosis/Urology/Renal
A 15 year-old male patient presents with oliguria, hematuria, proteinuria, and fatigue following streptococcal
pharyngitis 2 weeks ago. Which of the following is the most likely diagnosis?
A. Acute pyelonephritis
B. Acute glomerulonephritis
C. Systemic lupus erythematosus
D. Initial onset of type 1 diabetes mellitus
Explanations
(u) A. Acute pyelonephritis presents with fever, flank pain, urgency, and pyuria.
(c) B. Acute glomerulonephritis is a complication that can follow a streptococcal infection after 1 to 3 weeks.
(u) C. Systemic lupus erythematosus is not a complication of streptococcal infections, but it can cause rapid
progressive glomerulonephritis.
(u) D. Diabetes mellitus presents with polyuria, not oliguria or hematuria.
Ref: (28)

158. Diagnostic Studies/Cardiology


Pulmonary capillary wedge pressure indirectly measures which of the following?
A. Right ventricular end-diastolic pressure
B. Right atrial filling pressure
C. Left ventricular systolic pressure
D. Left atrial filling pressure
Explanations
(u) A. See D for explanation.
(u) B. See D for explanation.
(u) C. See D for explanation.
(c) D. Pulmonary capillary wedge pressure indirectly measures left atrial filling pressures.
Ref: (30)

159. Clinical Therapeutics/Endocrinology


Which of the following is a known complication to prescribing excessive doses of levothyroxine (Synthroid) for
patients with hypothyroidism?
A. Primary hyperparathyroidism
B. Papillary carcinoma of the thyroid gland
C. Osteoporosis
D. Diabetes Insipidus
Explanations
(u) A. Primary hyperparathyroidism is not related to excessive doses of levothyroxine.
(u) B. Papillary thyroid cancer is the most common thyroid cancer. It is most closely associated with radiation
exposure. Levothyroxine is typically given after surgical treatment in order to suppress any thyroid activity in any
remaining thyroid tissue.
(c) C. Osteoporosis may result from overaggressive therapy with levothyroxine (Synthroid) because of the increased
bone turnover that results from increased basal metabolic rate.
(u) D. Diabetes insipidus results from lack of production of ADH in the brain or from the kidney not responding to ADH
in the circulation. It is not related to levothyroxine therapy.
Ref: (28)

59

160. History & Physical/Psychiatry/Behavioral Medicine


A patient tells you that he is receiving special messages from the TV every night at 7:00 pm. This is an example of
which of the following?
A. Delusions
B. Ideas of reference
C. Paranoia
D. Suicidal ideation
Explanations
(u) A. Delusions are false fixed beliefs.
(c) B. Ideas of reference are fixed beliefs that people are referring to you and about you through media.
(u) C. He is frankly telling you this and not displaying anything that makes you think he is paranoid.
(u) D. He is not stating he is having thoughts of harming or killing himself.
Ref: (14)

161. Clinical Intervention/Orthopedics/Rheumatology


A 22 year-old male presents several hours after sustaining a hand injury when he punched a wall. X-rays of the hand
demonstrate fracture of the fifth metacarpal neck with 65 degrees dorsal angulation and a claw hand. What is the
most appropriate intervention?
A. Antibiotic treatment and ulnar gutter splint immobilization
B. Closed reduction and ulnar gutter splint immobilization
C. Open reduction and ulnar gutter splint immobilization
D. Ulnar gutter splint immobilization only
Explanations
(u) A. Antibiotic treatment would be appropriate if the skin was broken and the injury was sustained in a fist fight with
the potential for introduced oral flora.
(u) B. Reduction followed by splinting is recommended for fifth metatarsal fractures with angulation of 15-40 degrees.
(c) C. Open reduction is indicated with angulation of greater than 40 degrees.
(u) D. With angulation of greater than 15 degrees, reduction should be performed prior to splinting.
Ref: (29)

162. Diagnosis/Gastrointestinal/Nutritional
A male patient presents for a routine physical examination. He denies chronic health problems, regular medication
use or previous surgeries. He exercises 4-5 times weekly, does not use tobacco products, and consumes alcohol in
moderation. His last intake of alcohol was two weeks ago while on a trip to Mexico. His review of systems and
physical examination are both negative. Routine labs were drawn which were significant for the following:
Total serum bilirubin 3.5 mg/dL (0.3-1.0 mg/dL), direct bilirubin 0.2 mg/dL (0.1-0.3 mg/dL), AST 35 U/L (0-35 U/L),
ALT 30 U/L (0-35 U/L), Alkaline Phosphatase 48 U/L (30-120 U/L) and GGT 12 U/L (1-94 U/L). What is the most
likely diagnosis in this patient?
A. Alcoholic hepatitis
B. Crigler-Najjar syndrome
C. Gilbert's syndrome
D. Wilson's disease
Explanations
(u) A. Alcoholic hepatitis presents with elevated ALT and AST.
(u) B. The Crigler-Najjar syndrome is typified by an isolated elevation in the unconjugated serum bilirubin and usually
causes the bilirubin to be elevated at higher levels (6-45 mg/dL).
(c) C. Gilbert's syndrome is a relatively common cause of mild isolated elevations in indirect serum bilirubin.
(u) D. Wilson's disease is a disorder of impaired biliary copper excretion which among many other problems will
cause a rise in total serum bilirubin but also significant elevations in AST.
Ref: (7)

60

163. Health Maintenance/ENT/Ophthalmology


Which of the following is the leading cause of permanent visual loss in a patient over the age of 75?
A. Blepharitis
B. Cataracts
C. Central retinal artery occlusion
D. Macular degeneration
Explanations
(u) A. Blepharitis is a chronic bilateral inflammatory condition of the lid margins.
(u) B. Cataracts are the clouding of the lens sufficient to reduce vision. Most develop slowly as a result of aging,
leading to gradual impairment of vision.
(u) C. Central retinal artery occlusion presents as a rare cause of sudden profound monocular visual loss.
(c) D. Age-related macular degeneration is the leading cause of permanent visual loss in the older population. The
exact cause is unknown, but the prevalence increases with each decade over age 50 years.
Ref: (28)

164. Diagnostic Studies/Pulmonology


A 42 year-old male smoker presents for further evaluation of a 4 cm solitary pulmonary nodule discovered on a recent
chest x-ray. Which of the following diagnostic tests is indicated next?
A. Bone scan
B. Thoractotomy
C. Mediastinoscopy
D. CT scan of chest
Explanations
(u) A. Bone scanning is indicated for evaluation of bone metastases, most commonly secondary to cancer of the
breast or prostate.
(u) B. Diagnostic thoractotomy is indicated for biopsy of the lesion should the CT scan of the chest indicate a
suspicious malignant lesion that is inaccessible to thoracoscopy.
(u) C. Mediastinoscopy can be utilized to further evaluate any enlarged mediastinal lymph nodes that may be found
on the CT scan of the chest, but is not indicated prior to the CT scan.
(c) D. A CT scan of the chest is needed to further evaluate the characteristics of the solitary pulmonary nodule and to
determine lymph node involvement or presence of multiple lesions.
Ref: (7)

165. Clinical Therapeutics/Infectious Diseases


A 16 year-old female who is a competitive swimmer presents with a complaint that her toe nails are thickened and
discolored. Examination reveals that the nails have a dry yellow brittle appearance along with a friable surface of the
nail plate. Which of the following is the treatment of choice?
A. Metronidazole (Flagyl)
B. Diflucan
C. Terbinafine (Sporonox)
D. Acyclovir
Explanations
(u) A. Metronidazole is not indicated for fungal infections of the nails.
(u) B. Diflucan is indicated for dermal fungal infections.
(c) C. Oral Terbinafine 250 mg is considered the best treatment for onychomycosis.
(u) D. Acyclovir is indicated for viral infections, not fungal infections.
Ref: (28)

61

166. History & Physical/Cardiology


A 26 year-old male presents with increased dyspnea with exercise. He has noted a decrease in his exercise
tolerance over the past several months. He denies chest pain or skipped heart beats. Echocardiogram reveals left
ventricular hypertrophy with asymmetric septal hypertrophy. Ejection fraction is 65%. Which of the following is the
most likely presenting history or physical exam finding?
A. He has an older brother with the same diagnosis.
B. An S3 gallop is heard.
C. Patient notes completing a course of adriamycin.
D. Elevated jugular venous distension is noted.
Explanations
(c) A. Hypertrophic cardiomyopathy can be genetic and present in 25% of first degree relatives.
(u) B. Dilated cardiomyopathy may present with an S3 gallop due to volume overload.
(u) C. Adriamycin chemotherapy can lead to cardiac dysfunction, dilated cardiomyopathy and eventually heart failure.
(u) D. Restrictive or dilated cardiomyopathy may present with JVD due to abnormalities of filling.
Ref: (11)

167. Clinical Intervention/Neurology


A 23 year-old patient presents with two days of fatigue, headache, fever and pain around the area in which she was
bitten by a stray baby raccoon in an unprovoked attack 10 days ago. She cleaned the small wound thoroughly. Which
of the following is the most appropriate intervention in this patient?
A. Do nothing and treat her symptoms
B. Human diploid cell rabies vaccine, 5 injections given all at once now
C. Rabies immunoglobulin only
D. Rabies immunoglobulins and human diploid cell rabies vaccine given 5 times in a 1-month period
Explanations
(u) A. See D for explanation.
(u) B. See D for explanation.
(u) C. See D for explanation.
(c) D. Bites by bats, skunks, and raccoons always require this regimen if the animal is not caught and tested.
Ref: (28)

168. Diagnosis/Dermatology
A 55 year-old female presents with a lesion on her face that is painful, bright red, with distinct raised borders. She
also is complaining of fever and chills. There is no evidence of any marks which would show a portal of entry. Which
of the following is the most likely diagnosis?
A. Impetigo
B. Erythrasma
C. Cellulitis
D. Erysipelas
Explanations
(u) A. Impetigo is a bacterial infection of the skin that is associated with honey-colored crusts.
(u) B. Erythrasma is a bacterial infection of the skin that is red in appearance and fluoresces a coral red color under a
Wood's lamp. Systemic symptoms are rarely seen and the borders tend to be less distinct.
(u) C. Cellulitis has many features of erysipelas but it goes on to involve the subcutaneous tissues. Patients with
cellulitis will not have raised borders and demarkation from uninvolved skin is not distinct with this condition.
(c) D. Erysipelas is a distinct type of superficial cellulitis with redness, a distinct and raised border, and sharp
demarkation from uninvolved skin. It is typically associated with systemic symptoms such as fever and chills. It is
caused by group A strep most commonly.
Ref: (10)

62

169. Diagnostic Studies/Obstetrics/Gynecology


What should be the initial evaluation of the etiology of infertility in a 25 year-old female who has been trying to
conceive for 1 year?
A. Endometrial biopsy
B. Hysterosalpingogram
C. Serum progesterone levels
D. Basal body temperature measurements
Explanations
(u) A. Endometrial biopsy will identify changes associated with infertility, however it is invasive and often done later in
the evaluation.
(u) B. Hysterosalpingogram provides information about the internal female genital tract. This is usually done later in
the work-up.
(u) C. Serum progesterone levels drawn at mid-luteal phase will help determine normal from abnormal cycles. Levels
above 15 ng/mL will indicate a normal cycle.
(c) D. Basal body temperature measurement is an excellent screening test for ovulation, and will help with the timing
of coitus.
Ref: (4)

170. Scientific Concepts/Gastrointestinal/Nutritional


What is the pathologic mechanism of Hirschsprung's disease?
A. Pyloric circular muscle hypertrophy causing gastric outlet obstruction
B. Absence of ganglion cells in the mucosal and muscular layers of the colon
C. A defect in the diaphragm leading to protrusion of the abdominal viscera into the thoracic cavity
D. Absence of relaxation of the lower esophageal sphincter and lack of peristalsis in the esophageal body
Explanations
(u) A. Pyloric circular muscle hypertrophy causing gastric outlet obstruction describes pyloric stenosis.
(c) B. Hirschsprung's disease results from an absence of ganglion cells in the mucosal and muscular layers of the
colon.
(u) C. A defect in the diaphragm leading to protrusion of the abdominal viscera into the thoracic cavity describes
congenital diaphragmatic hernia.
(u) D. Absence of relaxation of the lower esophageal sphincter and lack of peristalsis in the esophageal body
describes achalasia of the esophagus.
Ref: (13)

171. History & Physical/ENT/Ophthalmology


Which of the following is described as a harmless triangular nodule in the bulbar conjunctiva on either side of the iris?
A. Corneal arcus
B. Hordeolum
C. Pinguecula
D. Xanthelasma
Explanations
(u) A. Corneal arcus is an extremely common, bilateral, benign peripheral corneal degeneration, associated
hyperlipidemia.
(u) B. Hordeolum is characterized by a localized red, swollen, acutely tender area on the upper or lower lid.
(c) C. Pinguecula is a yellow elevated conjunctival nodule, more commonly on the nasal side, in the area of the
palpebral fissure.
(u) D. Xanthelasma appear as yellow plaques that occur on the anterior surface of the eyelid, usually bilaterally near
the inner angle of the eye.
Ref: (3)

63

172. Clinical Therapeutics/Pulmonology


A 2 year-old presents to the emergency department in acute respiratory distress. The parents relate a history of a
recent upper respiratory illness that was followed by a sudden onset of barking cough during the night, but this
morning they noted increased difficulty breathing. The child is noted to have stridor at rest, but has no evidence of
cyanosis. Which of the following is the most appropriate initial intervention?
A. Intravenous antibiotics
B. Endotracheal intubation
C. Inhaled mucolytic agent
D. Nebulized racemic epinephrine
Explanations
(u) A. Laryngotracheobronchitis is caused by viruses, not bacteria, and therefore antibiotic therapy is not indicated.
(u) B. If patients fail to respond to initial treatment and progress to impending respiratory failure, endotracheal
intubation is then indicated.
(u) C. Inhaled mucolytic agents are not indicated in the treatment of laryngotracheobronchitis.
(c) D. This patient most likely has laryngotracheobronchitis (viral croup). Treatment with nebulized racemic
epinephrine and glucocorticosteroids is indicated for patients with stridor at rest.
Ref: (13)

173. Health Maintenance/Cardiology


Which of the following lifestyle recommendations would most benefit the heart failure patient's quality of life?
A. Begin a regular exercise program
B. Total salt restriction
C. Home monitoring of blood pressure
D. Increase home oxygen requirements
Explanations
(c) A. Exercise training improves activity tolerance and deconditioning offering some recompensation.
(u) B. Although salt restriction is a recommendation total salt restriction would be unrealistic to achieve.
(u) C. Home monitoring of blood pressure will not improve a heart failure patient's quality of life.
(u) D. Increasing the requirements for home oxygen could signal that they are worsening and is palliative for their
quality of life.
Ref: (28)

174. Diagnosis/Obstetrics/Gynecology
On rectovaginal examination of a 72 year-old post-menopausal female a 3 cm by 3 cm right adnexal mass is
palpated. The rest of her physical examination is unremarkable. Her last gynecological examination was last year and
was unremarkable. Which of the following is the most likely diagnosis?
A. Ovarian cancer
B. Endometrioma
C. Uterine cancer
D. Functional ovarian cyst
Explanations
(c) A. Ovaries should not be palpable in the postmenopausal patient. Consider ovarian cancer until proven otherwise.
(u) B. An endometrioma is an endometriosis cyst of the ovary. The cyst is filled with thick, chocolate-colored fluid,
often called a "chocolate cyst". These cysts occur in menstruating women.
(u) C. 90% of patients with endometrial cancer present with vaginal bleeding. The uterus may be enlarged on
examination, however, a palpable ovary is not associated with this condition.
(u) D. Functional ovarian cysts occur in women who are ovulating.
Ref: (4)

64

175. Clinical Intervention/Gastrointestinal/Nutritional


A pregnant female presents at 32 weeks gestation with painless rectal bleeding and a bulging perianal mass when
straining which goes away. Which of the following is the most appropriate management of this patient?
A. Hemorrhoidectomy
B. Metronidazole (Flagyl)
C. Psyllium (Metamucil)
D. Sclerotherapy
Explanations
(u) A. Hemorrhoidectomy and sclerotherapy are reserved for severe Grade III and IV hemorrhoids.
(u) B. Metronidazole is not indicated in the treatment of hemorrhoids.
(c) C. Dietary fiber or psyllium bulk laxatives can be used to decrease straining with defecation.
(u) D. See A for explanation.
Ref: (28)

176. Diagnostic Studies/Neurology


A 12 month-old in the emergency department is diagnosed with possible viral meningitis. Which of the following
cerebral spinal fluid (CSF) laboratory results is most consistent with this diagnosis?
A. Decreased CSF glucose level and increased protein
B. Decreased CSF total protein level and very few neutrophils
C. Increased CSF mononuclear cells and normal glucose
D. Increased CSF C-reactive protein and normal glucose
Explanations
(u) A. Decreased CSF glucose level and increased protein is consistent with bacterial meningitis.
(u) B. Decreased CSF total protein level and very few neutrophils is a finding in syphilitic meningitis.
(c) C. In aseptic meningitis, CSF shows mainly mononuclear cells within 6-8 hours, glucose is normal and there is
normal to lower protein.
(u) D. Increased CSF C-reactive protein and normal glucose
is found in any inflammatory process affecting the CSF.
Ref: (1)

177. Scientific Concepts/Psychiatry/Behavioral Medicine


Which of the following medical complications of eating disorders is due to purging (vomiting/laxative abuse)?
A. Intestinal obstruction
B. Reduced thyroid metabolism
C. Amenorrhea
D. Electrolyte abnormalities
Explanations
(u) A. Intestinal obstruction is not caused by vomiting.
(u) B. Reduced thyroid metabolism is a complication of anorexia nervosa related to cachexia.
(u) C. Amenorrhea occurs as a result of anorexia nervosa with weight loss.
(c) D. Electrolyte abnormalities, particularly hypokalemia, hypochloremic alkalosis, and hypomagnesemia may occur
as a result of purging.
Ref: (14)

178. Clinical Therapeutics/Dermatology


Which of the following is considered the antibiotic of choice in the treatment of human bite wounds?
A. Ampicillin
B. Penicillin
C. Augmentin
D. Ciprofloxacin

65

Explanations
(u) A. See C for explanation.
(u) B. See C for explanation
(c) C. Augmentin is considered to be the antibiotic of choice for human bites that may be contaminated with Eikenella
corrodens, strep viridans, and staph aureus.
(u) D. See C for explanation.
Ref: (7)

179. History & Physical/Cardiology


A newborn presents with blue discoloration of the peripheral extremities which worsens with exposure to cold and
improves with warming. Which of the following is the most likely cause?
A. Raynauds phenomenon
B. Livedo reticularis
C. Erythromelagia
D. Acrocyanosis
Explanations
(u) A. Raynauds phenomenon has a triphasic color response and is exacerbated by cold or emotions.
(u) B. Livedo reticularis is characterized by a lacy pattern on the skin of the lower extremities.
(u) C. Erythromelagia is red, painful extremities.
(c) D. Acrocyanosis is a blue discoloration of the digits, usually of the peripheral extremities in newborns, which
worsens with exposure to cold and improves with warming.
Ref: (11)

180. Diagnosis/Urology/Renal
A 25 year-old female presents with right lower quadrant pain, right flank pain, nausea, and vomiting. Her temperature
is 39.6 degrees C. There is right CVA tenderness and RLQ tenderness. Pelvic exam is unremarkable. Urinalysis
reveals pH 7.0, trace protein, negative glucose, negative ketones, positive blood, and positive nitrates. Specific
gravity is 1.022. Microscopic shows 102 RBCs/HPF, 50-75 WBCs/HPF, rare epithelial cells, and WBC casts. The
most likely diagnosis is
A. acute salpingitis.
B. nephrolithiasis.
C. acute pyelonephritis.
D. appendicitis.
Explanations
(u) A. Acute salpingitis would be suggested if pelvic exam abnormalities were present.
(u) B. Nephrolithiasis does not usually present with fever or casts. Urinalysis will have RBCs present.
(c) C. Acute pyelonephritis presents with flank pain, fever, and generalized muscle tenderness. Urinalysis shows
pyuria with leukocyte casts.
(u) D. This scenario is consistent with acute pyelonephritis, not acute appendicitis.
Ref: (28)

181. Clinical Intervention/Pulmonology


A 62 year-old male presents with a right hilar mass. Needle-biopsy of the mass reveals the presence of small-cell
carcinoma and a bone scan reveals the presence of scattered hot spots throughout the skeleton. Which of the
following is the most appropriate treatment?
A. Lobectomy
B. Pneumonectomy
C. Thoracic radiation therapy
D. Combination chemotherapy

66

Explanations
(u) A. Small-cell carcinoma of the lung is rarely treatable with surgical resection. Surgery may be indicated as part of
the treatment protocol for small peripheral lesions without any evidence of metastasis.
(u) B. See A for explanation.
(u) C. While thoracic radiation therapy has been shown to be beneficial for patients with limited small-cell lung cancer,
no benefit has been observed for patients with extensive disease defined as the presence of metastatic disease.
(c) D. Combination chemotherapy is the treatment of choice for a patient with small-cell carcinoma of the lung.
Ref: (28)

182. Diagnostic Studies/Gastrointestinal/Nutritional


A patient presents with a long history of dyspepsia refractory to maximum appropriate therapy. A recent upper GI
series revealed multiple gastric and duodenal ulcerations with prominent mucosal folds. What diagnostic study would
be confirmatory of this patient's suspected diagnosis?
A. EGD with duodenal biopsy
B. Serum gastrin level
C. Helicobacter pylori serology
D. Abdominal CT scan
Explanations
(u) A. An endoscopic biopsy of the duodenum may be necessary in some cases to ensure the absence of cancer,
etc., but a biopsy in this area is not confirmatory for Zollinger-Ellison Syndrome, which is highly suspect, nor does it
allow samples of the gastric mucosa to be studied.
(c) B. Serum gastrin levels can be elevated for many reasons; however significant elevations in a fasting state as well
as with the secretin stimulation test are confirmatory for Zollinger-Ellison syndrome which is highly suspect in this
case.
(u) C. Helicobacter pylori can cause symptoms refractory to medical management and an infection can occur
concomitantly with Zollinger-Ellison syndrome, but it does not confirm the diagnosis of a gastrinoma or similar
hypersecretory disease.
(u) D. An abdominal CT scan is not specific enough to confirm the diagnosis of Zollinger-Ellison syndrome.
Ref: (7)

183. Scientific Concepts/ENT/Ophthalmology


What is the most common location of anterior nasal epistaxis?
A. Middle turbinate
B. Posterior ethmoid artery
C. Kiesselbach's triangle
D. Inferior turbinate
Explanations
(u) A. See D for explanation.
(u) B. See D for explanation.
(c) C. 90% of all cases of anterior epistaxis originate from Kiesselbach's triangle.
(u) D. The inferior turbinate is the most common location for posterior epistaxis.
Ref: (29)

184. Diagnosis/Hematology
A 48 year-old male complains of weakness and general malaise for about 2 months. Patient denies any recent illness
and does not take any medications. Physical examination reveals a pale looking male in no acute distress. His heart
rate is 110 beats/minute without a murmur and his abdominal examination reveals hepatosplenomegaly. A CBC
reveals the WBC to be 62,000/microliter, Hgb is 8.3 gms/dl, Hct is 24.6%. A differential reveals a predominance of
monoblasts and promyelocytes with Auer rods present. What is the most likely diagnosis?
A. Acute lymphocytic leukemia
B. Acute myelogenous leukemia
C. Chronic lymphocytic leukemia
D. Chronic myelogenous leukemia

67

Explanations
(u) A. Acute lymphocytic leukemia (ALL) is a leukemia most often seen in children with lymphoblasts predominating.
Adults who develop ALL usually have a prolymphocytic cell presentation.
(c) B. Acute myelogenous leukemia (AML) is a leukemia that presents in adults with a cell lineage of the immature
granulocytic cells seen in the peripheral blood e.g. myeloblasts, promyelocytes. Auer rods are commonly seen in this
condition.
(u) C. Chronic lymphocytic leukemia (CLL) is a leukemia that presents in older adults with WBC counts up to
100,000/mm3. On peripheral blood smears the cell that predominates is a mature lymphocyte.
(u) D. Chronic myelogenous leukemia (CML) is a leukemia that presents in adults with a cell lineage of mature cell
lines with a marked increase in basophils and eosinophils. These cells are also known for having the Philadelphia
chromosome.
Ref: (28)

185. Clinical Therapeutics/Obstetrics/Gynecology


A 56 year-old female patient comes to the office for evaluation. She complains of dyspareunia and a thin vaginal
discharge. On physical examination atrophic vulvar changes are noted associated with vaginal petechiae and a thin
clear discharge. What medication is recommended to treat this patient's symptoms?
A. Topical estrogen (Estrace) cream
B. Topical hydrocortisone (Gynecort)
C. Metronidazole (MetroGel) vaginal gel
D. Terconazole (Terazol) vaginal suppository
Explanations
(c) A. This patient has atrophic vaginitis which is very common in postmenopausal patients. The vagina will appear
atrophied, will look pale, and thin and dry in appearance. Best to treat with topical or oral estrogen preparations.
(u) B. Topical hydrocortisone would not be used in atrophic vaginitis.
(u) C. MetroGel is the drug of choice for bacterial vaginosis.
(u) D. Terazol is the treatment for non-albicans yeast infections.
Ref: (4)

186. History & Physical/Orthopedics/Rheumatology


The neuromuscular hallmark of polymyalgia rheumatica is
A. asymmetric muscle pain and stiffness.
B. distal paresthesias.
C. muscle weakness.
D. proximal muscle pain and stiffness.
Explanations
(u) A. Polymyalgia rheumatica is associated with pain and stiffness but typically occurs in a symmetric distribution.
(u) B. Peripheral neurons are unaffected by polymyalgia rheumatica and is not associated with neuropathic
symptoms.
(u) C. Although patients may appear to have proximal muscle weakness, this is usually related to pain and is not true
measurable weakness.
(c) D. Proximal symmetric muscle pain and stiffness, particularly involving the shoulder, neck and pelvic girdle, is the
musculoskeletal hallmark of polymyalgia rheumatica.
Ref: (11)

187. Diagnosis/Cardiology
A two month-old infant appeared well until three weeks ago when he became dyspneic and had difficulty feeding. A
4/6 holosystolic murmur is heard at the left lower sternal border in the 3rd ICS. An electrocardiogram (ECG) shows
left and right ventricular hypertrophy. Which of the following is the most likely diagnosis?
A. Atrial septal defect
B. Pulmonary hypertension
C. Ventricular septal defect
D. Tricuspid insufficiency

68

Explanations
(u) A. An ASD usually presents with a soft mid-systolic murmur in the second left ICS with a widely split and fixed S2.
(u) B. With pulmonary hypertension the murmur may be most consistent with pulmonary or tricuspid insufficiency.
(c) C. This is a classic presentation for a ventricular septal defect.
(u) D. Tricuspid insufficiency is a systolic murmur heard best at the 4th ICS that may radiate to the apex.
Ref: (11)

188. Diagnostic Studies/Pulmonology


A 56 year-old male with a 40 pack-year smoking history presents complaining of progressive shortness of breath.
Spirometry reveals an FEV1 of 2 L (40% of predicted), an FVC of 4 L (80% of predicted) and an FEV1/FVC of 50%.
These findings are most consistent with
A. sarcoidosis.
B. chronic bronchitis.
C. interstitial lung disease.
D. congestive heart failure.
Explanations
(u) A. Sarcoidosis, interstitial lung disease and congestive heart failure most commonly produce a restrictive pattern
on spirometry with a reduction in forced expiratory volume in one second (FEV1) and forced vital capacity (FVC)
secondary to the decrease in total lung capacity (TLC), but the FEV1/FVC ratio is usually normal or increased, not
decreased.
(c) B. The reduced FEV1 and FEV1/FVC is characteristic of an obstructive pattern that is seen in chronic obstructive
pulmonary diseases, such as chronic bronchitis.
(u) C. See A for explanation.
(u) D. See A for explanation.
Ref: (1)

189. Clinical Intervention/Dermatology


An elderly woman presents to your clinic complaining of unilateral facial pain and painful lesions. She also complains
of blurred vision in the ipsilateral eye. On examination she has several vesicles on an erythematous base, some of
the lesions with crusts. They are distributed in a dermatomal pattern and involve the skin overlying the maxillary
region and the tip of her nose. Which of the following is the next most appropriate intervention in the care of this
patient?
A. KOH prep
B. Culture for bacteria
C. Referral to an ophthalmologist
D. Application of corticosteroids
Explanations
(u) A. A KOH prep examining for fungal elements is not indicated in this case.
(u) B. While bacterial infection may occur along with the viral infection, a bacterial culture is not usually employed.
(c) C. Immediate referral to an ophthalmologist is needed when herpes keratitis is suspected, as in this case. A
fluorescein stain of the eye might reveal the typical dendritic corneal lesion.
(h) D. Application of corticosteroids may cause proliferation of the virus and should only be used by an
ophthalmologist.
Ref: (28)
190. Diagnosis/Endocrinology
A male is found to have an eunuchoid body habitus, gynecomastia, diminished secondary sex characteristics, and
small and firm testicles. Which of the following is the most likely diagnosis?
A. Fragile X syndrome
B. Gaucher disease
C. Klinefelter syndrome
D. Kallmann syndrome

69

Explanations
(u) A. Fragile X syndrome is the second most common inherited form of mental retardation. Affected males show
macroorchidism (enlarged testis) after puberty, large ears, and a prominent jaw, high-pitched voice, and mental
retardation.
(u) B. Patients with Gaucher disease have anemia and thrombocytopenia as its major manifestation. These patients
may also have cortical erosions of bone and may have bone pain due to local infarctions.
(c) C. Men with Klinefelter syndrome have an extra X chromosome and are phenotypically normal until puberty. After
puberty, these men have long legs and arms, a female escutcheon, gynecomastia, and small testes. Low serum
testosterone and infertility along with a decrease in executive functioning are common in these men.
(u) D. Kallmann's syndrome is the most common cause of congenital isolated gonadotropin deficiency. It is
associated with anosmia and hyposmia caused by hypoplasia of the olfactory bulbs. About one-half of these patients
will have renal agenesis. There is also an increase in cryptorchism, sensorineural hearing deafness, and cleft lip.
Ref: (28)
191. History & Physical/Gastrointestinal/Nutritional
Which of the following is best described as a hard nodule in the periumbilical region?
A. Hodgkin's lymphoma
B. Gastric bezoar
C. Sister Mary Joseph nodule
D. Virchow's node
Explanations
(u) A. Hodgkin's lymphoma will more likely affect intra-abdominal lymph nodes.
(u) B. Gastric bezoar can present as a palpable abdominal mass, but is not indicative of intra-abdominal/intra-pelvic
cancer.
(c) C. The Sister Mary Joseph nodule can occur with metastatic gastric and pancreatic cancers. The nodule is
generally minimally to non-tender to palpation.
(u) D. Virchow's node is found in cases of metastatic disease, but is represented by a left supraclavicular lymph node.
Ref: (7)

192. Diagnostic Studies/Cardiology


Which of the following laboratory tests would be elevated in a patient with symptomatic heart failure?
A. Lactate dehydrogenase (LDH)
B. Troponin I (TnI)
C. C-reactive protein (CRP)
D. Brain natriuretic peptide (BNP)
Explanations
(u) A. LDH is not a sensitive marker in patients with heart failure.
(u) B. Troponin measurements are specific for myocardial infarctions.
(u) C. Increased serum levels of CRP are found in patients with unstable angina and MI. They can be a strong
predictor of coronary events.
(c) D. BNP is expressed in the ventricles and is a sensitive assay in patients with heart failure.
Ref: (28)

193. Clinical Therapeutics/ENT/Ophthalmology


A 34 year-old female presents with recurrent bouts of dizziness, tinnitus, and hearing loss. She states that the
episodes are incapacitating and cause her to become nauseous and vomit. The attacks last about one hour and the
symptoms disappear after a few days. The last two episodes were treated with meclizine (Antivert) and
prochlorperazine (Compazine) at the emergency room. Audiologic testing reveals low-tone frequency hearing loss.
Which of the following is the most appropriate long-term management for this patient?
A. Epley maneuver
B. Diuretics and low-sodium diet
C. Broad-spectrum antibiotics and Ibuprofen
D. Scopolamine transdermal patch

70

Explanations
(u) A. Epley maneuver is used to treat benign paroxysmal positional vertigo.
(c) B. Diuretics and a low sodium diet will decreases the endolymphatic pressure in the semicircular canals, which is
believed to be elevated in Meniere's disease, and help relieve symptoms.
(u) C. Broad-spectrum antibiotics and ibuprofen are used to treat otitis media, not Meniere's disease.
(u) D. A scopolamine patch is useful for treatment of a single episode, but not long-term management.
Ref: (28)
194. Scientific Concepts/Pulmonology
A 3 year-old male with cystic fibrosis develops pneumonia. Which of the following is the most likely etiology of the
pneumonia?
A. Escherichia coli
B. Staphylococcus epidermidis
C. Pseudomonas aueroginosa
D. Streptococcus pneumoniae
Explanations
(u) A. See C for explanation.
(u) B. See C for explanation.
(c) C. Initially in the first few months of life, respiratory infection is common with Staphylococcus aureus and
Haemophilus influenzae, but after that Pseudomonas aueroginosa becomes the major causative organism for
infections.
(u) D. See C for explanation.
Ref: (13)

195. Clinical Intervention/Urology/Renal


A 40 year-old patient with a history of recurrent kidney stones presents with acute onset of right flank pain and
hematuria. The patient is afebrile and pain is poorly controlled on oral medications. On CT scan a 1 cm stone is noted
in the renal pelvis. Which of the following is the most appropriate intervention for this patient?
A. Antibiotics
B. Shock wave lithotripsy
C. Ureterolithotomy
D. Fluid hydration
Explanations
(u) A. There is no indication of infection and antibiotics are not warranted at this time.
(c) B. Extracorporeal shock wave lithotripsy is indicated in patients with stones greater than 6 mm in size or
intractable pain.
(u) C. Ureterolithotomy is an open surgical procedure and therefore very invasive. In this situation an ESWL would be
as effective and safer.
(u) D. While fluid hydration is indicated, a stone greater than 6 mm will typically not spontaneously pass and surgical
therapy is indicated.
Ref: (11)

196. Diagnosis/Infectious Diseases


An unimmunized child has had a mild fever for several days. Today, the child is brought to the clinic because of the
development of a rash. The rash is a pink discrete macular eruption mostly on the face and trunk. Postauricular and
suboccipital lymph nodes are palpable. The child has a slight fever, but does not appear sick. Which of the following
is the most likely diagnosis?
A. Roseola
B. Rubeola
C. Rubella
D. Erythema infectiosum

71

Explanations
(u) A. Continued fever after rash onset and lymphadenopathy are not consistent with roseola.
(u) B. High fever and lethargy are prominent features of rubeola and help distinguish it from rubella.
(c) C. The characteristic lymphadenopathy and lack of systemic symptoms are most consistent with rubella.
(u) D. The presence of lymphadenopathy is not consistent with erythema infectiosum, which typically presents with
reddened cheeks.
Ref: (28)

197. Diagnostic Studies/Orthopedics/Rheumatology


A 74 year-old male presents with one month history of right shoulder pain without any known precipitant. His pain
involves an area from the right paraspinous musculature to the right deltoid with occasional radiation down the arm.
Pain is worse with movement of the shoulder and is not relieved by acetaminophen. He reports numbness of the right
index finger and thumb. Physical examination of the shoulder is limited by pain. There is decreased grip strength and
absent pinprick sensation in the index finger and thumb. Relexes are normal. What is the most appropriate initial
study to obtain?
A. EMG with nerve conduction
B. MRI of the right shoulder
C. Radiographs of the neck
D. Radiographs of the right shoulder
Explanations
(u) A. Although electromyography and nerve conduction studies would be helpful in localizing the nerves involved,
this is not an appropriate first-line diagnostic test.
(u) B. Although there is pain in the shoulder, this patient presents with cervical radiculopathy and shoulder studies are
not appropriate.
(c) C. Cervical radiculopathy may be due to intervertebral foraminal osteophytes which will be identified on plain film
radiographs of the neck making this the appropriate first step in evaluation.
(u) D. Although there is pain in the shoulder, this patient presents with cervical radiculopathy and shoulder studies are
not appropriate.
Ref: (26)

198. Clinical Therapeutics/Cardiology


When prescribing loop diuretics which of the following electrolytes should be most closely monitored?
A. Calcium
B. Potassium
C. Sodium
D. Chloride
Explanations
(u) A. See B for explanation.
(c) B. Potassium along with magnesium should be monitored when prescribing loop diuretics.
(u) C. See B for explanation.
(u) D. See B for explanation.
Ref: (11)

199. Diagnosis/Neurology
An 8 year-old male presents with episodes described as 20-second lapses of awareness during which he blinks his
eyes. After these attacks, he resumes his previous activity. Which of the following is the most likely diagnosis?
A. Tonic-clonic seizure
B. Myoclonic seizure
C. Absence seizure
D. Focal seizure

72

Explanations
(u) A. Tonic-clonic seizures are associated with a loss of consciousness, tonic-clonic movements, and postictal
confusion or sleep.
(u) B. Myoclonic seizures are associated with mild myoclonic jerks of the neck and shoulder flexor muscles when
falling asleep.
(c) C. Absence seizures are characterized by lapses of consciousness associated with postictal confusion.
(u) D. Focal seizures are associated with motor or sensory symptoms involving any portion of the body.
Ref: (5)

200. History & Physical/Obstetrics/Gynecology


You are performing a routine gynecological examination on a 49 year-old female. She states that for the last six
months her periods have been getting heavier and lasting for seven days duration. She also tells you that she has
been experiencing urinary frequency and constipation. Her abdominal exam is unremarkable. On pelvic examination
you feel a slightly enlarged uterus and a left adnexal mass that moves with the uterine fundus. Which of the following
is the most likely diagnosis?
A. Adenomyosis
B. Uterine fibroids
C. Ovarian neoplasm
D. Diverticular disease in the colon
Explanations
(u) A. Adenomyosis occurs when endometrial glands and stroma grow into the muscle of the uterus. Patients often
have severe secondary dysmenorrhea and menorrhagia; the uterus will be symmetrically enlarged and often "boggy".
(c) B. Patients with uterine fibroids often have menorrhagia and/ or metrorrhagia. On examination, the uterus will be
larger, and irregularly-shaped. If a mass is present and it moves with the uterus, it is suggestive of a fibroid.
(u) C. It is important to consider an ovarian neoplasm in a patient with a palpable adnexal mass, however, most
adnexal masses are asymptomatic and not associated with menstrual changes.
(u) D. Left-sided bowel disease is more common in older patients and would not be associated with menstrual
changes.
Ref: (4)

201. Diagnostic Studies/Pulmonology


Which of the following findings confirm the adequacy of a sputum specimen for Gram stain and culture?
A. Decreased red blood cells
B. Decreased bronchial epithelial cells
C. Increased Gram positive cocci
D. Increased polymorphonuclear leukocytes
Explanations
(u) A. The presence or absence of red blood cells is not a criterion for determining the adequacy of a sputum sample.
(u) B. The presence of bronchial epithelial cells confirms the sample came from the lower respiratory tract.
(u) C. The presence of Gram positive cocci has no bearing on the adequacy of the sputum sample.
(c) D. The presence of increased polymorphonuclear leukocytes and the absence of squamous epithelial cells are the
criteria utilized to evaluate the adequacy of a sputum sample.
Ref: (1)

202. Clinical Therapeutics/Psychiatry/Behavioral Medicine


A 25 year-old female presents to the emergency department after she collapsed at work. Medical history is significant
for a mood disorder that causes her to have "wild mood swings and reckless behavior" according to her husband.
She was diagnosed a year ago and since has been treated with several medications. Her symptoms today consist of
nausea, vomiting, fatigue, tremor, and hyperreflexia. Lab results show an elevated BUN and creatinine, low sodium
and elevated drug levels. All other results are normal. Which of the following medications is most likely the cause of
her symptoms?
A. Lithium (Lithobid)
B. Lorazepam (Ativan)
C. Carbamazepine (Tegretol)
D. Risperidone (Risperdal)

73

Explanations
(c) A. Any sodium loss results in increased lithium levels. Signs and symptoms include vomiting and diarrhea which
exacerbate the problem. Tremors, muscle weakness, confusion, vertigo, ataxia, hyperreflexia, rigidity, seizures, and
coma may also be present.
(u) B. See A for explanation.
(u) C. See A for explanation.
(u) D. See A for explanation.
Ref: (28)

203. Diagnosis/ENT/Ophthalmology
A 58 year-old patient presents with spells of dizziness which is described as a spinning sensation. This has occurred
several times a day for the last month. The patient also complains of some mild hearing loss, fullness, and a blowing
sound in the right ear. Which of the following is the most likely diagnosis?
A. Meniere's syndrome
B. Labyrinthitis
C. Benign paroxysmal positioning vertigo
D. Vestibular neuronitis
Explanations
(c) A. The classic findings of Meniere's syndrome consists of episodic vertigo, with discrete vertigo spells lasting 20
minutes to several hours in association with fluctuating low-frequency sensorineural hearing loss, tinnitus, and a
sensation of aural pressure.
(u) B. Labyrinthitis is an acute onset of continuous, usually severe vertigo lasting several days to a week,
accompanied by hearing loss and tinnitus.
(u) C. Benign paroxysmal positioning vertigo is a type of vertigo associated with changes in head position, often
rolling over in bed.
(u) D. Vestibular neuronitis is a paroxysmal, usually single attack of vertigo that occurs without accompanying
impairment of auditory function and will persist for several days to weeks before clearing.
Ref: (28)

204. History & Physical/Orthopedics/Rheumatology


A 30 year-old diabetic female complains of persistent numbness in her right thumb and forefinger that has been
awakening her from sleep for the past week. She is right hand dominant and denies any history of activities involving
repetitive motion of the hands. Which of the following is the next step in the evaluation of this patient?
A. Electromyogram (EMG)
B. Nerve conduction velocity (NCT)
C. Phalen maneuver
D. Finkelstein test
Explanations
(u) A. EMG may be performed in a patient who is being considered for surgery.
(u) B. Nerve conduction test may be performed if operative intervention is being planned.
(c) C. The Phalen maneuver is used to reproduce the symptoms of carpal tunnel syndrome by flexion of the wrist.
(u) D. The Finkelstein test is performed when deQuervain's tenosynovitis is suspected.
Ref: (26)

205. Clinical Intervention/Hematology


A patient with a history of multiple trauma has received a combination of 12 units of packed red blood cells and whole
blood while in the emergency department. Later that night it is noted that the patient is beginning to ooze blood from
sites of injury and needle punctures. Which of the following is the most appropriate intervention?
A. Continue transfusions of packed red blood cells
B. Fresh frozen plasma
C. Albumin
D. Factor VIII concentrate

74

Explanations
(h) A. Continued administration of PRBC without replacement of FFP will result in possible volume overload and
continued bleeding.
(c) B. When a patient receives more than one total blood volume (10 units) without replacement of clotting factors, the
patient will continue to bleed without administration of fresh frozen plasma.
(u) C. Albumin is used for protein and intravascular volume replacement.
(h) D. Factor VIII concentrate is used in the treatment of hemophilia A and would not effect the bleeding in this
patient.
Ref: (29)

206. Clinical Therapeutics/Gastrointestinal/Nutritional


A person presenting with bleeding esophageal varicies should be treated with which of the following while awaiting
arrival of endoscopy?
A. Carafate (Sucralfate)
B. Octreotide (Sandostatin)
C. Omeprazole (Prilosec)
D. Enoxaparin (Lovenox)
Explanations
(u) A. Carafate is not indicated in acute gastrointestinal bleeding.
(c) B. Octreotide is a vasoacctive drug used in the treatment of GI bleeding as well as somatostatin, vasopressin, and
terlipressin. Somatostatin and octreotide are preferred due to safety and less incidence of serious side effects.
(u) C. Omeprazole, a proton pump inhibitor, is not indicated in acute gastrointestinal hemorrhage.
(h) D. Enoxaparin will increase bleeding and therefore contraindicated in GI bleeding.
Ref: (29)

207. Diagnostic Studies/Cardiology


Which of the following studies is the best initial diagnostic evaluation to estimate ventricular size and hypertrophy?
A. Electrocardiogram (ECG)
B. Cardiac CT scan
C. Echocardiogram
D. Myocardial perfusion imaging
Explanations
(u) A. ECG is not sensitive or reliable to estimate ventricular size and hypertrophy.
(u) B. Cardiac CT scan can detect coronary calcification, but is most sensitive to assess disorders of the aorta.
(c) C. Echocardiogram provides the safest and most reliable means to evaluate ventricular size, hypertrophy and
function.
(u) D. Myocardial perfusion imaging is used for measurement of LV ejection fraction and assess regional wall motion
abnormalities.
Ref: (11)

208. Diagnosis/Psychiatry/Behavioral Medicine


Formications are most commonly associated with which of the following?
A. Delusional disorder
B. Adverse drug reaction
C. Alcohol withdrawal
D. Obsessive compulsive disorder
Explanations
(u) A. The diagnosis of a delusional disorder requires the presence of nonbizzarre delusions of at least one months
duration that are not attributed to another disorder.
(u) B. Adverse drug reactions are not known to cause formication.
(c) C. Formications are the sensation of insects crawling on the skin and is commonly associated with delirium
tremens from alcohol withdrawal and cocaine addiction.
(u) D. Patients with OCD do not have formication with their disorder.
Ref: (14)

75

209. History & Physical/Pulmonology


A 24 year-old male presents in respiratory distress and appears quite ill. A Gram stain and culture of the sputum
reveals gram-positive cocci in clumps and a chest x-ray reveals multiple patchy infiltrates with some cavitations.
Which of the following is most likely to also be found in his medical history?
A. IV drug abuse
B. Alcohol abuse
C. Poor dental hygiene
D. HIV positive patient
Explanations
(c) A. This patient has pneumonia caused by Staphylococcus aureus which is commonly associated with a history of
intravenous drug use, influenza epidemics and the hospital setting.
(u) B. A history of alcohol abuse is commonly seen with pneumonia caused by Klebsiella pneumoniae.
(u) C. Poor dental hygiene is associated with pneumonia caused by anaerobes.
(u) D. HIV positive patients are most at risk for development of pneumonia caused by Pneumocystis jiroveci.
Ref: (28)

210. Clinical Therapeutics/Neurology


Which of the following is first-line treatment for acute inflammatory demyelinating polyneuropathy variant of GuillainBarre?
A. Neostigmine
B. IV Solu-Medrol
C. Phenytoin
D. IV immunoglobulin

Explanations
(u) A. Neostigmine may be used for a more chronic variant for symptoms, but not in acute inflammatory variant.
(u) B. Treatment with corticosteroids is ineffective and may prolong recovery time.
(u) C. Phenytoin may be used for ongoing neuropathic pain, but is not useful in any type of acute neuropathy.
(c) D. IV immunoglobulin is effective in patients with Guillain-Barre.
Ref: (28)

211. Diagnostic Studies/Urology/Renal


A 21 year-old female presents with dysuria. On examination of the urine, many squamous epithelial cells are noted.
Which of the following is the next best step in the evaluation or treatment of this patient?
A. Order urine culture and sensitivity
B. Obtain renal ultrasound
C. Repeat urinalysis with a clean catch sample
D. Refer to a nephrologist
Explanations
(u) A. See C for explanation.
(u) B. See C for explanation.
(c) C. The presence of many squamous epithelial cells indicates contamination with vaginal flora. The test should be
repeated with a clean catch specimen.
(u) D. See C for explanation.
Ref: (24)

76

212. Diagnosis/Orthopedics/Rheumatology Topic


A 42 year-old male sustained a closed left tibial fracture in a fall two days ago. He was treated with a cast for
immobilization. Acutely, he developed severe pain in his left leg. Examination reveals the anterolateral aspect of the
leg to be exquisitely tender to palpation. The patient has extreme pain with plantar flexion. What is the most likely
diagnosis?
A. Compartment syndrome
B. Deep venous thrombosis
C. Osteomyelitis
D. Complex regional pain syndrome
Explanations
(c) A. One of the earliest signs of compartment syndrome is severe pain that occurs with extension of the involved
muscles.
(u) B. DVT below the knee usually occurs in the posterior compartment and, while potentially painful, should not
produce the degree of pain described.
(u) C. The pain of osteomyelitis, while potentially severe, should not be greatly exacerbated by manipulation of the
overlying muscles.
(u) D. Complex regional pain syndrome can present with pain but it is typically a burning pain and often accompanied
by vasomotor symptoms.
Ref: (27)

213. History & Physical/Gastrointestinal/Nutritional


What is considered the most common physical examination finding for intestinal obstruction?
A. Distention
B. Fluid wave
C. Rigidity
D. Tenderness
Explanations
(c) A. Abdominal distention is the most common hallmark of all kinds of intestinal obstructions though its presence
can be variable depending on the duration and exact location of the obstruction.
(u) B. Fluid wave is noted with ascites, not intestinal obstruction.
(u) C. Rigidity is most often minimal to absent except in the most extreme of late presenting strangulated obstructions
and is more commonly found early in the course of peritonitis and related phenomenon.
(u) D. Tenderness, like rigidity, is not a predominant finding in obstructed bowel until very late in the course of
advancing, untreated cases. At this time distention and symptoms of shock will also be present and command the
clinical picture.
Ref: (7)

214. Clinical Therapeutics/Psychiatry/Behavioral Medicine


Which of the following classes of antidepressants is associated with anticholinergic side effects, including cardiac
dysrhythmias, dry mouth, sedation, and orthostatic hypotension?
A. Selective serotonin reuptake inhibitors
B. Monoamine oxidase inhibitors
C. Tricyclic antidepressants
D. Atypical antidepressants
Explanations
(u) A. SSRI's do not generally cause anticholinergic side effects.
(u) B. MAOI's mainly cause orthostatic hypotension and sympthomimetic effects.
(c) C. TCA's have well known anticholinergic effects.
(u) D. Atypical antidepressants do not cause anticholinergic side effects.
Ref: (28)

77

215. Scientific Concepts/Endocrinology


Which of the following hormones primarily inhibits growth hormone secretion from the pituitary gland?
A. Testosterone
B. Somatostatin
C. Dopamine
D. Insulin-like growth factor
Explanations
(u) A. Testosterone has no relationship to growth hormone.
(c) B. Somatostatin inhibits the release of growth hormone from the pituitary gland as well as hyperglycemic states. It
is therefore useful in the treatment of excessive growth hormone release that occurs with gigantism and acromegaly.
(u) C. Dopamine is a vasoactive substance whose main effect is to cause vasoconstriction making it useful in treating
hypotension that does not respond to a fluid challenge.
(u) D. Insulin-like growth factor parallels the release of growth hormone and growth hormone itself stimulates the
release of insulin-like growth factor from the liver and other tissues.
Ref: (28)

216. Diagnosis/Cardiology
A 44 year-old female presents to clinic for evaluation of a syncopal episode that occurred while walking her dog two
days ago. She denies amnesia or head trauma. She has had increasing dyspnea on exertion and pedal edema.
Physical examination reveals clubbing of her fingers and central cyanosis. Auscultation of the heart reveals tricuspid
insufficiency, widely split second heart sound with a palpable P2. Echocardiogram reveals a large ostium secundum
atrial septal defect with bidirectional flow. Which of the following is a secondary complication in this patient?
A. Left heart failure
B. Ebsteins anomaly
C. Tricuspid stenosis
D. Pulmonary hypertension
Explanations
(u) A. This patient would more likely have right heart failure than left heart failure.
(u) B. Ebsteins anomaly is apical displacement of the septal tricuspid leaflet and not caused by an ASD.
(u) C. Tricuspid stenosis is not caused by an ASD.
(c) D. Her symptoms and exam findings are consistent with pulmonary hypertension and in her case, Eisenmengers
disease, which is a late finding.
Ref: (11)

217. Clinical Therapeutics/Orthopedics/Rheumatology


A 27 year-old male presents with gradually worsening low back pain and stiffness for the past two years. His
symptoms are worse upon awakening and gradually improve throughout the day. Lumbosacral flexion is less than
50%. Lumbosacral spine films show erosions in the joint line of both sacroiliac joints. HLA-B27 test is positive. Which
of the following is the most appropriate first-line medication for this patient?
A. Indomethacin (Indocin)
B. Methotrexate (Rheumatrex)
C. Prednisone (Medrol dose pack)
D. Sulfasalazine (Azulfidine)
Explanations
(c) A. NSAIDs, such as indomethacin, are the mainstay of therapy in ankylosing spondylitis (AS).
(u) B. Methotrexate is beneficial in treatment of rheumatoid arthritis, but there is little evidence that it changes the
course of disease in ankylosing spondylitis.
(u) C. Oral prednisone may be used sparingly in ankylosing spondylitis, but is less useful for this condition because of
its association with osteoporosis.
(u) D. Sulfasalazine is potentially useful in the treatment of spondyloarthropathies but studies suggest that it is
minimally effective in patients with axial disease.
Ref: (27)

78

218. Health Maintenance/Pulmonology


A mother of a newborn infant presents to the office concerned about reducing the risk of sudden infant death
syndrome (SIDS). The infant was delivered at 39 weeks gestation weighing 7 pounds 9 ounces. There is no family
history of SIDS and this is her first child. Which of the following is appropriate advice to reduce the risk of SIDS?
A. Bottle feeding with soy formula
B. Offer a pacifier at nap and bedtimes
C. Have the infant sleep in the prone position
D. Infant should sleep with the parents to allow close observation
Explanations
(u) A. Bottle feeding with soy formula does not reduce the risk of SIDS.
(c) B. Use of a pacifier during sleeping is a current recommendation to decrease the risk of SIDS.
(u) C. Sleeping prone has been consistently shown to increase, not decrease, the risk of SIDS.
(u) D. Bed sharing with parents has been shown to increase, not decrease, the risk of SIDS. This risk is increased in
infants less than 4 months old and when older children are also present in the bed.
Ref: (5)

219. Clinical Intervention/Cardiology


Which of the following treatments will most benefit the diabetic patient with two vessel coronary disease?
A. Stent placement
B. Percutaneous balloon angioplasty
C. Medical management
D. Coronary artery bypass graft
Explanations
(u) A. See D for explanation.
(u) B. See D for explanation.
(u) C. See D for explanation.
(c) D. CABG is the treatment of choice in a diabetic with two or three vessel disease.
Ref: (11)

220. Clinical Therapeutics/Urology/Renal


Which of the following agents can be used as a urinary analgesic?
A. Phenazopyridine (Pyridium)
B. Oxybutynin (Ditropan)
C. Finasteride (Proscar)
D. Imipramine (Tofranil)
Explanations
(c) A. Phenazopyridine is a urinary tract analgesic used in the treatment of urinary tract discomfort.
(u) B. Oxybutynin is an antispasmodic and anticholinergic used in the treatment of overactive bladder.
(u) C. Finasteride is an alpha-blocker used in the treatment of benign prostatic hypertrophy.
(u) D. Imipramine is an anticholinergic used to treat childhood enuresis.
Ref: (28)

221. Diagnosis/Pulmonology
An infant born at 30 weeks' gestation begins to have respiratory difficulty shortly after birth. Examination reveals
rapid, shallow respirations at 80 per minute with associated intercostal retractions, nasal flaring and progressive
cyanosis. Chest x-ray reveals the presence of air bronchograms and diffuse bilateral atelectasis. Which of the
following is the most likely diagnosis?
A. Respiratory distress syndrome
B. Spontaneous pneumothorax
C. Transient tachypnea syndrome
D. Meconium aspiration syndrome

79

Explanations
(c) A. Respiratory distress syndrome (hyaline membrane disease) is the most common cause of respiratory distress
in a premature infant. This diagnosis is supported by the chest x-ray findings of air bronchograms and diffuse bilateral
atelectasis, causing a ground-glass appearance.
(u) B. Although spontaneous pneumothorax will present with respiratory distress at birth, the chest x-ray would reveal
findings of lung collapse.
(u) C. While transient tachypnea syndrome also may present at birth with respiratory distress, the chest x-ray would
reveal findings of increased pulmonary vasculature markings, perihilar streaking and fluid in the interlobular fissures.
(u) D. Meconium aspiration syndrome usually occurs in term or post-term infants. Typical chest x-ray findings include
patchy infiltrates, coarse streaking of both lung fields, increased anteroposterior diameter and flattening of the
diaphragm.
Ref: (5)

222. Health Maintenance/Orthopedics/Rheumatology


A 23 year-old male presents to the emergency department with severe right shoulder pain. He is holding his shoulder
in internal rotation. Which of the following would increase your suspicion that he has a posterior glenohumeral
dislocation?
A. History of a direct posterior blow to the humerus
B. History of seizure
C. Palpable mass in the anterior axilla
D. Palpable space beneath the acromion
Explanations
(u) A. This is a common mechanism for anterior glenohumeral dislocation.
(c) B. Tonic-clonic seizures can create severe contraction of the internal rotator muscles of the shoulder.
(u) C. This is a classic finding associated with anterior dislocation. The mass is the humeral head outside of the
glenoid.
(u) D. This is also a classic finding associated with anterior dislocation and the space is where the humeral head
should be in a non-dislocated shoulder.
Ref: (26)

223. Clinical Therapeutics/ENT/Ophthalmology


A 24 year-old patient presents after a recent vacation. He complains of left ear pain. Physical examination reveals an
inflamed external auditory canal and the tympanic membrane can not be visualized. Which of the following is the
most appropriate treatment?
A. Oral penicillin
B. Ciprodex otic drops
C. VoSoL drops
D. Oral prednisone
Explanations
(u) A. Oral antibiotics are not indicated in acute otitis externa.
(c) B. Topical steroid/antibiotic drops are most useful in acute bacterial otitis externa.
(u) C. VoSoL drops are utilized as preventive treatment of otitis externa, but are not effective for treatment.
(u) D. Oral prednisone is not indicated in the treatment of acute otitis externa.
Ref: (28)

80

224. Scientific Concepts/Gastrointestinal/Nutritional


A 33 year-old patient returns from a community picnic that was held on a hot July day. The patient developed severe
explosive diarrhea, nausea, vomiting and abdominal cramps six hours after having homemade potato salad and hot
dogs. Multiple people who ate at this picnic have similar symptoms. There is no associated fever. What is the most
likely causative organism for these symptoms?
A. Clostridium perfringens
B. Campylobacter jejuni
C. Shigella
D. Staphylococcus aureus
Explanations
(u) A. Clostridium perfringens has a 12 to 24 hour incubation period and symptoms are limited to diarrhea and
crampy abdominal pain but nausea and vomiting are unusual.
(u) B. Campylobacter jejuni, the most common cause of acute bacterial diarrhea, has a prodrome period of 12 to 24hours with headache, fever, and malaise that is followed by crampy abdominal pain and diarrhea.
(u) C. Shigella infection causes bacterial dysentery in tropical regions most commonly and transmission is usually
from person to person rather than from food-borne spread. Nausea and vomiting are unusual.
(c) D. Staphylococcus aureus is an acute illness caused by the ingestion of enterotoxin-producing organism. It is the
only common acute ingestion of a preformed toxin. The clinical picture in the question is the classic presentation for
this illness. Foods most responsible for this infection are ham, cream, custard pastries, and mayonnaise-based
salads.
Ref: (11)

225. Diagnosis/Cardiology
A 24 year-old male comes to the clinic with a one week history of pain and swelling that involves the entire right upper
extremity. He exercises frequently and has noticed the pain worsening while lifting weights. Examination shows
enlarged cutaneous veins over the right anterior chest wall with a palpable cord. His right hand appears dusky. Which
of the following is the most likely diagnosis?
A. Axillary-subclavian venous thrombosis
B. Thromboangiitis obliterans
C. Superficial thrombophlebitis of the cephalic vein
D. Brachial artery occlusion
Explanations
(c) A. Axillary-subclavian venous thrombosis can occur in someone who strenuously exercises, has had a central
venous catheter or history of venous thrombosis.
(u) B. Thromboangiitis obliterans involves the arteries, although the smaller veins can be included and is linked to
tobacco use.
(u) C. This presentation is not consistent with superficial thrombophlebitis and there is no history of varicosities or
previous IVs.
(u) D. Symptoms are not consistent with brachial arterial occlusion.
Ref: (11)

81

Physician Assistant Education Association


PACKRAT

Comment Form
Please fill out this form and return it to: Physician Assistant Education Association
300 N. Washington St., Ste 505
Alexandria, VA 22314
YES

NO

1. I found the PACKRAT a useful study tool.


Comment:

2. I was able to understand the directions in the Directions and Explanations.

Comment:

3. The feedback provided was easy to understand.


Comment:

4. I know where my weaknesses lie and where to focus my studies.


Comment:

5. The content of the examination was comprehensive.


Comment:

82

6. The examination questions were clearly worded.


Comment:

7. Suggestions for improvement:


_________________________________________________________________
_________________________________________________________________
_________________________________________________________________
_________________________________________________________________

83

You might also like